Vous êtes sur la page 1sur 151

1) For this question take A-1,B-2,C-3..........X-24,Y-25,Z-26. Edwin is a judge and a numerologist.

He is married to a woman whose name: a) has a "product" that is the same as that for JUDGE:using the correspondence of letters and numbers above, this product is 10*21*4*7*5. b)has no letter in common with JUDGE. c)Has no third letter of the alphabet because 3 is his unlucky number. d)Has letters in alphabetical order when the first letter and second letter are interchanged. What is the name of the judge's wife? 2) The owner of the house has been murdered. The visitors to the mansion were Allen,bixby and Crain. a) The murderer ,who was one of the three visitors ,arrived at the mansion later than atleast one of the other two visitors. b)A detective,who was one of the three visitors,arrived at the mansion earlier than atleast one of the other two visitors c)The detective arrived at the mansion at midnight. d)Neither Allen nor Bixby arrive at the mansion after midnight e)the earlier arriver of Bixby and crain wasnot the detective f)the later arriver of allen and crain was not the murderer Who was the murderer? 3) some farm tools and farm equipments have been stolen..3 farm workers were interrogated..Each one makes 2 statements.(i dont remember the statements)One of the statements made by each of them is right while the other is wrong. We have to tell who stole them. 4) There is 4-digit number..the second digit is 2 more than the first..the last digit is five more than the third..something like this..it is very simple..jus have to go from the choices given.. 5) A B C G D E F

Each of seven digits from 0,1,2,3...8,9 is : a) represented by a different letter in the figure

above. b) positioned in the figure above so that A*B*C, B*G*E and D*E*F are equal. Which digit does G represent? 6) Given the names of four persons..and also given that one is taller than the other but shorter than someone else..statements like these..we have to find who is the shortest.. 7) A man is walking inside the tunnel AB. when he is 3/8th of the distance from A, he hears a train coming from behind him..he sees that when he moves towards B, the train just catches him at B, and similar logic if he tried to reach A..the man walks at a steady rate of 8m/h..what is the speed of the train? 8)Fifty minutes ago if it was four times as many minutes past four o clock, how many minutes is it to six o clock? 9) a simple problem related to ages.. 10) There are 5 dacoits who have together stolen some gold coins..they stay at a place for the night hiding the bounty safely...When evreyone's asleep, one of the dacoits wakes up, takes 1/5th of the bounty plus one for himself..the rest he puts back in the bag and goes to sleep..the next dacoit wakes up, does the same thing,ie takes 1/5th of the remaining bounty plus one for himself, puts the rest back and goes to sleep..the third,fourth and fifth dacoits do the same thing..in the morning, they all wake up and divide the bounty equally among them..how may coins were there originally? The key to this question is to proceed from the options..see which number leaves a remainder 1 when divided by 5 four times continuously and the fifth time divides it exactly. A boat was in the sea and sails about 25kmph.The time to travel the distance between the boat and point A in shore is 5 min.Another point B along the shore takes 20 min to walk.Find the minimum time taken to reach the point B from the Boat. 3. Puzzles a. You have 3q and 5 q jars.How many min number of steps will take to get 4q of water. b. How many times a clock coincides its hour and min hands together in 24 hrs. 4.series probs B,D,G,K,P,______

1) a person losses and gains 10% on selling a object for 200. ans:he losses 2)sum of 1 to 100 is divisible by 1.1,2,4,8 2.2 & 4 3.2 4.none ans: 2 3)10 consonants and 4 vowels , how many words with 3 consonants and 2 vowels? 1) 720 , 2)7200 ... complete series 4)AZX.... 5)ABCEFGIJK......( ver easy ) 6)how many nos start and end with 2 b/w 100 and 300? 7)if a sphere of dia 3 cm is melted & formed into 3 spheres , the diameter of 1st is 1.5cm and that of second is 2.0 cm , what is the diameter of the third?? 8)5 logical reasoning like :: All elephants are trained a few animals are trained ..........etc -> refer to IMS material for this 9)there are 6 steps from 1st floor to 2nd floor A si 2 stes below C B is next to D ONly one step is vacant NO 2 people are on any step Q!)if a is on the first step , which of the following are true (( very very easy )) 10) there are 4 novels like thriller , mystery , romance , fiction written by ,a,b,c,d not necessarily in the same order and published by e , f , g , h, not necessarily in the same order...and questions on this (( very very easy )) 11)6 people compete in a race A,b,c,d,e,f b is not in the 1st place there are 2 runners b/w d and e a is ahead of d ( one more condition ) and 2 quesitions --very easy 12)if prizes are increased by 25% , by how much should i reduce the consumption to keep the expenditure same?? 13)how many factors ( or what nos) divide 6400 1)24 , 2) 25 3) .... 14) "COURTESY" - how many words can be constructed with C in the begining and Y at the end

15) My mother's husband's father in laws son's child->what is the relation? 16)3 glasses containing mixture of water and alco in ratio 2:3 , 3:4 , 5:9 when all 3 are mixed what is the new ratio?? 1)Every station in N railroad issues every other station's ticket. Some stations are added. Now they have to issue 46 more tickets. Give the No. of stations after and before added. (5 marks). 2)There was a race between 3 ppl; Me, Doug and Anne. When i take 21 steps the distance covered is equal to Doug's 24 steps and Anne's 28 steps. I take 6 steps to every 7 steps of Doug and 8 steps of Anne. Who won the race? 3)How many bearers will an explorer need for a 6 day march if each man can carry the food stuff of one man for 4 days. 4) Consider the foll statements: Albert: Dave did it. Dave: Tony did it. Gug: I did not do it. Tony: Dave lied when he said that i did it. (a)If only one out of all above statements is true, who did it? (b)If only one out of all above statements is false, who did it? 5) A contribution of Rs. 500 was raisedfrom 500 people. The fee was as follows: Men: Rs.3.00 each Women: Rs. 2.00 each Childern: 0.48 each If number of women is more than number of men, how many childern are there? (5 marks) 6) Alice and Liu had some berries. The total of Alice's berries and square of number of berries with Liu is 62. The total of Liu's berries and square of number of berries with Alice is 176. How many berries does each of them have? (6 marks) 7)A rope ladder was left down from a ship. 12 steps of the ladder were exposed at 10:00 am. The queen who was going to visit the ship, said she would visit at 1:00 pm as she would have to climb lesser number of steps then. The tide in the sea increases from morning to afternoon at the rate of 1.2 meters per hour. The distance between any 2 steps of the ladder is 0.4 mts. How many steps will the queen have to climb?

8)5 hunters Doe, Deer,Hare,Boarand Row kill 5 animals. Each hunter kills an animal that does not corrspond to his name. Also each hunter misses a different animal which again does not correspond to his name. a) The Deer is killed by the hunter,known by the name of the animal killed by Boar. b) Doe is killed by the hunter, known by name of animal missed by Hare. c) The Deer was disappointed to kill only a Hare and missed the Roe. 9) there was 1 more on moon gravity 1/6, weight ... or something... don't remember. A difficult one. Needed some knowledge of physics i guess. Altogether there were 11 questions to be completed in 1 hr. English test was simple for 30 mins - 1 comprehension & other fill in the blank kinds, qts some sentence correction qts. 1. A man was travelling to a place 30 miles away from starting point. he was speeding at 60 miles/hr. but when he came back, his car got breakdown and half an hour was wasted in reparing that. altogether he took 1 hr for return journey. Find the avg. speed of the whole journey. 2.I'M NOT VERY SURE REGARDING THIS QUESTION. A detective was assigned to generate a code using 4 digits, so that no one could break it. he knew that if the code starts with 0,5 or 7 it will be cracked. so how many numbers can be formed using 4 digits. 3. A cow was standing on a bridge, 5feet away from the middle of the bridge. suddenly a lightning express with 90 miles/hr was coming towards the bridge from nearest end of the cow.seeeing this the cow ran towards the express and managed to escape when the train is one feet away from the bridge. if it would have ran to opposite direction(ie away from train) it would have been hit the train one ft away from the end of the bridge. Calculate the length of bridge. 4. there are 3 towns attacked by 3 dragons-x,y,z. Number of days x attack a town is equal to number of days y attacking another town. Number of days x attack is equal to half the square root of number of days z attacking a town.number of days y attacking the town is twice the square root of z.calculate how much days the curse of each dragon be. 5. 5 Boys and 5 Girls went for a movie. the problem is based on their seating. analytical one and 3 or 5 sub questions were there on this question simple one. 6. A town have a population of 500000 and 42% of males and 28% of females are married to same town. find the total number of males 7. An analytical one based on 4 persons and their occupation

8. another analytical one 9. A and B came back home after their exam and their father asked them about the test. A replied-- 1/3rd of my answers were wrong B replied-- 5 of my answers were wrong but together we got 3/4 of answers right. How many questions were there for the exam? 10. if x,y<0 and z>xy then which condition is 4 conditions were given and one condition is identify that. 1. If A can do a particular work in 8 days. B days. By C joining with them, they can finish If Rs.540 was paid to them, what is the share false wrong. you need to can do the work in 6 the work in 3 days. of C?

2. By selling a product a seller is gaining 10% on selling price. If he sell that product Rs.80 less, he would have been losing 10%. What is the cost price? 3. If A dricer drives a car four times a lap 10,20 30,60 kmph what is his average speed. 4. If M persons are going for a party, then their share is $D. If three persons were absent, then how much share will be increased for each individual. (Solution : Average = D/M. Three absent New average = D/M-3 Increase = Newaverage - Average = D/M-3 - D/M ) 5. If the radious is increased by 50%. How much will be the volume increased for a circle / Sphere in percentage. 6. One person is wearing Black and White shoes as his pair. If he is having 20 white and 20 Black shoes. What will be the less number of time he have try to take his correct pair of shoes. 7. The age of the grand father is the sum of his three grandsuns. The second is 2 year younger than first one and the third one is 2 year younger than the second one. Then what will be the age of the grandfather. 8. in a grass field, If there are 40 cows, they could eat for 40 days. If there are 30 cows, they could eat for 60 days. Than if 20 cows, How much day they could eat? 9. If the size of the fruit is two times the size of the seed. Then what is the volume of the seed. (derive general equation) 10. Speed of boat in still water 10 kmph.If the boat takes 24hr in up stream and 16hr in down stream to cross the same distance, then what will be the distance covered. 11. How many times do the hands of the clock coincide in a day? 12. If a person is standing in room, that is having mirrors in all the four walls. How many images of the person will appear.

<http://www.businessworldindia.com/sep01/coverstory01.asp> Opening Essay <http://www.businessworldindia.com/sep01/cover_amatter.asp> Column: Bob Levering <http://www.businessworldindia.com/sep01/cover_top25.asp> The Top 25 <http://www.businessworldindia.com/sep01/cover_no01.asp> No.1: Texas Instruments <http://www.businessworldindia.com/sep01/cover_no02.asp> No.2: Federal Express <http://www.businessworldindia.com/sep01/cover_no03.asp> No.3: Johnson & Johnson Consumer Products <http://www.businessworldindia.com/sep01/cover_no04.asp> No.4: Eli Lilly and Company India <http://www.businessworldindia.com/sep01/cover_no05.asp> No.5: Philips Software Centre <http://www.businessworldindia.com/sep01/cover_no06.asp> No.6: Godrej Consumer Products <http://www.businessworldindia.com/sep01/cover_no07.asp> No.7: WiproSpectramind <http://www.businessworldindia.com/sep01/cover_no08.asp> No.8: Nokia India <http://www.businessworldindia.com/sep01/cover_no09.asp> No.9: Birla Sun Life Insurance <http://www.businessworldindia.com/sep01/cover_no10.asp> No.10: Cadbury India

<http://www.businessworldindia.com/sep01/cover_no11.asp> No.11: Aviva Life Insurance <http://www.businessworldindia.com/sep01/cover_no12.asp> No.12: Tata Teleservices <http://www.businessworldindia.com/sep01/cover_no13.asp> No.13: NIIT <http://www.businessworldindia.com/sep01/cover_no14.asp> No.14: Ernst & Young SSL Division <http://www.businessworldindia.com/sep01/cover_no15.asp> No.15: Marico Industries <http://www.businessworldindia.com/sep01/cover_no16.asp> No.16: AV Birla Group <http://www.businessworldindia.com/sep01/cover_no17.asp> No.17: Bharat Petroleum Corporation <http://www.businessworldindia.com/sep01/cover_no18.asp> No.18: Hughes Software Systems <http://www.businessworldindia.com/sep01/cover_no19.asp> No.19: Infosys Technologies <http://www.businessworldindia.com/sep01/cover_no20.asp> No.20: Max New York Life Insurance <http://www.businessworldindia.com/sep01/cover_no21.asp> No.21: Dr. Reddy's Laboratories <http://www.businessworldindia.com/sep01/cover_no22.asp> No.22: Wipro <http://www.businessworldindia.com/sep01/cover_no23.asp> No.23: Tamil Nadu Newsprint & Paper <http://www.businessworldindia.com/sep01/cover_no24.asp> No.24: Anand Group

<http://www.businessworldindia.com/sep01/cover_no25.asp> No.25: Jindal Iron & Steel Company

http://www.rediff.com/money/2003/aug/23employers.htm MNCs best employers in India Meenakshi Radhakrishnan-Swami in New Delhi | August 23, 2003 10:52 IST Godrej Consumer Products, Wipro Spectramind and Birla Sun Life Insurance are the only Indian companies to feature among the top 10 in a survey on the best places to work in India. The other seven are multinational firms. "Great Workplaces in India" is the local version of the "Great Places to Work" survey carried out every year in 22 countries. This is the first time that India has been surveyed. At the top of the list is Bangalore-based Texas Instruments, where

employees' pride in their co-workers was noteworthy. The company also won points for employee benefits, policy on sabbaticals and overall professional development. The number two and three positions were taken by FedEx and Johnson & Johnson, respectively. FedEx's inward focus -- 59 per cent of managers in India have been promoted from within -- and Johnson & Johnson's minimal 'level-linked' benefits won praise from employees. These three companies are followed by Eli Lilly, Phillips Software, Godrej Consumer Products, Wipro Spectramind, Nokia, Birla Sun Life Insurance and Cadbury's. In India, the survey has been conducted by Grow Talent, a talent management consulting firm based in Gurgaon, in collaboration with Businessworld magazine and the US-based Great Place to Work Institute, which pioneered the survey in the US over 20 years ago. The results of the American and British surveys are published in Fortune and Financial Times, respectively. All organisations -- private and public companies, government agencies and non-government organisations -- that have been in existence for at least two years and have a workforce of over 100 employees are eligible to participate. Over 120 companies were part of the survey, which was started in February. The methodology and instruments of the survey in India did not differ from those used worldwide. Questionnaires were distributed at random to about 250 employees at each of the participating organisations. "The survey helps identify the strengths of companies, which they can then transform into something more visible," said Tushar Makkar, head, brand management, Grow Talent. Independent analyses conducted in the US show that a good workplace also means decent returns to the stakeholders. In 2001, annualised stock market returns of the American 100 best companies stood at around 30 per cent for a three-year period; the S&P500's annualised returns for the same period was 11 per cent. Similarly, according to research carried out by Franklin Research and Development in 1997, $1,000 invested 10 years earlier in the 100 best companies returned $8,188, against a return of $3,976 for $1,000 invested in the Russell 3000. Clearly, being a great place to work isn't good only for the employees. What matters * Employee trust in management, clarity of vision. * Balance between performance management, compensation and employee care. * _____ TCS, HP, Infosys among best 5 employers: survey : SATYAPRAKASH SINGH : TIMES NEWS NETWORK[ FRIDAY, AUGUST 22, 2003 07:44:37 PM ] : Focus on employee development and family.

BANGALORE: Cadence, TCS, HP, Infosys and IBM have emerged as the best five employers in the IT industry, according to DQ-IDC 'Best Employer Survey 2003'. <http://203.199.70.171/cgi-bin/adceptclickthrough.cgi?random_key=25/7/20 0311:0:530.1824391807803209&host_name=economictimes.indiatimes.com&url_n ame=www.timesofindia.com/cms.dll/html/uncomp/articleshow> Infosys goes down the best employer rankings for the first time, but remains the dream company for IT employees in the country. Most of the effects of recession on the HR environment in the country seem to recede. Fewer people complain of cut in salary increments and perks but morale is not looking up yet. "The third DQ-IDC Best Employers Survey of 2003 threw up some interesting results. For the first time Bangalore based Infosys Technologies fell from its no. 1 position to be replaced by the offshore development centre of EDA tools giant, Cadence Design Systems,' says a release from IDC. Also up in the rankings were IT services major Tata Consultancy Services at no. 2 and Hewlett Packard at no. 3. This is the first full year of the new HP, which in May 2002 went through a major international merger with Compaq Systems. IBM enters the survey and the rankings for the first time at no. 5. In addition, seven new companies made it to the Top 20 Best Employers list for the first time including - Computer Sciences Corporation, Siemens Information Systems, WeP Peripherals, Oracle India, RMSI and Chennai-based Accel ICIM Systems and Services. _____ Luv, Suri "If loving you is wrong, I don't want to be right" WebSite @ http://surionline.topcities.com Note: Since it's a free WebSite, it will be automatically down when the number of visitors per day exceeds 600. In such case, try to visit after 1 or 2 days. You will be able to view it.

[This message contained attachments]

________________________________________________________________________ ________________________________________________________________________ Message: 3 Date: Mon, 25 Aug 2003 18:17:49 -0000 From: "prasadg_foru" <prasad_group@hotmail.com>

Subject: (Query) Help regarding Axes interview dear friends, if any body have attended HR interview in axes please help me in the same. i have been told by axes that i have been selected for HR interview and the call might come for me in the next week. please help me in this for me. how many attended with u and how many of them got selected. did u get the call letter on the same day. pls if possible give ur contact number. what type of question they asked you.please help me in this regard as soon as possible.this is a very humble request.i once again request u to do this favour to me. i am requesting so much becos this is my first HR interview,so pls help me.I will be very much thankful to u. prasad g 958153-351166

________________________________________________________________________ ________________________________________________________________________ Message: Date: From: Subject: 4 Mon, 25 Aug 2003 19:23:22 -0000 "Archana A." <archana_achoo@yahoo.co.in> (Paper Pattern) Syntel

Question pattern of SYNTEL test conducted at COCHIN(SHREDS) ON 23-82003: section I: 25 questions (ENGLISH) in 15 minutes.. section II: 45 questions (QUANTITATIVE) in 45 minutes.... section III: 30 questons (REASONING) in 30 minutes..... we were first asked to do section II and then section I..... SECTION II QUANTITATIVE (TIME IS A SEVERE CONSTRAINT)

Qn 1.) (this was the first question) Maximum numbers that can be formed using all the 4 digits 6 4 8 1 without repetition and which is divisible by 9.( ans none) 2.) Find the number of sides of a regular convex polygon whose angle is 40degrees. 3.) a+b+c=0, then roots of ax^2+bx+c=0 is

1.imag

2.real

3.coincidental

4.zero

4.) Difference b/w the compound interest and simple interest for Rs.2500 for 2 years is given-----. find the rate of interest. 5.) there was one more question on S.I and C.I 6.) the minimum number by which 60 is to be multiplied to generate a square. ans 15 7.) A monkey climbs 6 mts and falls 3mts in alternate minutes.Then time taken to climb a tree 60metres high? a. 35 b.37 c.32 d.34 (think the answer is 37) 8.) (This was the second last question) A bucket contains z drops. and it leaks x drops in t secs.then the time required to empty the bucket(in minutes)? 9.) 6 pipes fill or empty the cistern. find the number of emptying pipes iff it takes 18hrs to fill and 18 hrs to empty.... (don't remember the question exactly) 10.) the largest no: which is a factor of 1080 and 729

11.) No: of spheres of radius 1 that can be got from sphere of radius (or diameter don't recall) 8 12.) (think the last but three question)Travelling at 3/4th the speed a man is 20 minutes later then speed is?? 13.) there are 6 keys and 6 locks. then number of combinations to be tried out to get the actual solution a. 5^6 b.6^5 (don't remember the rest) 14.) choosing 2 people out of 10 in how many combinations can a particular person(some name) be always included.... 15.) from 6 white balls and 7 black balls probability that 2 balls drawn at random are of the same color? 16.) if a sales man gets successive gain of 15% and 20% then his actual gain? ans. 38 17.) a string of pearls such that 1/3 is lost and of that 1/4th is missing, remaining is 20 then actual number of pearls? ans. 40 18.) a man gets a gain of x%. but if he had sold at twice the cost price, what will be his gain?(question not sure) a. 2x b.200-2x c.100+x (not sure of the options) 19.) a clock was 7mts behind the actual time on 3 p.m. on wednesday and 8 mts ahead of actual time on (not sure) 4 p.m. friday. when will it show the correct time?

20.) boat moves upstream in 6 hrs and covers the same distance downstream in 5 hrs. then speed of a raft floating?(accuracy of question not sure) 21.) (this was the last question) no idea what it stands for........ some kind of notation like S(P(M((D(a,b),2))):P(M(S(D(a,b),........ options were 1. ab 2.(a-b)^2 3.(a+b)^2 4.none 22.) if x men working x hrs per day can do x units of work in x days, then y men working y hrs/day would be able to complete how many units of work i y days? ans. y^3/x^2 (question in R.S. Agarwal) 23.) ( this was a question in the first page of the section II booklet) a cone with radius----- and height -----. a hemisphere covers the cone such that base of hemisphere meets that of the cone. then the enclose volume.....(R.S. Agarwal consists of similar questions) 24.) there was one more question on volume and surface area..... 25.) 1 Rs, 50 ps , 25 ps coins are in the ratio ---------, then the number of 50 ps coins if they sum to ------Rs. (similar question in R.S.Agarwal) 26.) there was one more question on coins i.e. abt getting a change of 10ps and 25 coins for ------Rs.(how many possible combinations or so possible) 27.) x/y+y/x=40/21(don't remember the exact value, believe this is the one) find x and y there were 2 questions on train and one was like: 28.) a goods train starts and after 2 hrs a passenger train at 4km/hr starts and overtakes the goods train after 4 hrs, then the speed of goods train? 29.) 15hrs of boys work=6 hrs of women's work. 3/5 of the work is done by -----boys and -----women. How much time would be taken by the women to complete the work? 30.) using false weight a man gains -----%, for a k.g. then he uses a weight of.....(this kind of question has some formula that can be found in R.S.Agarwal) 31.) there was one question on L.C.M. and co primes(don't remember the question) 32.) there was one question on triangle 33.) a figure was given a square with four corners shaded and asked to find the area of the shaded portion.... ie area of square-area of the regular octagon.....

This is all i can remember from section II SECTION I ENGLISH (EASY) PATTERN: 1.) few questions on finding out the segment of sentence which is wrong.... 2.) analogy. (few i can remember) 1.) celebrate::marriage ans. lament::bereavement 2.) bouquet::flowers ans. chain::link 3.) germ::disease ans. war::destruction 4.) (remember only the answer) ans. book::pages

3.) to fill up the blanks in the sentences with the most appropriate words..... 4.) to find the word opposite in meaning (few qns i can remember) 1.) equanamity 2.) sequester 3.) dormant 4.) apathetic

SECTION III REASONING (TIME IS A CONSTRAINT) 1.) (last question and the easiest) a simple flowchart whose steps are as follows. value of r and h are given, a and b=0 m=pi*r*r*h n=(pi*r*r*h)/3 if (m==n) do ------else b=4*pi*r*h print a and b [DON'T REMEMBER IF conditon is m==n or m<n] 2.) (second last question this was also easy) it was abt a set of people talking 4 languages and then qns were abt who could act as a translator to whom ........ 3.) there were 3 questions based on an argument given... questions were to find out which statement in the objectives given could strengthen the argument or to weaken the argument....(bit confusing) 4.) blood relationship question( i remember all the statements given, though not the question)

10 people in a family A B C are husbands D E F are wives altogether there are only 4 women. each family can have atmost 2 children and C has atleast 1 child. D and G are related to A. E cannot be related to H and I and J are not related..... 5.) (this consists of the most confusing set of questions) a set of statements are given and then questions were asked, the statements were like All A B C D E F are Q's All A's are B's -------------(better leave this question for the last) 6.) Alice works on Monday, Wednesday and Friday B does not work on Wed C works only on Tuesday and someotherday(don't remember) D does not work on Friday E works on all days except on the first Monday and Thursday then questions like who all will be available on which days ..... 7.) question based on venn diagram given ......read the question properly and then answer 8.) (first question) abt 4 buses with different seating capacity.and different charges. and questions were to find out the minimum number of trips required, the maximum charge, the second highest charge etc.... 99999999999999999999999999999999999999 one more thing: though they said abt 50% cut-off for each section, for us THEY HAD CONSIDERED THE TOTAL(OUT OF 100) only AND NOT THE CUT-OFF...... seems they have changed their processing technique......don't think they are are going to change the questions atleast for quite a while.....so this being the latest set, those preparing for syntel, i would suggest u to peruse through the questions and be ready to answer them.... remember rate of answering counts in a lot for this test...... this is all i can recollect........HOPE THIS PROVES TO BE HELPFUL FOR ALL THOSE ASPIRING FOR SYNTEL................................... AND BEST OF LUCK FOR THE SAME!!!!!!!!!!!

________________________________________________________________________ ________________________________________________________________________ Message: 5 Date: Mon, 25 Aug 2003 14:01:57 -0700 (PDT) From: Rizwan <rizwan_ks2003@yahoo.com>

Subject: (freshers) CDAC Noida > > > > > > > > > > > > > > > > > > > > > > > > > > > > > > > > > > > > > > > > > > > > > > > > > > > > >

Dear Friends, Check the following requirement. Please don't send me any resumes. luv, Riz. 1. CDAC, Noida Dear Applicants, We have requirement for position as Project Engineers in CDAC Noida. The qualification requirement is as follows: BE (CSC/ ECE/ Electrical/ Mech), MCA, MCM with 60% marks in final year and above. All the papers should be cleared by/on 31st Dec 2003. The age of the candidate should not be over 26 years and should not be a passouter of 1998 or before. Freshers can also apply. Send your resumes in the following format only otherwise resumes will be rejected: Subject line: Qualification/branch/marks/year of graduation Send on the following email address : To: vnshukla @ cdacnoida.com Cc: munishkumar@c..., poonamgupta@c... Make sure that you mail your resume to all the three mail id's by 26th Aug 2003 You can also send hardcopies to : V N Shukla, Anusandhan Bhawan, CDAC Noida,Sector 62 Noida, UP. V N Shukla CDAC Noida

> 2. Idea Cellular Limited > > IDEA Cellular Limited, one of the leading cellular > services provider in India, with operations in the > Circles of Maharashtra (except Mumbai), Gujarat, > Andhra Pradesh, Madhya Pradesh and Delhi requires > > Designation > Graduate Trainee Engineers > > Desired Profile > B.E. - Computer/Electronics & Tele Com./Electrical > engineering from premier institutes with throughout > 1st class. > > Remuneration > As per company rules > > Location > Pune > > Reference > GET - Computers/Electronics & > Telecommunications/Electrical > > Contact > Vishal Sharma > Asst Mgr HR > Idea Cellular Ltd > > Just go to this link and apply online. > > http://www.naukri.com/alljobs/minnernew.php? cat=22&filename=110803001691 >

--------------------------------Do you Yahoo!? Yahoo! SiteBuilder - Free, easy-to-use web site design software [This message contained attachments]

________________________________________________________________________ ________________________________________________________________________ Message: Date: From: Subject: 6 Mon, 25 Aug 2003 12:37:07 -0700 (PDT) Rizwan <rizwan_ks2003@yahoo.com> (freshers) Computer Associates, Chennai

Dear Friends,

Check out the following one. Sorry, i have got this message late but as it is given in the mail that if u can't make it to the interview u can mail ur CVs so, i think u can still mail ur CVs to the mail id given. Please don't send me any resumes. luv, Riz. Computer Associates, the third largest software products company in the world is hiring freshers as Trainee, Technology Consultant. The walk in will be held in CA's state of the art, Global Technology Support Center at Tidel Park on Saturday 23rd August 2003 between 12 Noon and 7 PM. The criteria for selection is: Smart English Speaking Communicators whose ability to think in English is high Articulate, great presentation skills A positive, can-do attitude and a willing learner Strong academics - a proven track record of high achievement Preferably a BE / MCA / BSc Computer Science. This criteria may be overlooked for candidates who are strong on the other requirements. For the first round you do not need to bring in your credentials, except a cv. The credentials can be submitted for perusal at later stages. Should you not be able to make it for the interview, please send your cv to wannabe@c... indicating a time that is convenient for you. All the very best Bobby

--------------------------------Do you Yahoo!? Yahoo! SiteBuilder - Free, easy-to-use web site design software [This message contained attachments]

________________________________________________________________________ ________________________________________________________________________ Message: 7 Date: Tue, 26 Aug 2003 22:02:43 +0530 From: "Sourirajan Srinivasan" <shourirajan@hotmail.com> Subject: (3+ years experience) Urgent Requirement - Code: CUT/VZN/2308/2003

Please Please Please Please Hi,

send send send send

resume resume resume resume

to to to to

the the the the

concerned concerned concerned concerned

& & & &

not not not not

to to to to

me me me me

... ... ... ...

We are a Leading Placement organization catering to Top IT Companies in India. We have an Urgent requirement for C++, UNIX, Telecom professionals. This requirement is for a Fortune 8 Company for their Offshore Development Center in Tidel Park - Chennai. The compensation and work environment is the best in the industry. Pls let us know your willingness to take up this opportunity. In any case, Pls send us your updated CV at the earliest. As the requirement is a bit urgent, an early response from you would be highly appreciated. The requirement is enclosed below: Designation : Software Engineer/ Sr. Software Engineer Qualification : Engineering in CS/EC (PG/UG) / MCA Experience: 3 6 years Experience

Skills: C++, UNIX (Shell Scripting), Knowledge in Telecom Protocols like ATM, Frame Relay etc. We have a requirement of 20 position for the similar skill-set. Please refer your friends also for this opportunity. Looking forward to your immediate and positive response. Note: Pls do not delete the subject line in your response mail. Regards, R. Sriram BINOTECH Private Limited # 32, Karpagambal Nagar, Mylapore, Chennai - 600 004 Phone: +91 44 2498 8164 / 65 Email: binotech@vsnl.net _________________________________________________________________ Dress up your desktop! Get the best wallpapers.

http://server1.msn.co.in/msnchannels/Entertainment/wallpaperhome.asp Just click here!

________________________________________________________________________ ________________________________________________________________________ Message: Date: From: Subject: 8 Mon, 25 Aug 2003 12:37:46 -0700 (PDT) Rizwan <rizwan_ks2003@yahoo.com> (freshers) some requirements

Dear Friends, Check out the following requirements. I am neither a HR nor resumes. a consultant. So, please don't send me any

luv, Riz. 1. Details of Vacancy

Position Vacant : Fresher Software Developers Company Name: A Client of NIHT Software India Ltd. Company Profile: Our Client Company is a leading software company engaged in the creation, development of the industry's most advanced technologies, including computer systems, software, networking systems, storage devices Job Description : Websphere appliaction server (WAS),* Websphere Commerce server (WCS) Professional edition Ver 5.4,,PS: Candidate profile : MCA/BE Computetr science with sound knowlwdge of appliction programming Compensation : Best In Industry Location : Delhi/Bbanglore/hydrabad Reference: 250803/swdev/fresh

Contact info : tisha Hr Exec NIHT Software India Ltd. Phone: 9818554560 Fax: 91-120-2515602 E-Mail: jobs@nihtindia.com

Website: http://www.nihtindia.com 2. Details of Vacancy

Position Vacant : Java Freshers 0-1yrs (mumbai- Pune Only) Company Name: Resourcing Hr Consultants Company Profile: A good IT company based out of PUNE. Job Description : Freshers with BE, MCA, MCM, BScTech with technical skill in Java, j2ee having completed diplomas from NIIT, Aptech or other credible institutes Candidate profile : Good communication skills. ONLY FROM MUMBAI and PUNE. (other location cvs will not be entertained)..,Pl send text files preferably Location : Pune Reference: naukri/pune/fresh/dp

Contact info : CFernandes manager Resourcing HR Consultants Phone: 91-022-8311420, E-Mail: resourcing@vsnl.com 3. Details of Vacancy

Position Vacant : Software - Trainee Company Name: JUMPP India Pvt. Ltd. Company Profile: Providing sate-of-art "Electronic Funds Transfer" Solutions & Services that have changed the very perception of Payment Transaction Systems. JUMPP, is a technology leader in the forefront in the Secure Financial Transaction Solutions and Services. Job Description : Responsible in development Application Software of large / medium projects. Candidate profile : Candidate should be BE / BTECH / MCA in Computer Engineering. Should have sound knowledge of C/C++, VC++, Oracle, SQL, JSP / ASP. Compensation : Open to Discussion Location : Delhi

Reference:

ST-1003-103/Naukri.com

Contact info : Jumpp India Pvt. Ltd. Publicies House, 1 - 2 Aram Bag Panchkain Road Delhi, India 110055 Phone: 91-11-23516712, 91-11-24631739 Fax: 91-11-23516722 E-Mail: careers@jumppindia.com Website: http://www.jumppindia.com 4. Details of Vacancy

Position Vacant : Software Professional(Only local candidates) Company Name: Cyberthink Infotech Pvt. Ltd. Company Profile: CIPL, An ISO Co., Affiliated with cyberThink, USA, Branch Office in UK, situated at prominent location of Ahmedabad, India providing consulting, S/W & IT services globally with talented & experienced professional staff & well-equipped infrastructure. Job Description : Knowledge of VB/ SQL with more than six Months of experience of Business Systems.Pl. send resume in word format with current salary. Candidate profile : Person should be BE(IT) or MCA with 6 months of experience OR Any Graduate with two years experience in VB/ SQL. Good oral & written communication skill. Compensation : Negotiable for Right candidate Location : Ahmedabad Reference: VB/SQL-Aug03-01-Naukri.com

Contact info : Cyberthink Infotech Pvt. Ltd. 51, Premier House-1, Sarkhej Gandhinagar Rd, Opp. Gurudwara, Bodakdev Ahmedabad, India 380054 Phone: 91-079-6857501, 91-079-6857502 Fax: 91-079-6857504 E-Mail: jobs@cyberthinkinfotech.com Website: http://www.cyberthinkinfotech.com --------------------------------Do you Yahoo!? Yahoo! SiteBuilder - Free, easy-to-use web site design software

[This message contained attachments]

________________________________________________________________________ ________________________________________________________________________ Message: 9 Date: Tue, 26 Aug 2003 17:48:57 +0100 (BST) From: ramana pallapotula <ramana_pvrk@yahoo.co.in> Subject: (experienced 1yr) requirement on ASP.NET+C#.NET(1 person) Salary(nominal)+hyd candidates only Hi This is Ramana Pallapothula,i have requirement on ASP.NET with C#.NET(1 person), i need the person who has ateleast one project experience on ASP.NET+C#.NET, intial salary is nominal, depends on your performence salary can fix, strictly for the hyderabad candidates only(preferable near S.R.Nagar) eligable candidates call me on 55109767. -Ramana Pallapothula Win TVs, Bikes, DVD players and more!Click onYahoo! India Promos [This message contained attachments]

________________________________________________________________________ ________________________________________________________________________ Message: Date: From: Subject: Hello all Please dont send any resumes to me. Please dont send any resumes to me. Please dont send any resumes to me. This is the paper pattern conducted for the 'Technical and Non-Technical' assistants in ITC Infotech.There is walkin going on regularly there.I got through first 2 rounds,but couldnt make through the 3rd. General Pattern Technical and Non-Technical test Listening comprehension HR and ome more...which i couldnt attend.n 10 Tue, 26 Aug 2003 02:44:47 +0100 (BST) Vishnu T P <iceblue_career@yahoo.co.in> (Paper Pattern) ITC Infotech,Bgore walk-in

Technical and Non-Technical Test This contains a lot of technical questions(mainly windows trouble shooting)and english.They select a major chunk for tech-support based on technical section marks and select some more for non-tech-support based on english. Technical questions which i remember(multiple choice/no negatives) 1) dll stands for : Dynamic Link Library 2) IP Address is:32 bit 3) POP is:Post Office Protocol 4) Protocol used to receive mails: SMTP 5) Protocol to receive pics and all through mails: MIME 6) Outlook express folders are saved with extension: Inbox.dbx 7) What menu do you get when you right click Start button : Open/Explore/Find 8) Can you rename 'My Computer'?(True/False->answer is True) 9) Win95 work without autoexec.bat and config.sys(True/False->dunno) 10) Things you need for taking internet->Answer is all of the above 11) How do you install Fonts in windows?(Control Panel->Fonts) 12) Internet Explorer is a :Webbrowser 13) Program that can interpret HTML:Webbrowsers 14) Port used for serial communication: COM Port 15) ISDN is:Integrated Services Digital Network 16) Organisation which gives provision for internet connection:ISP's 17) There was this question on trouble shooting.Your speaker is not giving any sounds after playing a cd.You are given 4 options to do.Arrange them in the correct checking order.I guess the answer is : *Checking the speaker volume and connection cables *Checking volume control in windows *Opening the system and checking the connections behind CDROM drive *Re-installing the sound card drivers 18) OS which has inbuilt firewall:WinXP 19) OS which has got both NTFS and FAT32 support: WinXP 20) Win2000 supports which all filesystems: FAT,FAT32,NTFS 21) When you move a system physically to another location,then what happens?:change in IP address/change in domain name/both/none 22) URL is:answer is all of the above 23) The OS's which you can install in a system with only one primary partition. 24) General format used for pics in background:BMP 25) Popular motion picture format: MPEG 26) Microprocessor speeds are expressed in :MHz/GHz 27) Command used to partition hdd's: fdisk 28) How do you know the version of OS you are using?:My Comp->Right Click As you can see,the test is very easy.But because of the same reason,cut off's are huge i guess. English

I dont remember any particular questions.It was all fill-in-the-blank type and can be answered easily. Analytical 1)There is this quesion on weight of Susan (!!!).Answer is 70Kg. 2) Question on Venn-diagram-type.Language 'A' is spoken by 40%,Language B is spoken by 50%,10% speaks both,so none of the languages is spoken by how many? 3)Similar question as above 4)You have got equal number of Rs1,Rs 5 and Rs 10 notes.You have Rs 480 with you.Total number of notes you have is:90 5)A question on how much % family expenditure must decrease if the price increases by 25%(answer is 20%). They shortlist something in the range of 30.They will be given a reading comprehension test.All you have to do is to listen carefully to some converstions and answer the questions.They take 10 from this i guess.This will be followed by a "personal introduction" round.Make sure you have something to say on 'why you opted for call-centres" because i made the mistake of not giving a 'genuine' reason.All the best anyways. By the way,if anybody know the info about DRDO Computer Science & Engineering paper pattern,please email them to vishnutp1@rediffmail.com . --Vishnu-"I had the blues because i had no shoes. Until upon the street,i met a man who had no feet"

Win TVs, Bikes, DVD players and more!Click onYahoo! India Promos [This message contained attachments]

________________________________________________________________________ ________________________________________________________________________ Message: Date: From: Subject: 11 Tue, 26 Aug 2003 16:51:50 +0100 (BST) govindadas Pai <govindadas_pai@yahoo.co.in> (Query) Regarding INFY interview for experienced people

Dear Members , I have been a silent member of this group. This group has been of immense help for me. well I wote Infy test on sunday 24th Aug in Chennai. I got selected in the written test and was called for a Interview the next day. Since i had experience of Approx 3 years they asked me a few tech Questions, Oracle related questions etc. At the end

they told me they will come back to me in another 2 to 3 weeks . I just wanted to know whether This the mode being followed or will the candidates will be selected immediately. Can Any of the group members throw some light in this matter. A few words for encouragement from Group members would be of great use regards govind Win TVs, Bikes, DVD players and more!Click onYahoo! India Promos [This message contained attachments]

________________________________________________________________________ ________________________________________________________________________ Message: Date: From: Subject: 12 Mon, 25 Aug 2003 20:21:02 -0700 (PDT) Ganesh S <imsganesh1@yahoo.com> (PAPER) INFOSYS QUESTIONS - 24 - AUG- 2003.

Hi all, This is Ganesh. I attend the Infosys written test on 24th august 2003. I here attached the questions that i faced in the test. These ques are relatively easier compared to other infosys questions. the names that were given here is little bit different. but the logic behind all questions is correct to my knowledge. SO THOSE WHO GOT THE ANSWERS Mail to this group. LETS HAVE A GOOD DISCUSSION regarding the answers for these questions. Totally 9 Questions. - 50 marks.

1) There is a five digit number. The fifth digit is one fourth of the third digit and one half of the fourth digit. Third digit is one half of the first digit. second digit is 5 more than the fifth digit. What is that 5 digit no.? 2) There are three associates and they have some money in a room. But no one is believing others alone. so, a) No one should be able to open the room alone. b) When any 2 persons are combined they should be able to open the room. What is the minimun no. of locks & keys required.? (5M) 3) A simple racing prob. like Samantha beat Jim. Jack lost to Jim. John was not last. And the prob goes like this... (simple) 4) A boy goes to school from his house.on one fourth oh his way to school, he crosses a

machinery station. And on one third of his way to school, he crosses a Railway station. He crossed the machinery station at 7:30 and he crosses the Railway station at 7:35. when does he leave the house & when does he reach the school ? (5M) 5) An Eraser,Pencil,Notebook together costs $1.00. Notebook costs more than the cost of 2 Pencils. 3 Pencil costs more than 4 Erasers. 3 Erasers costs more than a Notebook. How much does a pencil costs? (5M) 6) I can't remember the prob. fully. (* Check this in previous questions. Its there!) Old Problem with three inhabitants of three castes. Abhor,Magar & Dravid. (8M) 1) Abhor women cannot marry Dravid man. 2) Dravid women cannot marry Magar man. 3) Male child gets the father's caste and female child gets the mother's caste. 4) all other marriages are allowed. 5) ... 4 questions related with this...each of 2 marks... like... 1) A magar man can have ... a) a dravid grandmother. b) a abhor mother. c).... 7) Series... a) 3,6,13,26,33,66, ? b) 364, 361, 19, 16, 4, 1, ? 8) A large cube is painted on all sides with Red color. It is then cut into 27 small cubes. How many sides of the cube will have... a) 3 red faces. b) 2 red faces. c) 1 red face. d) No red faces. (8M) 9) Four persons A,B,C,D were there. All were of different weights. All Four gave a statement.Among the four statements only the person who is lightest in weight of all others gave a true statement. A B C D Says Says Says Says : : : : B A I C is is am is heavier heavier heavier heavier than than than than D. C. D. B.

Find the lightest & List the persons in ascending order according to their weights. (5M)

Qn 1.) (this was the first question) Maximum numbers that can be formed using all the 4 digits 6 4 8 1 without repetition and which is divisible by 9.( ans none) 2.) Find the number of sides of a regular convex polygon whose angle is 40degrees. 3.) a+b+c=0, then roots of ax^2+bx+c=0 is 1.imag 2.real 3.coincidental 4.zero 4.) Difference b/w the compound interest and simple interest for Rs.2500 for 2 years is given-----. find the rate of interest. 5.) there was one more question on S.I and C.I 6.) the minimum number by which 60 is to be multiplied to generate a square. ans 15 7.) A monkey climbs 6 mts and falls 3mts in alternate minutes.Then time taken to climb a tree 60metres high? a. 35 b.37 c.32 d.34 (think the answer is 37) 8.) (This was the second last question) A bucket contains z drops. and it leaks x drops in t secs.then the time required to empty the bucket(in minutes)? 9.) 6 pipes fill or empty the cistern. find the number of emptying pipes iff it takes 18hrs to fill and 18 hrs to empty.... (don't remember the question exactly) 10.) the largest no: which is a factor of 1080 and 729

11.) No: of spheres of radius 1 that can be got from sphere of radius (or diameter don't recall) 8 12.) (think the last but three question)Travelling at 3/4th the speed a man is 20 minutes later then speed is?? 13.) there are 6 keys and 6 locks. then number of combinations to be tried out to get the actual solution a. 5^6 b.6^5 (don't remember the rest) 14.) choosing 2 people out of 10 in how many combinations can a particular person(some name) be always included.... 15.) from 6 white balls and 7 black balls probability that 2 balls drawn at random are of the same color? 16.) if a sales man gets successive gain of 15% and 20% then his actual gain? ans. 38 17.) a string of pearls such that 1/3 is lost and of that 1/4th is

missing, remaining is 20 then actual number of pearls? ans. 40 18.) a man gets a gain of x%. but if he had sold at twice the cost price, what will be his gain?(question not sure) a. 2x b.200-2x c.100+x (not sure of the options) 19.) a clock was 7mts behind the actual time on 3 p.m. on wednesday and 8 mts ahead of actual time on (not sure) 4 p.m. friday. when will it show the correct time? 20.) boat moves upstream in 6 hrs and covers the same distance downstream in 5 hrs. then speed of a raft floating?(accuracy of question not sure) 21.) (this was the last question) no idea what it stands for........ some kind of notation like S(P(M((D(a,b),2))):P(M(S(D(a,b),........ options were 1. ab 2.(a-b)^2 3.(a+b)^2 4.none 22.) if x men working x hrs per day can do x units of work in x days, then y men working y hrs/day would be able to complete how many units of work i y days? ans. y^3/x^2 (question in R.S. Agarwal) 23.) ( this was a question in the first page of the section II booklet) a cone with radius----- and height -----. a hemisphere covers the cone such that base of hemisphere meets that of the cone. then the enclose volume.....(R.S. Agarwal consists of similar questions) 24.) there was one more question on volume and surface area..... 25.) 1 Rs, 50 ps , 25 ps coins are in the ratio ---------, then the number of 50 ps coins if they sum to ------Rs. (similar question in R.S.Agarwal) 26.) there was one more question on coins i.e. abt getting a change of 10ps and 25 coins for ------Rs.(how many possible combinations or so possible) 27.) x/y+y/x=40/21(don't remember the exact value, believe this is the one) find x and y there were 2 questions on train and one was like: 28.) a goods train starts and after 2 hrs a passenger train at 4km/hr starts and overtakes the goods train after 4 hrs, then the speed of goods train? 29.) 15hrs of boys work=6 hrs of women's work. 3/5 of the work is done by -----boys and -----women. How much time would be taken by the women to complete the work? 30.) using false weight a man gains -----%, for a k.g. then he uses a weight of.....(this kind of question has some formula that can be found in R.S.Agarwal)

31.) there was one question on L.C.M. and co primes(don't remember the question) 32.) there was one question on triangle 33.) a figure was given a square with four corners shaded and asked to find the area of the shaded portion.... ie area of square-area of the regular octagon..... This is all i can remember from section II SECTION I ENGLISH (EASY) PATTERN: 1.) few questions on finding out the segment of sentence which is wrong.... 2.) analogy. (few i can remember) 1.) celebrate::marriage ans. lament::bereavement 2.) bouquet::flowers ans. chain::link 3.) germ::disease ans. war::destruction 4.) (remember only the answer) ans. book::pages

3.) to fill up the blanks in the sentences with the most appropriate words..... 4.) to find the word opposite in meaning (few qns i can remember) 1.) equanamity 2.) sequester 3.) dormant 4.) apathetic

SECTION III REASONING (TIME IS A CONSTRAINT) 1.) (last question and the easiest) a simple flowchart whose steps are as follows. value of r and h are given, a and b=0 m=pi*r*r*h n=(pi*r*r*h)/3 if (m==n) do ------else b=4*pi*r*h print a and b [DON'T REMEMBER IF conditon is m==n or m<n]

2.) (second last question this was also easy) it was abt a set of people talking 4 languages and then qns were abt who could act as a translator to whom ........ 3.) there were 3 questions based on an argument given... questions were to find out which statement in the objectives given could strengthen the argument or to weaken the argument....(bit confusing) 4.) blood relationship question( i remember all the statements given, though not the question) 10 people in a family A B C are husbands D E F are wives altogether there are only 4 women. each family can have atmost 2 children and C has atleast 1 child. D and G are related to A. E cannot be related to H and I and J are not related..... 5.) (this consists of the most confusing set of questions) a set of statements are given and then questions were asked, the statements were like All A B C D E F are Q's All A's are B's -------------(better leave this question for the last) 6.) Alice works on Monday, Wednesday and Friday B does not work on Wed C works only on Tuesday and someotherday(don't remember) D does not work on Friday E works on all days except on the first Monday and Thursday then questions like who all will be available on which days ..... 7.) question based on venn diagram given ......read the question properly and then answer 8.) (first question) abt 4 buses with different seating capacity.and different charges. and questions were to find out the minimum number of trips required, the maximum charge, the second highest charge etc.... 99999999999999999999999999999999999999 one more thing: though they said abt 50% cut-off for each section, for us THEY HAD CONSIDERED THE TOTAL(OUT OF 100) only AND NOT THE CUT-OFF...... seems they have changed their processing technique......don't think they are are going to change the questions atleast for quite a while.....so this being the latest set, those preparing for syntel, i would suggest u to peruse through the questions and be ready to answer them.... remember rate of answering counts in a lot for this test...... this is all i can recollect........HOPE THIS PROVES TO BE HELPFUL FOR ALL THOSE ASPIRING FOR SYNTEL................................... AND BEST OF LUCK FOR THE SAME!!!!!!!!!!! CENSURE a. purify b. approve

c. edit d. uncertain ONUS a. b. c. d. honest inclination responsibility accuse

DIVERGENT a. b. c. deviating d. FIDELITY a. restlessness b. disloyalty c. feeble d. vagueness SURVEILLANCE a. inattention b. visibility c. census d. prevention WHIMSICAL a. victorious b. swift c. fanciful d. momentary NASCENT a. threat b. purpose c. quality d. emerging BENIGN a. kindly b. malignant c. envy d. tenfold ANALOGOUS a. capable b. culpable c. comparable d. corporeal ATTENUATE a. appear b. weaken c. testify d. soothe EFFUSIVE a. wise b. reserved c. peaceful d. spontaneous GREGARIOUS a. logical b. helpful c. solitary

d. noisy EMPIRICAL a. theoretical b. mathematical c. verbal d. royal CITE a. galvanize b. quote c. locate d. visualize TENACIOUS a. intentional b. obnoxious c. holding fast d. collecting TRANSIENT a. ephemeral b. permanent c. clear d. emptiness VOLUBLE a. worthwhile b. loquacious c. circular d. serious CANDID a. vague b. outspoken c. experienced d. anxious VERACITY a. b. c. truthfullness d. STANDING a. reputation b. activity c. long time d. duration SACROSANCT a. too important b. worship c. sacrifice d. best CLUTCH a. b. c. d. AUGMENT a. decrease b. belittle c. simplify d. magnify

GENERIC a. b. c. d. DIVULGE a. b. c. d. EFFIGY a. dummy b. organ c. charge d. accordion PRECARIOUS a. hazardous b. priceless c. premature d. primitive SONOROUS a. reassuring b. resonant c. repetitive d. sisterly CIRCUITOUS a. indirect b. complete c. obvious d. aware PEDIGREE a. dogs b. vast c. courage d. line of ancestry ANTONYMS: EXPEDIENT a. illiterate b. delayed c. mistake d. impediment IRRADIATE a. agreement b. distance c. flight d. clarity ANOMALY a. desperation b. requisition c. registry d. regularity BENIGN a. peaceful b. blessed look refuse deride reveal external particular personal subdued

c. wavering d. malignant ANALOGUE a. b. c. d. ANALOGOUS a. not comparable b. not capable c. not culpable d. not congenial CENSURE a. b. c. d. DIVULGE a. converge b. intake c. involve d. conceal SURVEILLANCE a. inattention b. visibility c. census d. prevention HAMPER a. b. c. d. DANGLE a. hanging b. loose c. secure d. mingle SPENDTHRIFT a. miser b. savings c. cautious d. extravagant INDIGENOUS a. b. c. d. CRYPTIC a. futile b. famous c. candid d. indifferent OPTIMUM a. pessimistic b. minimum c. chosen process enclose praise penetrate same digital lengthy dull

d. worst RETROGRADE a. progressing b. inclining c. evaluating d. concentrating TRANSIENT a. carried b. close c. permanent d. certain VERITY a. falsehood b. sanctity c. rarity d. household CENSURE a. augment b. eradicate c. enthrall d. commend EMPIRICAL a. theoretical b. mathematical c. verbal d. royal AUGMENT a. keep away b. be disturbed c. to increase d. dig out BOLSTER a. defeat b. to strengthen c. be angry d. depth COMPLIANCE a. light b. fresh c. take away d. energize DEBILITATE a. balmy b. bedevil c. animate d. deaden DEROGATORY a. roguish b. immediate c. conferred d. praising ERRONEOUS a. accurate b. dignified c. curious d. abrupt

EXONERATE a. b. c. d. GREGARIOUS a. b. c. d. OBJECTIVE a. b. c. d. forge accuse record reimburse anticipating glorious antisocial similar indecisive apathetic emotionally involved authoritative

------------------------------------------------------------------------------SECTION I 1. If VXUPLVH is written as SURMISE, what is SHDVD ? Ans. PEASA (hint: in the first word, the alphabets of the jumbled one is three alphabets after the corresponding alphabet in the word SURMISE. S = V-3, similarly find the one for SHDVD) 2. If DDMUQZM is coded as CENTRAL then RBDJK can be coded as --------Ans. QCEIL (hint: Write both the jumbled and the coded word as a table, find the relation between the corresponding words, i.e C= D-1, N=M+1 & so on 3. In the word ECONOMETRICS, if the first and second , third and forth ,forth and fifth, fifth and sixth words are interchanged up to the last letter, what would be the tenth letter from right? Ans. word is CENOMOTEIRSC tenth word is R 4. Find the result of the following expression if, M denotes modulus operation, R denotes round-off, T denotes truncation: M(373,5)+R(3.4)+T(7.7)+R(5.8) Ans. 19 5. What is the largest prime number that can be stored in an 8-bit memory? Ans. 6. Find the physical quantity in units from the equation: (Force*Distance)/(Velocity*Velocity) Ans. Ns2/m 7. Find the value of @@+25-++@16, where @ denotes "square" and + denotes "square root". Ans: 621 8. If f(0)=1 and f(n)= f(n-1)*n, find the value of f(4). Ans: 24 9. Convert the decimal number 310 to the base 6. Ans: 1234 10. Find the missing number in the series: 2, 5, __ , 19 , 37, 75 Ans: 9 11. In a two-dimensional array, X(9,7), with each element occupying 4 bytes of memory, with the address of the first element X(1,1) is 3000,

find the address of X(8,5). Ans. 12. Find the fourth row, having the bit pattern as an integer in an 8-bit computer, and express the answer in its decimal value. A 0 0 0 0 1 1 1 1 B 0 0 1 1 0 0 1 1 C 0 1 0 1 0 1 0 1 (AU(B-C)) ? Ans. 29 13. Complete the series 2, 7, 24, 77,__ (hint: 2*12= 24, 7*11= 77, therefore 24*10= 240) Ans: 240 14. Consider the following diagram for answering the following questions: A. Find the difference between people playing cricket and tennis alone. Ans: 4 B. Find the percentage of people playing hockey to that playing both hockey and cricket. Ans: C. Find the percentage of people playing all the games to the total number of players. Ans: 6% 15. One more question of the same type (Same type of diagram; of course in a different set) 1. How many more or less speak English than French? 2. What % people speak all the three languages? 3. What % people speak German but not English? {In another set cricket, hockey and tennis are changed with the name of some computer languages, such as Java, Cobol, Fortran (may be some other name)} 16. Select the odd one out a. Oracle b. Linux c. Ingress d. DB2 17. Select the odd one out a. SMTP b. WAP c. SAP d. ARP 18. Select the odd man out. a. Java b. Lisp c. Smalltalk d. Eiffel 19. Which of the following are orthogonal pairs? a. 3i+2j b. i+j c. 2i-3j d. -7i+j 20. Number of faces, vertices and edges of a cube a. 12,8,6 b. 4,6,8 c. 6,8,12 d. 6,12,8 21. Given a Bar Chart showing the sales of a company. (In Figure) The sales in years as shown in the figure are (in crores) 1998-1999 - 130, 1997-1998 - 90, 1996-1997 - 90, 1995-1996 - 70 1. The highest growth rate was for the year Ans. 1998-1999 2. The net increase in sales of the company in the year span of 1995-1999

Ans. 60 crores. 3. The lowest growth rate was for the year Ans. 1997 22. Find the value of the decimal number to the base 7. Ans. 1436. 23. Complete the series:5,6,7,8,10,11,14,__. Ans. 15 24. If the vertex (5,7) is placed in the memory. First vertex (1,1) s address is 1245 and then address of (5,7) is ---------Ans. 25. In which of the system, decimal number 384 is equal to 1234? Ans. 26. A man, a woman, and a child can do a piece of work in 6 days. Man only can do it in 24 days. Woman can do it in 16 days and in how many days child can do the same work? Ans. 27. In Madras, temperature at noon varies according to -t^2/2 + 8t + 3, where t is elapsed time. Find how much temperature more or less in 4pm to 9pm. Ans. 28. The size of the bucket is N kb. The bucket fills at the rate of 0.1 kb per millisecond. A programmer sends a program to receiver. There it waits for 10 milliseconds. And response will be back to programmer in 20 milliseconds. How much time the program takes to get a response back to the programmer, after it is sent? Ans. 29. The size of a program is N. And the memory occupied by the program is given by M = square root of 100N. If the size of the program is increased by 1% then how much memory now occupied ? Ans. 30. A power unit is there by the bank of the river of 750 meters width. A cable is made from power unit to power a plant opposite to that of the river and 1500mts away from the power unit. The cost of the cable below water is Rs. 15/- per meter and cost of cable on the bank is Rs.12/per meter. Find the total of laying the cable. Ans. Rs. 22,500 (hint: the plant is on the other side of the plant i.e. it is not on the same side as the river) {There are two questions, both showing a curve. In the first one, you have to identify the curve. In the second one you have to Write the equation of the curve. In } SECTION II 1. If A can copy 50 pages in 10 hours and A and B together can copy 70 pages in 10 hours, how much time does B takes to copy 26 pages? a. b. c. d. 2. Match the following: 1. Male - Boy ---> a. A type of 2. Square - Polygon ---> b. A part of 3. Roof - Building ---> c. Not a type of 4. Mushroom - Vegetables ---> d. A superset of Ans: 1- d, 2- a, 3- b, 4- c 3. Match the following. 1. brother sister ---> a. Part of

2. Alsatian dog 3. sentence paragraph 4. car - steering of

---> --->

b. Sibling c. Type of ---> d. Not a type

Ans. 1-b, 2-c, 3-a, 4-d Questions 20- 24 are based on the following passage: The office staff of the XYZ corporation presently consists of three bookkeepers (A, B and C) and five secretaries (D, E, F, G and H). Management is planning to open a new office in another city using three secretaries and two bookkeepers of the current staff. To do so they plan to separate certain individuals who do not function well together. The following guidelines were established to set up the new office: I. Bookkeepers A and C are constantly finding fault with one another and should not be sent as a team to the new office. II. C and E function well alone but not as a team. They should be separated. III. D and G have not been on speaking terms for many months. They should not go together. IV. Since D and F have been competing for promotion, they should not be a team. Ans. 4. If A is to be moved as one of the bookkeepers, which of the following cannot be a possible working team? (a) ABDEH (b) ABDGH (c) ABEFH (d) ABEGH (e) ABFGH 5. If C and F are moved to the new office, how many combinations are possible? (a) 1 (b) 2 (c) 3 (d) 4 (e) 5 6. If C is sent to the new office, which member of the staff cannot go with C? (a) B (b) D (c) F (d) G (e) H 7. Under the guidelines developed, which of the following must go to the new office? (a) B (b) D (c) E (d) G (e) H 8. If D goes to the new office which of the following is (are) true? I. C cannot go. II. A cannot go. III. H must also go. a. I only. b. II only. c. I and II only. d. I and III only. e. I, II and III. 9. Two stations A & B are 110 km apart. One train starts from A at 7 am, and travels towards B at 20kmph. Another train starts from B at 8 am and travels towards A at 25kmph. At what time will they meet? a. 9 am b. 10 am c. 11 am d. 10.30 am 10. If a man can swim downstream at 6kmph and upstream at 2kmph, his speed in still water is: a. 4kmph b. 2kmph c. 3kmph d. 2.5kmph Answer the following three questions based on the paragraph:

A student applying at a college should take three courses. There are altogether four courses, namely, Science, Maths, Social Sudies and Economics. (The names may vary, but the pattern is the same) One can take a Science course onty if he has taken a Maths course. One can take a Maths course only if he has taken a Science course. One can take a n Economics course only if he has taken a Social Studies course. 11. Which of the following is a possible course? a. Two Science courses and a Social Studies Course. b. Two Maths courses and an Economics course. c. One Mats course, one Science course and a Social Studies course. d. One Maths course, one Science course and an Economics course. 12. Which of the following courses a student can take? I. One Science, one Social Studies, one Economics II. Two Science, one Social Studies III. Two Science, one Maths. a. I only b. III only c. I and II only d. II and III only 13. Which of the following is not a possible course? a. Two Science courses and a Maths course. b. Two Maths course and a Science course. c. One Maths course, one Science course and a Social Studies Course. d. One Maths course, one Science course and an Economics course. SECTION III 1. There are 9 people I, J, K, L, M, N, O, P and Q living in a five storied building. The top floor has only one room and all other floors have two rooms. No rooms are vacant. No rooms carry two people. { There are some conditions like the following } K and N live on the same floor. L is living in a floor lower than I. Q is living in an upper floor than N. O lives in the third floor. { Four questions are asked based on this paragraph regarding the floor a specific person lives on; some conditions given and to state which one is valid; some condition, based on which to state in which floor a specific person lives etc. } --------------------------------paper 2--------------------------------] Do Quanta was repeated but wordlist was new. Gre pattern Verbal 10 synonym 10 antonym

5 sentence completion 0ne reading comp synonym dwindle=less efface=destroy indiginity inept=unsuitable infirmity=weak from age or illness. candid=frank dangle=tempting Antonym restivness Irksome Jaunty Nebulous=shape. Misapprehension Obese Obloguy(abuse)= . Quanti Very easy similar to Jampshed nit Data have been changed Critical 3 puzzles.12 question (4each) go throudh this section very seriously.there is lower cutoff in this section many student didnt do well in this section.go through barrons gre (its very important)

QUESTIONS OF TCS- 5-7 JUNE 2003 IN ANNA UNIVERSITY SECTION 1: VERBAL WE WERE NOT ABLE TO GET THE COMPREHENSION AND SENTENCE COMPLETION QUESTIONS AS EVERYONE WROTE THE TEST IN THE MIDNIGHT BETWEEN 11 PM 4 AM. MOST OF THEM WERE NOT ABLE TO UNDERSTAND THE PASSAGE. I HAVE GIVEN BELOW THE WORDS THAT WERE ASKED IN SYNONYMS AND ANTONYMS. WHIMSICAL=fanciful. CENSURE. =criticism OPTIMUM. MISAPPREHENSION. CANDID.=frank TURSE.= CITE. EFFUSIVE= overenthusiastic IRRADIATE. =Radiate(darken) TENACIOUS.= firm., stubborn VOLUBLE.= talkative. BANAL.= commonplace,. Trite. RUPTURE.=damage STANDING.=present, NASCENT.= emerging., budding.

TRANSIENT.=temporary CLUTCH.=hold GENERIC.= general.,( specific.) EMPIRICAL.=experimental ANOMALY.= abnormality., glitch. CIRCUITOUS.=roundabout,not following the direct path. HAMPER.=stop SURVEILLANCE.= observation., watch. OBJECTIVE.= purpose RAUCOUS.= harsh., rasping. VORACIOUS.= greedy. hungry. PEDIGREE.= ancestry.. derivation. FIDELITY.= loyalty, commitment AUGMENT.=support PRECARIOUS.= unstable., uncertain. (stable.) ALACRITY.=cheerful eagerness. DEROGATORY.=absurd remarks ONUS. ANALOGUE.= EXPEDIENT.=suitable ANALOGOUS.=similar ASSUAGE.=to soothe,pacify. COMPLIANCE.=obedient DIFFIDANT. =timid,lacking in self confidence ant. bold PLAINTIVE.= lamenting. Sorrow &sadness. INSINUATE. = hint. intimate. MISDEMEANOR.=misdeed EXONERATE.=to free from blame. GREGARIOUS.=fond of company ANATHEMATIZE. = curse(bless) BENIGN.=calm ATTENUATE.=weaken SONOROUS.=resonant BOLSTER.=support DivERGENT.= DECOLLATE=low necked HETERODOX ant:-orthodox RESTIVENESS=restless IGNONIMOUS.= dishonor , deep disgrace PLAGIARIZE= steal another ideas and pass them as ones own EFFIGY=image TENACIOUS.=persistent RETROGADE.=degrade SACROSANCT.=sacred DANGLE.=hopes temptingly ANOMALY.=abnormal CRYPTIC.=mysterious DEBILITATE.=weaken DIVULGE.=to show,reveal SCEPTIC.=unwilling to believe things SPENDTHRIFT.=prodigal INDIGENOUS.=native ERRONIUS.= RUPTURE.=damage

MINION.=assistant VERACITY.=truthfulness MITIGATE=less severe. APEX 3.BREKISH COGENT=convincing LETHARGY=dull,stupor 6.HAMPER 7.BANAL=common place CANDID=frank TACITURN PEDIGREE hapless Inundate Fledged Revile Revere Ignominious effte vouchsafe SECTION 2: QUANTITATIVE AND LOGICAL REASONING. MOST OF THE QUESTIONS WERE FROM TCS OLD PAPERS. THE QUESTIONS WILL BE OF THIS PATTERN TO EVERYONE. ONLY THE DATA GET CHANGES. In a two-dimensional array, X (9, 7), with each element occupying 4 bytes of memory, with the address of the first element X (1, 1) is 3000, find the address of X (8, 5). Ans 3212 In the word ORGANISATIONAL, if the first and second, third and forth, forth and fifth, fifth and sixth words are interchanged up to the last letter, what would be the tenth letter from right? Ans I What is the largest prime number that can be stored in an 8-bit memory? 251 Select the odd one out. a. Java b. Lisp c. Smalltalk d. Eiffel. Select the odd one out a. SMTP b. WAP c. SAP d. ARP Select the odd one out a. Oracle b. Linux c. Ingress d. DB2 Select the odd one out a. WAP b. HTTP c. BAAN d. ARP Select the odd one out a. LINUX b. UNIX c. SOLARIS d. SQL SERVER Select the odd one out a. SQL b. DB2 c. SYBASE d. HTTP The size of a program is N. And the memory occupied by the program is given by M = square root of 100N. If the size of the program is increased by 1% then how much memory now occupied? Ans 0. 5%( SQRT 101N) A man, a woman, and a child can do a piece of work in 6 days. Man only can do it in 24 days. Woman can do it in 16 days and in how many days child can do the same work? Ans 16 In which of the system, decimal number 184 is equal to 1234? Ans 5 Find the value of the 678 to the base 7. ans 1656 Number of faces, vertices and edges of a cube ans 6 8 12 Complete the series 2, 7, 24, 77,__ ans 238 Find the value of @@+25-++@16, where @ denotes "square" and + denotes "square root". Ans- 621 Find the result of the following ex-pression if, M denotes modulus operation, R denotes round-off, T denotes truncation:

M(373,5)+R(3.4)+T(7.7)+R(5.8) ans 19 If TAFJHH is coded as RBEKGI then RBDJK can be coded as --------- ans qcckj G(0)= -1, G(1)=1, G(N)=G(N-1) - G(N-2), G(5)= ? ans - 2 What is the max possible 3 digit prime number? Ans 997 A power unit is there by the bank of the river of 750 meters width. A cable is made from power unit to power a plant opposite to that of the river and 1500mts away from the power unit. The cost of the cable below water is Rs. 15/- per meter and cost of cable on the bank is Rs.12/per meter. Find the total of laying the cable. Ans 1000 (24725 cost) The size of a program is N. And the memory occupied by the program is given by M = square root of 100N. If the size of the program is increased by 1% then how much memory now occupied? Ans 0. 5%( SQRT 101N) In Madras, temperature at noon varies according to -t^2/2 + 8t + 3, where t is elapsed time. Find how much temperature more or less in 4pm to 9pm. Ans. At 9pm 7.5 more The size of the bucket is N kb. The bucket fills at the rate of 0.1 kb per millisecond. A programmer sends a program to receiver. There it waits for 10 milliseconds. And response will be back to programmer in 20 milliseconds. How much time the program takes to get a response back to the programmer, after it is sent?ans 30 A man, a woman, and a child can do a piece of work in 6 days. Man only can do it in 24 days. Woman can do it in 16 days and in how many days child can do the same work? Ans 16 If the vertex (5,7) is placed in the memory. First vertex (1,1) s address is 1245 and then address of (5,7) is ----------1279 Which of the following are orthogonal pairs? a. 3i+2j b. i+j c. 2i-3j d. -7i+j ans a,c If VXUPLVH is written as SURMISE, what is SHDVD? ans PEASE If A, B and C are the mechanisms used separately to reduce the wastage of fuel by 30%, 20% and 10%. What will be the fuel economy if they were used combined. Ans 20% What is the power of 2? a. 2068 b.2048 c.2668 Complete the series. 3, 8, --, 24, --, 48, 63 . ans 15, 35 Complete the series. 4, -5, 11, -14, 22, --- Ans - 27 A, B and C are 8 bit nos. They are as follows: A 1 1 0 1 1 0 1 1 B 0 1 1 1 1 0 1 0 C 0 1 1 0 1 1 0 1 Find ( (A-B) u C )=? Hint : 109 A-B is {A} {A n B} A Flight takes off at 2 A.M from northeast direction and travels for 11 hours to reach the destination which is in north west direction. Given the latitude and longitude of source and destination. Find the local time of destination when the flight reaches there? 7 AM A can copy 50 papers in 10 hours while both A & B can copy 70 papers in 10 hours. Then for how many hours required for B to copy 26 papers? 13 A is twice efficient than B. A and B can both work together to complete a work in 7 days. Then find in how many days A alone can complete the work? 10.5

A finish the work in 10 days. B is 60% efficient than A. So how days does B take to finish the work? Ans 100/6 curve was given & in option equation was given , you have to mark the correct equation(curve and equation was of general logx,ex ,sinx,cosx,tanx) value of log3.142& log3.143 was given and log3,141 was asked so you have to only see on answer and mark values of x &f(x) was given (like x=1000&f(x)=3) and equations was given Sum of slopes of 2 perpendicular st. lines is given. Find the pair of lines from the given set of options which satisfy the above condition? 43. (a) 2+3i (b)1+i (c) 3-2i (d) 1-7i .Find which of the above is orthogonal. Ans a,c 44. (Momentum*Velocity)/(Acceleration * distance ) find units. ans mass 45. The number 362 in decimal system is given by (1362)x in the X system of numbers find the value of X a}5 b) 6 c) 7 d) 8 e) 9 46. Given $ means Tripling and % means change of sign then find the value of $%$6-%$%6 47. My flight takes of at 2am from a place at 18N 10E and landed 10 Hrs later at a place with coordinates 36N70W. What is the local time when my plane landed. a) 6:00 am b) 6:40am c)7:40 d)7:00 e)8:00 (Hint : Every 1 deg longitude is equal to 4 minutes . If west to east add time else subtract time) 48. Find the highest prime number that can be stored in an 8bit computer. 49. Which of the following set of numbers has the highest Standard deviation? 1,0,1,0,1,0 -1,-1,-1,-1,-1,-1 1,1,1,1,1,1 1,1,0,-1,0,-1 50. Match the following: 1. Male - Boy ---> a. A type of 2. Square - Polygon ---> b. A part of 3. Roof - Building ---> c. Not a type of 4. Mushroom - Vegetables ---> d. A superset of &nb! sp; Ans: 1- d, 2- a, 3- b, 4- c 51. Match the following. 1. brother sister ---> a. Part of 2. Alsatian dog ---> b. Sibling 3. sentence paragraph ---> c. Type of 4. car - steering ---> d. Not a type of &! nbsp; Ans. 1-b, 2-c, 3-a, 4-d *****Remaining Questions were from Graph, pie charts, Bar chart and Venn diagram which were very easy.

SECTION 3: CRITICAL REASONING I.The players G,H,J,K,L,M,N,O are to be felicitated of representing the county team in Baseball Out of these H,M,O also are in the Football

team and K,N are there in the Basket ball team . These players are to be seated on a table and no two players who have represented the county in more than one game are to sit together. 1.Which one of the orders can they be seated in 2. Which of the orders is not possible 3. If N is seated in the middle then which of the following pairs cannot be seated near him . 4. If M is seated then which of the following pairs can be seated next to him. Choices are given for all the questions

II There are 2 groups named Brown and red. They cant marry in the same group. If the husband or wife dies then the person will convert to their own group. If a person is married then the husband will have to change his group to his wifes group. The child will own the mothers group. From these a set of 4 questions were given .Solve them Eg; 1.Browns daughter is red (False) 2. If a person is red. Then his/her mothers brother belong to which group if he is married (Brown)

III 7 people a,b,c,d,e,f,g Need to make a seating arrangement for them. Conditions: 1)A should be at the center 2) B,F should be at the right extreme 3)C,E always in pair 4)D,G as far as possible Questions from the above were asked? Eg: Which of the following pairs were not possible? Others questions were similar to the above. More questions were from Barrons. HR QUESTIONS: For all the questions they expect a elaborate answer with justifications and not a short one. 1) Market urself 2) Why TCS ? 3) Will u switch over to any other company after joining TCS? If NO then why? 4) R u mobile? ( R u ready to go anywhere ?) 5) R u ready to go to places of extreme temperature. If yes what do u think of ur safety? 6) What are the requirements for leadership quality ? 7) Why u switch over to s/w from ur own back ground? 8) What are the qualities required for a s/w engineer and Project manager? 9) Rate ur good qualities? 10) What is the difference between hard and smart work? 11) Do u have a plan of doing higher studies ?

Technical Questions : General : The questions were different for different persons and depends on the field of interest that u say. Communication : 1) OSI Layers in n/w with the functionalities 2) TCP/IP protocol 3) Bridges, Routers, LAN, WAN, MAN 4) Token bus, FDDI, Ethernet C Language : Datastructures esp :Linked list and trees Structures , unions, Kind of variables ( local ,Global) Strrev( ) program Case structure (it is not the usual switch case stat) Calloc,malloc Divide 2 number and print the o/p ( NOTE: Check for divide by zero error) PSYCHOMETRY TEST Direction: In this section you will find different questions with the same meaning. In all such questions your answer has to be same. for e.g.: In being thrown by chance with a stranger, you wait for the person to introduce himself or herself. (a) Yes (b) No (c) ? It is difficult for you to chat about things in general with people. (a) Yes &! nbsp; (b) No (c) ? These two questions have similar meanings. If you answer the first one 'NO' and the second one 'YES', i.e. if you differ in your answers to similar questions you lose marks for every question with the above meaning. The choices to these questions are: (a) Yes. (b) No. (c) ? 1. You start to work on a project with great deal of enthusiasm. 2. You would rather plan an activity than take part in it. 3. You have more than once taken lead in organizing project or a group of some kind. 4. You like to entertain guests. 5. Your interests change quickly from one thing to another. 6. When you eat a meal with others, you are usually one of the last to finish. 7. You believe in the idea that we should " eat, drink and be merry, for tomorrow we die." 8. When you find that something you have bought is defective, you hesitate to demand an exchange or a refund. 9. You find it easy to find new acquaintances. 10. You are sometimes bubbling over with energy and sometimes very

sluggish. 11. You are happiest when you get involved in some projects that calls for rapid action. 12. Other people think of you as being very serious minded. 13. In being thrown by chance with a stranger, you wait for the person to introduce himself or herself. 14. You like to take part in many social activities. 15. You sometimes feel "just miserable" for no good reason at all. 16. You are often so much " on the go" that sooner or later you may wear yourself out. 17. You like parties you attend to be lively. 18. If you hold an opinion that is radically different that expressed by a lecturer, you are likely to tell the person about it either during or after the lecture. 19. It is difficult for you to chat about things in general with people! . 20. You give little thought to your failures after they are passed. 21. You often wonder where others get all the excess energy they seem to have. 22. You are inclined to stop to think things over before you act. 23. You avoid arguing over a price with a clerk or sales person. 24. You would dislike very much to work alone in some alone place. 25. You often find it difficult to go to sleep at night because you keep thinking of what happened during the day. ???? 26. You find yourself hurrying to get to places even when there is plenty of time. 27. You like work that requires considerable attention to details. 28. You are satisfied to let some one else take the lead in group activities. 29. You enjoy getting acquainted with people. 30. It takes a lot to get you emotionally stirred up or excited. 31. You work more slowly and deliberately than most people of your sex and age. 32. You are a carefree i! ndividual. 33. When people do not play fair you hesitate to say anything about it to them. 34. It bothers you to have people watch you at your work. 35. You have usually been optimistic about your future. 36. You like to have plenty of time to stop and rest. 37. You take life very seriously. 38. You enjoy applying for a job in person. 39. You would like to be a host or hostess for parties at club. 40. You often feel uncomfortable or uneasy. 41. You are the kind of person who is "on the go" all the time. 42. You often crave excitement. 43. The thought of making a speech frightens you. 44. You find it easy to start conversation with strangers. 45. You often feel guilty without a very good reason for it. 46. People think you are a very energetic person. 47. You sometimes make quick decisions that you later wish you hadn't made.?????? 48. You find it difficult to ask people for money or other donations, even for a cause in which you ! are interested. 49. You are so naturally friendly that people immediately feel at ease

with you. 50. You daydream a great deal. 51. You are quick in your actions. 52. You have a habit of starting things and then losing interest in them. 53. When you were a child many of your playmates naturally expected you to be the leader. 54. You sometimes avoid social contacts for fear of doing or saying the wrong thing. 55. You have frequent ups and downs in mood, sometimes with and sometimes without apparent cause. 56. You always seem to have plenty of vigor and vitality. 57. It is difficult for you to understand people who get very concerned about things. 58. When a clerk in a store waits on others who come after you, you call his or her attention to the fact.?????? 59. You would be very unhappy if you were prevented from making numerous social contacts. 60. There are times when your future looks very dark. 6! 1. You sometimes wish that people would slow down a bit and give you a chance to catch up. 62. Many of your friends think you take your work too seriously. 63. You hesitate to walk into a meeting when you know that everyone's eye will be upon you. 64. You limit your friendships mostly to members of your own sex. 65. You almost always feel well and strong. 66. You seem to lack the drive necessary to get as much as other people do. 67. You make decisions on the spur of the moment. 68. You are rather good at bluffing when you find yourself in difficulty. 69. After being introduced to someone , you just cannot think of things to say to make good conversation. 70. You feel lonesome even when with other people. 71. You are able to work for unusually long hours without feeling tired. 72. You often act on the first thought that comes into your head. 73. At the scene of an accident, you take an active part in helping out. 74. You have difficulty i! n making new friends. 75. Your mood often changes from happiness to sadness or vice versa without knowing why. 76. You talk more slowly than most people. 77. You like to play practical jokes upon others. 78. You take the lead in putting life into a dull party. 79. You would like to belong to as many clubs and social organizations as possible. 80. There are times when your mind seems to work very slowly and other times when it works very rapidly. 81. You like to do things slowly and deliberately.?? 82. You are a happy-go-lucky individual. no 83. When you are served stale or inferior food in a restaurant, you say nothing about it. 84. You would rather apply for a job by writing a letter than by going through with a personal interview. 85. You are often in low spirits.

86. You are inclined to rush from one activity to another without pausing enough for rest. 87. You are so concerned about the future that you do not get as much fun out of the! present as you might. 88. When you are attracted to a person whom you have not met earlier you make an active attempt to get acquainted even though it may be quite difficult. 89. You are inclined to limit your acquaintances to select few 90. you seldom give your past mistakes a second thought. 91. You are less energetic than many people you know. 92. You often stop to analyze your thoughts and feelings. 93. You speak out in meetings to oppose those whom you feel sure are wrong. 94. You are so shy it bothers you. 95. You are sometimes bothered by having a useless thought come into your mind over and over. 96. You get things in hurry. 97. It is difficult for you to understand how some people can be so unconcerned about the future. 98. You lie to sell things (i.e. to act as a sales person) ?? 99. You are often "Life of the Party".??? 100. You find daydreaming very enjoyable. 101. At work or at play other! people find it hard to keep up with the pace you set. 102. You can listen to a lecture without feeling restless. 103. You would rather work for a good boss than for yourself. 104. You can express yourself more easily in speech than in writing. 105. You keep in fairly uniform spirits. 106. You dislike to be hurried in your work. 107. You sometimes find yourself "crossing bridges before you come to them". 108. You find it somewhat difficult to say "no" to a sales person who tries to sell you something you do not really want. 109. There are only a few friends with whom you can relax and have a good time. 110. You usually keep cheerful in spite of trouble. 111. People sometimes tell you to "slow down" or "take it easy". 112. You are one of those who drink or smoke more than they know they should. 113. When you think you recognize people you see in a public place, you ask them whether you have ! met them before. 114. You prefer to work alone. 115. Disappointment affect you so little that you seldom think about them twice. 116. You are slow and deliberate in movements. 117. You like wild enthusiasm, sometimes to a point bordering on rowdyism at a football or baseball game. 118. You feel self conscious in the presence of important people. 119. People think of you as being a very social type of person. 120. You have often lost sleep over your worries. 121. You can turn out a large amount of work in a short time. 122. You keep at a task until it is done, even after nearly everyone else has given up. 123. You can think of a good excuse when you need one.

124. Other people say that it is difficult to get to know you well. 125. You daydreams are often about things that can never come true. 126. You often run upstairs taking two steps at a time. 127. You seldom let your responsibilit! y interfere with your having a good time. 128. You like to take on important responsibilities such as organizing a new business. 129. You have hesitated to make or to accept "dates" because of shyness. 130. Your mood is very easily influenced by people around you. 131. Others are often amazed by the amount of work you turn out. 132. You generally feel as though you haven't a care in the world. 133. You find it difficult to get rid of sales person whom you do not care to listen or give your time. 134. You are a listener rather than a talker in a social conversation. 135. You almost always feel that life is very much worth living. 136. It irritates you to have to wait at a crossing for a long freight train to pass. 137. You usually say what you feel like saying at the moment. 138. You like to speak in public. 139. You like to be with people. 140. You generally keep cool and think clearly in exciting situations. 141. Other people regard ! you as a lively individual. 142. When you get angry, if you let yourself go, you feel better. 143. You seek to avoid all trouble with other people. 144. People seem to enjoy being with you. 145. You sometimes feel listless and tired for no good reason. 146. It is hard to understand why many people are so slow and get so little done. 147. You are fond of betting on horse races and games, whether you can afford it or not. 148. If someone you know has been spreading untrue and bad stories about you, you see the person as soon as possible and have a talk about it. 149. Shyness keep you from being as popular as you should be. 150. You are generally free from worry about possible misfortunes. PSYCHOMETRY GROUPING I have grouped the psychometric questions numbering 150. as it is not possible for me to write each question I am indicating only the numbers. Its up to u to correspond to the questions At the outset I would like to clearly state that this version has no guarantee to be the correct version. Actually some 30/150 questions fall in to many groups and the line differentiating them is very thin. I have tried my best to group them. Talking about the groups- they are 11 in number. As I have mentionedthis is not the only correct version- but this can be one. So if u feel u can group them more finer u can subdivide them in to many more. I would advise u strictly to go through the whole set of groups and define urselves- the key to psychometric test is to define who u are and then start looking in to deeply the questions. Each group represents a character or personality- u will decide which

of the following attributes u would like to include in ur personality. GROUP 1 LEADERSHIP QUALITIES 2 3 28 39 53 73 103 128 GROUP 2 SOCIABLE NATURE 9 13 14 24 29 44 49 54 59 64 70 74 79 88 89 109 114 119 139 GROUP 3 BALANCED PSYCHE people who are balanced- not wavery in their moods etc.. etc 10 15 16 35 40 41 45 52 55 60 65 75 80 95 105 130 145 148 GROUP 4 DAYDREAMERS 50 100 107 125 GROUP 5 PATIENCE AND ATTRIBUTE OF TAKING RID=GHT DECISIONS AFTER CAREFUL ANALYSIS 22 27 47 67 72 86 92 96 102 112 117 126 134 136 137 140 142 147 GROUP 6 EASYGOING AND CAREFREE NATURED PERSONALITIES 20 25 32 57 68 77 82 87 90 98 115 123 127 132 135 150 GROUP 7 ENTHUSIASTIC PERSONALITIES 17 42 66 78 85 99 110 141 144 GROUP 8 SLOW SLUGGISH DULL PERSONALITIES 11 21 30 31 36 46 51 56 61 76 81 91 106 116 101 146 GROUP 9 SERIOUS PERSONALITIES 12 26 37 62 97 111 120 124 143 GROUP 10 STRONG FULL OF ENERGY 121 122 131 GROUP 11 DIFFIDANT (shy, not able to ask even the most required thing.. etc..) 18 19 23 33 34 38 43(34) 48 58 63(34) 69 83 84(38) 93 104(34) 108 113 118(34) 129 133 149 read the following before proceeding the last group is bigger and within the group itself a small subgrouping is done- the no. in the bracket is the parent question to which the no outside the bracket belongs to- hope u understand strictly- this may not be 100 % right as many questions look very similar. So go thru and make alterations if u feel so. in each groups there r 2 kinds of questions- if u say yes to some and no to others u will land up at the same definition. So its ur common sense to understand the affirmative and negative tones of each question and answer accordingly- u shld know when to answer no and yes for each question in the same groupsome groups itself are opposite to each other and can be grouped together- but I have separated them on the basis of finer inferences eg:serious persons cant be easy going type. if u say no to any question on serious group question u will land as an easygoing personality and vice versa- so they can be put under a single group- but some finer points prevent from doing so Q1:(5 marks) a wall clock loosses 10 min in 1hr 4 one hr of wall clock table clock gains 10min for 1 hr of table clock alarm clock looses 5 min for 1hr of aramm clock,wrist watch gains 5min they were set correctly at 4 pm what was the time of wrist watch at 6 pm(of correct time clock)

Q2:3 marks mr.jonh was a inspector of falt finding in balancing sscal he found that 1arm was longer than the other one wn 8 pyramids were kept in the longer the cubes required were in the short arm. wn 1 pyramid is kept on longer arm the cubes required at shorts arm was 6. if the correct wt. of pyramid is 1 ounce.? is the true wt.of one cube. Q3:6marks they were 2 twins one of them say nothing but lies on mon,wed n fri. n truth on other days.2nd one says nothing but lies on tus ,thi, n sat. only on sun both speak truth. ........(some statements was given n we have 2 find correct one) which wer typical. Q4:5 marks 3(a,b,c) players along with their wife(d,e,)f play a golf game.who score less will win. altogether there were 18 games. c score 100,d102,e106 94 was the score of b 2 other man a,b scores 98 n 96. b's wife beats a's wife it was found that 2 pair score identical identifie the pairs Q5:8marks x y z + a b ___________ c d e f identifi x y z a b ----------b g a

d,e f g

Q6: 4 marks a father n boy rowin in rever up stream wn they move one mile boy noticed that his father hat was flown with current of rever.it was passed five min. in that procces they return back down stream n took the hat from the startin position in 5 min. tell the speed of the water current. Q7:8marks language q with some conditions which i dont remember Q8:6MARKS

there were 5 shopkeepers having pets, each named their shops on others pets name. they were putting the names of their friends pet as there shop name. there were some condition given& it was required to find the names of one of the shopkeepers pets name. q9: 5marks one neighbour said to other this year she had more squrer plot of land to cultivate cabbage if she could cultivate 599more cabbage this year wt is the size of the plot this year. ---------------------------------------------------------------------ENGLISH TEST (30 min) 7 Parts (no -ve marking) 1st 2nd 3rd 4th 5th 6th 7th Part part part part part part part :comprehension (easy) :fill most appropiate words(tougher) :fil in correct tense :choose the correct statements between four :choose the correct statements havin an error beween two : fill in the correct phrases : prepositions

BHU 2003 INFY PAPER (1) Sakunthala Devi Puzzles to Puzzle you 69

(2) The ball has thrown from 180 ft height, every time it jumps 1/10th of its height. How much distance it will travel. Ans : 220 ft. (3) Lusia has some sons, every son has sons as many brotehrs. Her age is between 50 to 70. What is the age of Lusia Ans. 64. (4) There is a squares are cut it. What is the == length == 14 rectangular sheat of unknown dimension. One inch from the pour edges. After it the box was made from dimensions of the volume of the box is 60 inch. Ans b breadth == 7.

(5) Trent bridge has a width of 4100 inches. One eight of the area is one side of the river and 1/7th is another side. Then what is length of bridge. Ans 5000 ft. (6) One problems from summers mostly in ( 1 to 5)

(7) There is some number of stations. For every station the tickets will available from that station. In that route some new stations are

opened then 34 tickets are sold out extra. (a) How many stations before : 8 (b) How many stations new : 2 (8) One man is coming down from the the 7 steps. After it another person stairs. When second one reaches top, more steps to reach bottom. How many steps fro second one. Ans 22. top of the stairs and he downed stated from the bottom of first person has to travel four steps are there. 1st one takes 2

There are 7 small bags and 18 laige bags with marbles. In travwelling those two bags are broken. That's why marbles are taken out individually the number of market are 233. Then how many marbles in each bag. Ans small : 5 large 11. if it takes 7 cats to kill 7 rats in 7 different ways in 7 days then how many days will it take for 10 cats to kill 10 rats in 10 different ways? ) Cynthia and Reuben go to town to shop. Reuben purchases a hat and a suite, which costs him 15 cents. Cynthia purchases a hat, the cost of which is equal to Reubens suite. The rest of the money Cynthia spends in buying a new dress. The new dress costs her one rupee more than her hat. Cynthia says, if one and a half times money of hat, had we divided unequally, we would had spent equal amount of money. a) Reuben says," In that case", what is the cost of my hat? b) How much money altogether they spent Part-I: Puzzles 60Min, 50Marks 1.If 7 cats can catch 7 rats 7 ways in 7days,then 10cats can catch 10rats 10ways in how many days? 2. S E N D 6 5 2 4 2 8 1 9 2 8 1 7 M O R E 0 7 3 5 0 3 6 8 0 3 6 8 ------------- --------------------------M O N E Y 0 7 2 5 9 0 3 1 8 7 0 3 1 8 5 ------------- --------------------------3. Find the total number of rectangles (include squares also as rectangles) in a 8 * 8 standard chessboard? ANS. 1296 4.Find the minimum no of tyres required for a standard old modeled four-wheeled car to travel 48000miles,if 24000miles per each tyre? 5. Mr. Sekhar and Ms. Sekhar are having conversation like this: Mr. Sekhar said to Ms. Sekhar: "Give me three quarters of what you have , i will add it with what i have, totally to buy a house at indiranagar for 5000/-,remaining at hand with you can be bought the running stream and groove". Ms.Sekhar, said ,"No, No, give me two thirds of what you have , i will add it with what i have ,totally we can buy the house ,and the groove and running stream we can buy with the remaining money we have". can you find the money required to buy the groove and stream? 6.EXACTLY LIKE THIS QUESTION WITH NAMES CHANGED (Practise this question) (1) Ashland is north of East Liverpool and west of Coshocton

(2) Bowling Green is north of Ashland and west of Fredericktown (3) Dover is south and east of Ashland (4) East Liverpool is north of Fredricktown and east of Dover (5) Fredricktown is north of Dover and west of Ashland (6) Coshocton is south of Fredricktown and west of Dover BG A EL F C D 1)Which of the towns mentioned is furthest to the northwest ? (a) Ashland (b) Bowling Green (c) Coshocton (d) East Liverpool (e) Fredericktown 2). Which of the following must be both north and east of Fredricktown? (I) Ashland (II) Coshocton (III) East Liverpool (a) I only (b) II only (c) III only (d) I and II (e) I and III 3. Which of the following towns must be situated both south and west of at least one other town? (a) Ashland only (b) Ashland and Fredricktown (c) Dover and Fredricktown (d) Dover,Coshocton and Fredricktown (e) Dover,Coshocton and East Liverpool 4. Which of the following statements, if true, would make the information in the numbered statements more specific? (a) Coshocton is north of Dover (b) East Liverpool is north of Dover (c) Ashland is east of Bowling Green (d) Coshocton is east of Fredericktown (e) Bowling Green is north of Fredericktown 7. Two weather reporting resources DD-I and BBC gave a report for the rain coming in Bangalore city.The guantee of rain not to come given by DD-I is 2/3, while the gaurantee of rain to fall given by BBC is 3/4.The chance of rain to fall is more.If rain falls, what is the guarntee to fall.(Values not exactly) 8. 9. Both these puzzles are good,and easy, but very big to remember and prepare,.... Part-II : English 30Min , 45 Marks All are very easy, we can score 45/45..IT-Surat ********************************************************************** * INFOSYS **********************************************************************

* > > > > > > > > > > > > > > > > > > > > > > > > > > > > > > > > > > > > > > > > > > > > > > > > > > > > > > >

The Questions are follows 1. Number of null pointers in any binary tree == n+1 2. max(t1,t2,...tn) 3. 50% -DBETXXXXXX == pipelining - density

4. print (Head(T)) Traverse(left(T)) print (Head(T)) Traverse(right(T)) 5. Boolean expn Evalvate 6. Common subexpn : 7. LRU : 1, 2, 3.

- ans: none of the above

- ans : a + e

8. Tr. Delay - 10000 bits ans. 10.01 9. Grammar of Number of shift / reduce operator 10. CPU scheduling 11. if even x/2 else p(p(3x+1)) 2^k + 1: 3 . 2^(k-1) clarify this with sans 9,8 ? : ans. 4

12. allocation 13. swapping :

ans: (ii) only ans: reference only

14. Compiler - related Qn. 15. LAN frames - ? related Qn.

16. parameter passing (35,20) 17. sliding window protocol BUFFER SIZE large 18. 19. 20. kernel mode deallocate resource

logic circuit ans . Minimum OR == 3 Combinatorics related

21. priority scheduling 22. cobegin begin x == y; x== x+1; y== x

> begin x ==y; z== z+1; y== z > coend > > ans. Number of values possi == 2 > > 23. 2 bits flip / 2 bits exchange > > ans : the word with one '1' > > 24. any addr > K^+ v(a) + 2I - 2a > > clarify with SANS. > > > Here are the infosys questions which appeared here., They had > five sets of que. papers A,B,C,D,E. This is set A. Others > are also of similar type with few modifications. > love-kalya. > > ********************************************************************** > INFOSYS ********************************************************************** > > > There are 5 q papers with them. this is only one of those. > > All otheres were more or less of the same model but different > > qs. This is just to give U an idea. > > > > The questions are not in order. > > > > 1)A,B,C,D,E related.4 of them made these statements each. > > i)C is my son in law's brother. > > ii)B is my father's brother. > > iii)E is my mother in law. > > iv)A is my brother's wife. > > who made these statements?(person mentioned is one of A,B,C,D,E) (10 mar ks). > > 2)e means belong. > > All members of E e D. > > All members of D e A. > > Not all members of D e E. > > Not all members of A e D. > > All members of C e both A and B.some questions are asked about relatio n.use venn diagram.(5 marks). > > > > 3)complete the table. > > > > Played won lost draw goals goals > > for against > > A 2 2 1 > > B 2 1 2 4 > > C 2 3 7 > > > > A,B,C are 3 hockey teams.(2 marks). > >

> > 4) A says Party was held on :Thursday ,May 8th. > > B says Party was held on :Tuesday,May 10th. > > C says party was held on :Friday ,June 8th. > > Given April 1 st was Tuesday.one of A,B,C says 1 correct.one says 1 > > wrong.and one was completely wrong of date,Month and day. Find the > > Day the party held. (5marks). > > > > 5) A ship is away from the shore by 180 miles.A plane is travelling at > > 10 times speed of the ship.How long from the shore will they meet? > > (2marks) > > > > 6) Every station in N railroad issue everyother station's ticket. > > some stations are added.Now they have to issue 46 more tickets. > > say the No.of stations after and before added.(5 marks). > > > > 7) 3 persons say these statements. > > A says either Democratic or liberal wins the elections. > > B says Democratic wins.C says neither democratic nor liberal wins > > the election.of these only one is wrong.who wins the election? > > (5 marks). > > > > 8) A clock showing 6 o'clock takes 30 secs to strike 6 times.How long > > will it take to strike 12 at midnight?Ans.66 secs.(2marks) > > > > 9) Only boys aged > 16 wear coats. > > Boys aged > 15 go to watch football.some more statements are given. > > What can be said about those who are watching football ? (age and > > costume) > > (5 marks). > > > > 10) There are 3 societies A,B,C having some tractors each. > > A Gives B and C as many tractors as they already have. > > After some days B gives A and C as many tractors as they have. > > After some days C gives A and B as many tractors as they have. > > Finally each has 24 tractors.what is the original No.of > > tractors each had in the beginning? > > Ans.A -39. > > B- 21. > > C- 12.(7 marks). > > 11) 4,5 statements.From that find the answer.(7 marks). > > > > > > Reference books > > 1.) Puzzles and teasers by summer's > > 2.) Shakuntala Devi. (puzzles). ********************************************************************** SMS using the Yahoo! Messenger; Download PSG 2003

1.A number gives a remainder of 1 when divided by 2,3,4,5,6 and is divisibe by 11?whats the number? Ans 121 2.There r three desks which consisting of 2 drawers each.Each drawer is filled either with a black or red ball.There r 3 labels naming RR,BB,RB.one desk consists of both red and another both blue and the third ared anda black ball.If these labels are to be pasted accoridingly to their corresponding desks whats the probablity that only one lable is pasted wrongly? Ans zero.(it cant be only one wrong, if one is wrong,the otherone is also wrong) 3.My age is 3 times my sons age and my fathers age is 40 yrs more than twice my age.all our ages combine to give 1240 yrs.Whats my age? Ans 360 yrs 4.Jack and Jill went up and down a hill.They started from the bottom and Jack met jill again 20 miles from the top while returning.Jack completed the race 1 min a head of jill.if the hill is 440 mts high and their speed while down journey is 1.5 times the up journey. how long it took for the Jack to complete the race ? ansMay be 6.3 min or 12.6 min(not sure) 5.how many nos formed by using the digits 1,2,3,4 with no repititions are divisible by 4? Ans no. which is divisible by 4 must have the last two digits divisible by 4. the last two digits are 12, 24,32 the nos.are: 3412,4312,1324,3124,1432,4132 the nos. are 3(2! * 2) == 6 6.i have to catch a train by 6 AM. there is still time remaining to catch the train.My friend told me that 50 min ago the time remaining was 3 times as much time as that after 3 AM.How much time is remaining for my train? Ans 26 min (Sakunthala devi's Q no 2) 7.There is one GEORGE SUMMERS puzzle it goes like Three tribes nororeen,sororeen,midroreen one who always speaks truth and onealways false and one alternatively truth and false and some statements are given by A,B and C.based on those statements u have to guess who is who?(who always speaks truth and who false and ......) Ans three tribes . A-nororeen- false always B- sororeen- true always C-midroreen- alternate true and false 8.There is a truck driver starts from one place to pick another person at another place.The driver goes to that place by 6 AM.and then picking him he returns back.one day the other person woke a bit early and started walking towards the first place.one the way he meets truck driver and joins him and both went back to the first place.This day the other person came 20 min early to the time he usually comes to the first place?Tell the time when the driver met the second person that day? Ans 5.50 AM(Sakunthala devi's "WALKING BACK TO HAPPINESS")

9.analytical question.....this was based on the speculation of 3 ppl. whose guess is right? ans: alcit (3 marks) 10This is another common Infosys puzzling question. It is about some goals in a football match. The scoring system was given and the points scored by each team was asked. This, I was told was frequently asked in Infosys questions, so you should be able to get your hands on this one too. teams goals against A 0 B C Ans teams against A 0 B 2 3 4 played 3 3 4 3 1 played 3 1 1 1 0 won lost 0 tie 1 goals for 1 goals 1 won lost tie goals for 1

C 3 1 0 1 4 1 11.Four girls S,Robin,E,Mandy belonging to 4 families Clerks,Miller,Flure,Jacobs make salads out of four fruitssuch as apples,bananas,cherries,grapes. Each girl uses three fruits.No two families use the same combintion of fruits. 1.Robin who does not belong to miller's family uses apples 2.Clerk's family uses cherries and grapes. 3.Flure family use either cherry or grape 4.E does not belong to miller or Clerk family 5.Mandy and miller family use apples and cherries. Questions a.Which family did Robin belong to? b. Which combination of fruits is used by miller family c.Which fruit is not used by mandy d. Which 2 fruits r common beteen Robin and E answer(Check this)a-apple b-banana c-cherry g-grape S Robin E Mandy Miller Flure Jacob Clerk abc abg cgb cga The topic of the comprhension is "Avalanche kills" The first line of the comprehension generally gives the idea about the topic etc. Look for the structure and not for the details here. The most important question in the reading comprehension is the question where "u will be asked to apply a learnt idea to a new situation" This is exactly what they are looking for. They will be sure to mention that in the pre-placement talk. Check out mathforum.org and allstarpuzzles.com Solve old papers. Atleast 4 -5 problems get repeated.

VERIFONE Verifone test Questions : There are two parts : Note: The Answers given here are what i wrote, May not be correct. 1. Aptitute test : 15 Minutes, 20 Questions

Towards the middle questions are easy than from the front. eg. Product of three consecutive nos. 210. What is the sum of two least numbers? ans.: 5 * 6 * 7 = 210 , sum = 11 is answer eg. If the area of the sqaure is increased by 69 % how much the length of the side will increase? ans.: 13 (i think) eg. if the sum of five consecutive nos. 35? how many prime nos are there : ans: 5 + 6 + 7 + 8 + 9 = 35 so two primes eg. if the length of the rectangle is reduced by 20% and is increased by 20 % what is the net change ? ans.: 4 % decrease 2. 4)if u were given 6 sticks of equal size and how will u form 3 equilateral(only 3 not more not less) triangles using them ? 1. In 1978, a kg of paper was sold at Rs25/-. I f the paper rate increases at 1.5% more than inflation rate which is of 6.5% a year , then what wil be the cost of a kg of paper after 2 years? a)29.12 (b) 29.72 (c) 30.12 (d) 32.65 (e) none of these 2. In A,B,C are having some marbles with each of them. A has giben B and C the same number of marbles they already have to each of them. then, B gave C and A the same no. of marbles they have, then C gave A and B the same no. of marbles they have. At the end A,B,and C have equal no. of marbles. (i) If x,y,z are the marbles initially with A,B,C respectively. then the no of marbles B have at the end (a) 2(x-y-z) (b) 4(x-y-z) etc. (ii)If the total no. of marbles are 72, then the no. of marbles with A at the starting breath

a. 20 b. 30 c. 32 3. If a car starts from A towards B with some velocity due to some problem in the engine after travelling 30km.If the car goes with 4/5 th of its actuval velocity the car reaches B 45min later to the actual time. If the car engine fails ofter travelling 45km, the car reaches the destination B 36min late to the actual time , what is the initial velocity of car and what is the distance between A and B in km ans) 20 & 130. 4. A person has Rs 100/- in his pocket, he can as 25 pencils or 15books. He kept 15% of the money for travelling expenses and purchased 5 pencils.So how many books he can purchase with the remaining money. 5. ten questions on analogies. ex: hammer : carpenter :: knife : butcher. 6. The values of shares A,B and C from january to june are as follows. OB month A B C JAN FEB MAR APR MAY JUNE 30 35 45 40 55 50 60 65 75 75 75 75 80 85 65 82 85 80

i) During this period which share has undergone max fluctuation? ii) In which month it is possible to buy B and C selling A? iii) In which month the share values are very low? iv) By purchasing one share of A and 4 each of B and C in the beginning of the period , to get max profit when this shares should be sold? v) ? 7. In a computer institute 9 languages can taught. The module is of 6 months duration and only six languages each of one month can be taught. In addition to that BASIC is always there and

should be in first month itself # word perfect is to be taught in the preceeding week of word star. # FORTRAN can not be taught until COBAL is coarsed prior to that # BINO, FIFO never be taught in single module languages are BASIC, WORD STAR, WORD PERFECT, FORTRAN, COBAL, BINO, FIFO, LOTUS, C i. Of the following which module is possible based on above conditions. ii) If word star is in 3rd month , what could be in 6th month. iii) If COBAL is in the 2nd month and BINO in 6th month are there in addition to the above condition, FORTRAN will be in which month. 8. In a class , except 18 all are above 50 years. 15 are below 50 years of age. how many people are there a) 30 b) 33 c) 36 d) none of these. 9. A square plot of some size , at four corners equal squares of some size are cut and is formed as open box. If this open box carries 128ml of oil. What is the size of the plate i.e. side a.17 b.14 c.13 square 10. In a square , all the mid points are joined. the inner

is shaded. If the area of the square is A, what is the shaded area? 11. two questions on basic angles i.e given a circle, a few chords or diameter is drawn etc. 12. @(a,b)= (a+b)/2 /(a,b)= a.b *(a,b)= ab , if a=1, b=2 find i) /(a,(@(a,b),*(a,b))) ii) 13. (x#y) = x+y-xy (x*y) = (x+y)/2 i) (x#y)#(x*y) < (x#y), which of the below values of x, y will satisfy this equation ii) (a*b)#(b*c)< (a#b)*(b*c) , what values of a,b,c satisfy the above. 14. By using the data given below answer the following

questions. B.tech M.sc male female total 20 80 M.A

60 some thing similar to that question. i) 40% of females are B.Techs ii) Half of the students are either from B.Techs

of M.Scs

iii) ... i. what is the no. of female B.techs ii....

it consists of number series.In some institutes alphabetical series is given instead of number series.Iam having number series so iam sending that.Please go through tha alphabetical tests also. 1. 19,24,20,25,21,26,? ans:22 2. 11,14,12,15,13,16,? ans: 14 3. 10,2,8,2,6,2,? a:4 4. 8,9,11,14,,18,23,? a:29 5. 25,25,22,22,19,19,? a:16 6. 14,2,12,4,10,6,? a:8 7. 7,16,9,15,11,14,? a:13 8. 40,42,39,44,38,46,? a:37 9. 3,18,4,24,5,30,? a:6 10. 18,20,22,20,28,20,? a:22 11. 18,20,10,12,4,6? a:0 12. 7,6,8,5,3,7,? a:4 13 9,18,21,25,20,? a:30 14 3,3,4,8,10,36,? a:33 15.30,28,25,20,34,28,? a:21 16. 4,8,16,32,64,128,? a:256 17. 8,16,24,32,40,48,? a:56 18. 13,11,14,12,15,13,? a:16 19. 6,18,36,108,216,648,? a:1296 20. 4,4,8,8,16,16,? a:32 21. 2,6,18,54,162,486,? a:1458 22. 4,20,35,49,62,74,? a:85 23. 10,18,15,23,20,28,? a:25 24. 4,10,8,14,12,18,? a:16 25 10,15,12,17,14,10,? a:16 part 2 consists of non-verbel reasoning(figures).So it is impossible for me to send those.(25 questions) part 3 (quantitative) 1.A clerk multiplied a number by ten when it should have been divided by ten.The ans he got was 100.what should the ans have been? a:1 2.If Rs20/- is available to pay for typing a research report & typist A

produces 42 pages and typist B produces 28 pages.How much should typist A receive? a:Rs12/3.The average salary of 3 workers is 95 Rs. per week. If one earns Rs.115 and second earns Rs.65 how much is the salary of the 3rd worker. Ans.105. 4.A 16 stored building has 12000 sq.feet on each floor. Company A rents 7 floors and company B rents 4 floors. What is the number of sq.feet of unrented floor space. Ans.60000 5. During a given week A programer spends 1/4 of his time preparing flow chart, 3/8 of his time coding and the rest of the time in debugging the programs. If he works 48 hours during the week , how many hours did he spend debugging the program. Ans. 18. 6. A company installed 36 machines at the beginning of the year. In March they installed 9 additional machines and then disconnected 18 in August. How many were still installed at the end of the year. Ans. 27 7. A man owns 2/3 of the market research beauro business and sells 3/4 of his shares for Rs. 75000. What is the value of Business. Ans.150000 8. If 12 file cabinets require 18 feet of wall space, how many feet of wall space will 30 cabinets require? Ans.45 9.A computer printer produced 176,400 lines in a given day. If the printer was in operation for seven hours during the day, how many lines did it print per minute? Ans.420 10. From its total income, A sales company spent Rs.20,000 for advertising, half of the remainder on commissions and had Rs.6000 left. What was its total income? Ans.32000 11. On Monday a banker processed a batch of cheques, on Tuesday she processed three times as many, and on Wednesday she processed 4000 cheques. In the three days, she processed 16000 cheques. How many did she process on Tuesday? Ans.9000 12. The cost of four dozen proof machine ribbons and five dozen accouting machine ribbons was Rs.160/-. If one dozen accounting machine ribbons cost Rs.20/-, what is the cost of a dozen proof machine ribbons? Ans.Rs.15 13. If a clerk can process 80 cheques in half an hour, how many cheques can she process in a seven and one half hour day? Ans.1200 14. In a library, there are two racks with 40 books per rack. On a given day, 30 books were issued. What fraction remained in the racks? Ans.5/8 15. The average length of three tapes is 6800 feet. None of the tapes

is less than 6400 feet. What is the greatest possible length of one of the other tapes? Ans.7600 16. A company rented a machine for Rs.700/- a month. Five years later the treasurer calculated that if the company had purchased the machine and paid Rs.100/- monthly maintenance charge, the company would have saved Rs.2000/-. What was the purchase price of the machine? Ans.Rs.34000 17. Two computers each produced 48000 public utility bills in a day. One computer printed bills at the rate of 9600 an hour and the other at the rate of 7800 an hour. When the first computer finished its run, how many bills did the other computer still have to print? Ans.9000 18. If a salesman's average is a new order every other week, he will break the office record of the year. However, after 28 weeks, he is six orders behind schedule. In what proportion of the remaining weeks does he have to obtain a new order to break the record? Ans.3/4 19. On a given day, a bank had 16000 cheques returned by customers. Inspection of the first 800 cheques indicated that 100 of those 800 had errors and were therefore the available immediately for data processing. On this basis, hwo many cheques would be available immediately for data processing on that day? Ans.14000 20. A company figured it needed 37.8 sq.feet of carpot for its reception room. To allow for waste, it decided to order 20% more material than needed. Fractional parts of sq.feet cannot be ordered. At Rs.9/- a sq.feet, how much would the carpet cost? Ans. a. Rs.324 b) Rs.405 c) Rs.410 d) Rs.414 e) Rs.685 21. A tape manufacturer reduces the price of his heavy duty tape from Rs.30/- to Rs.28/- a reel and the price of a regular tape from Rs.24/to Rs.23/- a reel. A computing centre normally spends Rs.1440/- a month for tapes and 3/4 of this is for heavy duty tapes. How much will they save a month under the new prices? Ans.Rs.87 22. In a team of 12 persons, 1/3 are women and 2/3 are men. To obtain a team with 20% women how many men should be hired? Ans.8 23. The dimensions of a certain machine are 48" X 30" X 52". If the size of the machine is increased proportionately until the sum of its dimensions equals 156", what will be the increase in the shortest side? Ans. 6" 24. In a certain company, 20% of the men and 40% of the women attended the annual company picnic. If 35% of all the employees are man, what

percent of all the employees went to the picnic? Ans.33% 25. It cost a college Rs.0.70 a copy to produce a Programme for the homecoming football game. If Rs.15,000/- was received for advertisements in the programme, how many copies at Rs.0.50 a copy must be sold to make a profit of Rs.8000/- ? Ans. 35000 "A monster rises from the ocean and gets into an island.The island has only ten people.The monster threatens to kill all the people.But after sometime it decides to give the people a chance to live on a condition. It wanted the people to stand in a line one behind the other.Forming such an arrangement the tenth man can see the rest nine in front of him,the ninth man can see the remaining eight men in front of him and so on.The monster would place one hat on top of each man.The hat can either be black or white but it will never be all white or all black,i.e. the no of black:white will always be either of 1:9,2:8,3:7,4:6,5:5.6:4,7:3,8:2,9:1.The number of black or white hats is at the mercy of the monster and cannot be predicted or assumed.The people were allowed to survive if they can tell the colour of their hat correctly when the monster asks them.If a man made a mistake he was sent to killed.There is also a restriction that no one can communicate to each other.You are required to dedsign a strategy by which you can save the maximum number of people."

1.problems on permutation combinations like BANANA,.. 2.if april 18 was Wednesday in 1998,when is april 18 in 1999? 3.if B has its shadow falling on right side early morning ,then in which direction is he facing? 4.age problems like. Mothers age was twice the the age of son two years ago.after how many years will she be three times her sons age?? 5.analyzing data interpretations and around 4-5 questions based on tat 6.a Guy has some chocolates with him he distributes them equally among 5 people and remaning with him is 3.if he distributes them equlay among 7 people remaining is 5 with him. Which of them isn't is possible no.of chocolates he had?? 53,208. 7.4-5 problems on profit And loss 8.in a test moti,ganesh,manali,rupali,raj, appear,according to the scores moti is not the leats scorer but is lower to raj.who occupies the 2 nd postion is the order? 9.mr a meets mrs b.mr b is has a son and daughter .son is moti and is married and has a son .mrs moti is mr a mother .how is mr a related to mr b??

10.ram buys some dozens of apples and peaches .their price ratio is 5:7.if the price of 1 dozen of peach is is rs 9,how many dozens of apple did he bring? 11.problems based on relationships like those in rs aggarwal 12.in a queue A is is in 7 th positon from the left and B is 9 position from the left .if they switch positions A is in 11 th position ,find the total people in the row?? MATHS 1. S 1=={1,2,3,4}, S 2 =={A,B.C.Z},S 3=={},(S1*S2)U S3==? 2. IF 5*4=,8*7=B,how much is 6*9==? 3. 4 to 5 questions on simplifying complex nos. equations 4. sphere of radius 4.5 cm is divide in three spheres ,two have radius 1.5 and 2 cm,find the other ones?? 5. if COMPLEX == 81,and.how much is ANALYSE==?? 2-3 questions based on similar pattern 6. two taps a and b fill up a cistern in 2 and3 hrs ,at wat time should b be clsed if the tank cistern is filled up in 17 minutes?? 7. a doctor checks 5 patients every 3 hrs with abreak of 10 minutes between each two check ups.how many he patients he checks in 10 hrs and 15 minutes?? 8. no. of diagonals friends if vertices of a octagon are joined?? 9. an equilateral triangle is formed by joining the centers of sides of a equilateral triangle.wat is the ratio of area ,s of both the triangles?? 10. A can finish the work in 3 days , B can finish the work in double efficiency,can finish with equal efficiency of both A and B.if all work together within how many days will they complete the work? 11. A.B,C,D stand at the edges of a square.they start moving along the square sides.after clockwise rotation where will each be?? 12. if 1 is stop,2 is run,3 is go,4 is wait,5 is walk,then would be the next stage if the following pattern 5 5 4 3 2 1 .. 412231? 13. C remembers meeting B after 14 but before 18 ,but D remembers meeting B after 16 but before 18.when did both meet B?? 14. avg age of boys in a class is 12.when the age of teacher is included it is 13.wat could be the age of teacher? 35 , 45,53,50

the 2nd set was repeated frm NIT jamshedpur.most of the ques are

repeated ..but there are some sets that are real tough... written test is very simple filteration will be done in GD. written test consists of 2 sections(apptitude and logical reasoning) of 30 quest. each and cut off is 12 each.and the total time is 1 hour. 1. which is greater (1000)pow1001 and 1001pow999 2.one container contains milk and water in the ratio 3:7 and the other contains 8:11,in what ratio these two containers is to be mixed so that the ratio of milk and water is 4:5. 3. there are two simple problems on time and work. 4.four points will be given and you have to frame two st. line eq.s in such a way that their point of interesection lies in one of the four options. 5.there are two to three problems on profit and loss which are little bit time taking ,so i didn't attempt. 6.a series of nos will be given where the ans can be found by observing the diff bet two consecutive nos logical reasoning 1. air is cloud cloud is rain rain is water wateris sand so what is cloud? ans :sand 2. one quesion on relations 3. trafic: signal ans : river :dam 4. two more quesions are there 5. dsoighkl now if lk-(?)-sd , find letter in 4th place 6. some mammals are donkeys allbuffalos will have horns based on this 3 quesions r ther 7. two more r simmilar to above 8. to decode the noise :H.................M ANS:C 9.cube is of size 5*5*5 .every side has been coloured. it is divided into 125 equal parts. 1) what is the no. of parts having only one side coloured - 54. 2) no. of parts having two sided coloured 36. 3) having no side coloured - 27. 10.find the no. of occurrence of T which is immediately preceded y P and no timmediately followed by S in some series for eg. (TPTSTRUST.......) ans - 3 11.Four persons are there wearing different coloured shirts eg. A,B,C,D wearing blue ,green, red, yellow. Now, 1. A cannot wear yellow. 2. B can wear blue or green. 3. C /D is wearing yellow. You have to find who is wearing which coloured shirt. 12.Find the root of 4a2+b2+c2+4ab-2bc-4ac Ans: 2a+b-c.Two pipes can fill a tank in 5 hrs and 8 hrs. while a hole can empty it in 40 hrs. What will be the time taken to fill if each operate at a time.

Soln: 1\x == 1\5 + 1\8 - 1\40 13.A can beat B by 20 mts. While C can beat B by 40 mts. In a race of 100 mts. By how much can C beat A? Soln: 75 mts. 14.If u start your journey 30 minutes late , u have to increase your speed by 250kms\hr. to cover up 1500 kms. In same time. What is your usual speed? Ans: 750kms\hr. 15.For a circle, radius is inc. by some % , find net change in area? these r some of the quesion me and my frends remember note : these r from 5 different sets gd topics are lov marriages r arranged marages r tv serials helpful Total no. of sets 0716 , 0717 , 0718 out of which 0717 was Hard . I am sending the qus of 0717 . other section?s qus . were not managed. Also other sections were easy in comparison to 0717 .in my college most of the students having 0717 set were not in the written test . it was time taking . I don?t remember the logic qus. For it go thru the R.S. AGRAWAL . Go thru 1>blood relation. 2>analogy. 3>coding ?decoding. 4> puzzles. Qus related to these topics were asked . I am sending you few questions. 1)complete the series 600 ,180 ,54, ans===.2( options were given) 2) some doctors are fool john is a doctor a> john is a fool.b>all b> fools are doctors from this you have to draw the conclusion given in the options such as only a is true only b is tru both are tru or none is tru. 3> similar question like2 some teacher are female.a is female. a>she is a teacher. b>all females are teacher. 4>river : dam :: traffic: ans.==== signal other qus related to analogy were there .go thru R.S.AGRAWAL. other qus were from the lbood relations . some qus were from puzzles . a,b,c,d,e,f,g are 7 epople seating on a wall facing towards north.b is not at the end.c and d can not seat together . qus like this were there . mathematical qus / aptitudes.

1)a sum of money doubles in 5 years by simple interest . how much time it will take for 300 to become 2400. a>40 years b>25 years c>35 years d> 20 years. 2)if length of a rectangle is made 4 less and breadth is increased by 3 then the resultant square ?s area is equal to the rectangle. What is the perimeter of the rectangle .( options were given) 3)what will be the no. at ? position below 51 11 61 64 30 32 35 ? 43 4>the L.C.M. and H.C.F. of two nos. are 84 and 21 and the nos. are in the ratio of 1:4 ,. Find the nos. ans==== 21, 84 5)find the volume of the hemisphere having radius 2m . 6>problem based on compound interest were asked . so see the formula of compound interest. 7)a train goes from a to b. if it travels with 50 km/s then it is late by 10 mins. When it travels with30km/s then it bis late by 50 mins. Fi nd the distance between a and b. 8)a river is 8 m deep and 150 m wide.river?s speed is 5 km/s. find the volume of water passed in one min.(1 m cube =1000 c.c.) 9) fin dthe equation of the st. line passing thru the intersection of the two lines and two other lines.(equ. Were given) 10) a man sells 20 mangoes + 15 oranges ath the same price as 15 mangoes+20 oranges. Ho would judge which is costlier . ( four options were given . I didn?t remember those) 11) a graph was given representing joint venture in the years. An dquestion related to the graph were asked such as A)in which year there was max. change.(ans. Was in the last year) Two other qus were asked.they were long so I don?t remember them . 12)A is counting the no from 1 to 32 and b from 32 .A is counting the odd no. only .both?s speed is same. What will be the number which will be pronounced by A & B together. ( ans . aws none of these )but confirm yourself . 13) in the word ?DISTURBANCE? 1st is replaced by last 2nd is replced by 10th , 3rd is by 9th and so on. Which will be the next to ?T?. 14)GIVEN if x/y==3/5. if 1 is added to num . and 1 is subtracted from denominator then ratio ==5/7. find the number. 15)there are 5 black and 9 white balls .if one ball is drawn then find the probability of being white. 16)Avg. age of 5 children =.a child of 5 year age dies.after 4 years what will be the avg. age. 17)three nos. are given the product of first and last is equal to the square of the middle. Find the nos.(options were given) (ans.== 10, 20, 40) 18) a rectangle was having length 100 m breadth 60

m. there is a road of 5m wide on each side of the rectangle . find the area of the road. 19) fir no. is the double of 2nd and of the 3rd . other statement was also given which I don?t rememeber.(ans=H,24,96) 20) a girl cuts a cake in two halevs. One half is again cut into six equal parts . wt. Of the smaller one was 20 gms. Fin the wt. Of the whole cake.(ans=240gms.) 21)other question was based on the relative speed in the river . i.e. a person first goes upstream then down stream . river?s speed was given .also the diff. Of time was given . find the speed of the man. 22) ravana speed is 5000 km/s when going towards the heaven. The distance == 75000. ravana has traveled 2 mins. When god?s messenger reaches the earth . Rama told him to go back. The speed of the god?s messenger =`00 km/s. By how much he will increase or decrease his speed so that he and Ravana reaches exactly at the same time. 23) a man sells a product by giving 10% reduction on it .in spite of this he gets 10% profit . on a product of 330 rs. Original price What will be the buying price. The highest Score in an innings was 3/11 of the total and the next highest was 3/11 of the reminder. If the scores differed by 9, find the total score. a) 151 b) 161 c) 121 a d) 101 e) 137 Ans: c A boy was asked to multiply a certain number by 53. He multiplied it by 35 and got his answer less than the correct one by 1206. Find the number to be multiplied. a) 37 b) 67 a c) 87 d) 97 e) 107 Ans: 67 A Problem like this not exactly the same but on same model. If the manufacturer gains 10 %, the wholesale dealer 15 % and the retailer 25 % then the cost of Production of a table, if the retail price is Rs.1265 a) 632.50 b) 800 c) 814 d) 834.24 Ans: b A trader marks his goods up by 50% and declares two successive discounts of 20% each. What is his overall gain? a) 10% gain b) 4% gain c) 4% loss (A) d) 10% loss e) No loss No gain Train Problems - 3 as finding speed of train, how many secs it will cross each other etc. Age problems -3 , find mothers age,fathers age etc. Statistics Problems(probability) as A-this much , B-This Much, A n B-this much and A u B - ? Angle Problems which deals with triangle - 3 problems.

Section III - Logical reasoning - 30 qns in 30 mins. Tower of Hanoi Problem - Ans : 7 problem on fig. find the shaded area, square of size 14cm.. ans: 42 problem on symmetric fig ;; ans less than the 1172(check) . Blood Relation Problems - 3 Then a passage is given on blood relation and 4 qns asked how many male: ans -4 and few etc. Find odd man out of the given series - 3 problems pretty simple put A-Z and number them 1-26 and then u can find easily odd man out. Find the missing letter - was a bit tough to establish relation - 3 problems u have 3 figure -based on the figures ques asked.. easy one.. eg: Fig Triangle means Strong Fig Square means Tall Fig Circle means Fair. Ques asked is find the number that represents Strong Tall and Fair persons.. easy one .. ANs : 4 ANother question Women,Sub inspector,graduates - Ans: 3 An Flow Chart - GIven Units - 250 find final Value with formulae - (units*1.25) and few calculations like this Ans:506.25 And two tedious flow charts couldn't remember them and the answers.

IBM GLOBAL SERVICES 1. In 1978, a kg of paper was sold at Rs25/-. I f the paper rate increases at 1.5% more than inflation rate which is of 6.5% a year , then what wil be the cost of a kg of paper after 2 years? a)29.12 (b) 29.72 (c) 30.12 (d) 32.65 (e) none of these 2. In A,B,C are having some marbles with each of them. A has giben B and C the same number of marbles they already have to each of them. then, B gave C and A the same no. of marbles they have, then C gave A and B the same no. of marbles they have. At the

end A,B,and C have equal no. of marbles. (i) If x,y,z are the marbles initially with A,B,C respectively. then the no of marbles B have at the end (a) 2(x-y-z) (b) 4(x-y-z) etc. (ii)If the total no. of marbles are 72, then the no. of marbles with A at the starting a. 20 b. 30 c. 32 3. If a car starts from A towards B with some velocity due to some problem in the engine after travelling 30km.If the car goes with 4/5 th of its actuval velocity the car reaches B 45min later to the actual time. If the car engine fails ofter travelling 45km, the car reaches the destination B 36min late to the actual time , what is the initial velocity of car and what is the distance between A and B in km ans) 20 & 130. 4. A person has Rs 100/- in his pocket, he can as 25 pencils or 15books. He kept 15% of the money for travelling expenses and purchased 5 pencils.So how many books he can purchase with the remaining money. 5. ten questions on analogies. ex: hammer : carpenter :: knife : butcher. 6. The values of shares A,B and C from january to june are as follows. OB month A B C JAN FEB MAR APR MAY JUNE 30 35 45 40 55 50 60 65 75 75 75 75 80 85 65 82 85 80

i) During this period which share has undergone max fluctuation? ii) In which month it is possible to buy B and C selling A? iii) In which month the share values are very low? iv) By purchasing one share of A and 4 each of B and C in the beginning of the period , to get max profit when this shares should

be sold? v) ? 7. In a computer institute 9 languages can taught. The module is of 6 months duration and only six languages each of one month can be taught. In addition to that BASIC is always there and should be in first month itself # word perfect is to be taught in the preceeding week of word star. # FORTRAN can not be taught until COBAL is coarsed prior to that # BINO, FIFO never be taught in single module languages are BASIC, WORD STAR, WORD PERFECT, FORTRAN, COBAL, BINO, FIFO, LOTUS, C i. Of the following which module is possible based on above conditions. ii) If word star is in 3rd month , what could be in 6th month. iii) If COBAL is in the 2nd month and BINO in 6th month are there in addition to the above condition, FORTRAN will be in which month. 8. In a class , except 18 all are above 50 years. 15 are below 50 years of age. how many people are there a) 30 b) 33 c) 36 d) none of these. 9. A square plot of some size , at four corners equal squares of some size are cut and is formed as open box. If this open box carries 128ml of oil. What is the size of the plate i.e. side a.17 b.14 c.13 square 10. In a square , all the mid points are joined. the inner

is shaded. If the area of the square is A, what is the shaded area? 11. two questions on basic angles i.e given a circle, a few chords or diameter is drawn etc. 12. @(a,b)= (a+b)/2 /(a,b)= a.b *(a,b)= ab , if a=1, b=2 find i) /(a,(@(a,b),*(a,b))) ii) 13. (x#y) = x+y-xy (x*y) = (x+y)/2

i) (x#y)#(x*y) < (x#y), which of the below values of x, y will satisfy this equation ii) (a*b)#(b*c)< (a#b)*(b*c) , what values of a,b,c satisfy the above. 14. By using the data given below answer the following questions. B.tech M.sc male female total 20 80 M.A

60 some thing similar to that question. i) 40% of females are B.Techs ii) Half of the students are either from B.Techs

of M.Scs

iii) ... i. what is the no. of female B.techs ii....

dont remember this question , just for an example this 1. there are two colcks one runs 1min/hrs faster and other 1min/hr slower when will the two clocks have time time difference of 1 hr : ans simple 30hrs 2. i take a taki whose no is 3 digit no. it is not divisible by 2,3,5,7 but divisible by 11 it is the smallest no possible: ans 121 3. three friends start with x marbles each after first round A gives ...to B...toC, after second round B does similar , after 3rd round C does similar, they have now marbles as ..... How many marbles they started with: ans 20 simple trick is to form equation and solve backwards S= 0; C = 0 Read A,B While ((2A+3B) < 100) { S = 3A - 2B c=c+1 } Print a,s,c Following this there were 5 quetions. every ques carried 1 mark. Wat is the output if a = 26 b = 12 wat is the output if a = 24 b = 16 etc.. In Mathematics some ques were simple and others were a bit time consuming. |X| + X = 0 wat is the value of X a) 1 b) 0 c) 1,0 d) None of these If 38473ABC(This was not the exact no given) is divisible by 75 wat is ABC in the same order x + 1/x = 2 x^100 + 1/x^100 f(z) = 1/1-x find f(f(f(1/x))) The following questions got repeated from the pattern found in freshersworld.com In the series 0, 3, 8, 15,__ What is the next number?

Ans : 24 X < 0, Y <> 0 then which of the following is always positive? Ans. (x + y)^2 Let ax2 + bx + c = 0 If the sum of the equal roots is equal to the product of the same roots. Then which of the following hold true Ans. b+c = 0 Find the value of (1.99)2 Ans. 3.9601 There is a room with 6' x 8'. A 1' tile is fixed along the 4 walls in one row. How many 1" tiles require to finish the work. Ans. 24 2 persons can finish a job in 8 days. First person alone can finish the work in 24 days. How many days does the second person take to finish the job? Ans. 12 days In a car wheel, two spokes cover 15 degree. Then for the entire car, how many spokes are there? Ans. 48. What is the angle of degree suspended when two hands of clock showing the time 2.30. Ans. 105 degrees A person's salary is getting reduced by 20%. What percentage should be added to get back his original salary? Ans. 25% Two persons start at the same point, walk in opposite directions with 5km/hr and 5.5km/hr respectively. What is the distance separated after 2 and half hrs? Ans. 26.25 A person starts walking at a speed of 5km/hr through half the distance, rest of the distance he covers with a speed 4km/hr. Total time of travel is 9 hours. What is the maximum distance he can cover? Ans. 40km. In a rectangle the length is increased by of the original length . By what proportion should the width be reduced so that the area will be the same? Ans. 33 Find the nth number in the series is 1, -3, 5, -7.___ Ans. (-1)^(n-1)*(2n-1) If a square is formed by the diagonal of the square as an edge, what is the ratio between the area? Ans. 2 The perimeter of a rhombus is 52 units. One of its diagonal is 24 units.What is its second diagonals length? Ans. 10 A cubical rectangular bar has the dimensions with the ratio 5 : 4 : 3. Its volume is 7500. What is the surface area of the bar? Ans. 2350 3 persons started placementpapers with a capital of Rs.3000 . B invest Rs.600 less than A, C invest Rs.300 less than B. Then what is the share amount of B in a profit of Rs.886 ? Ans: Rs. 265.810. Find the reminder when 333666777888999 divided by 3 or 9 or 11 ?

11. Which is the biggest perfect square amongst the following 15129, 12348, 23716, 20736 12. In the series 0, 3, 8, 15,__ What is the next number? 13. X < 0, Y <> 0 then what is the possibility that the result is always positive? Ans. (xy)^2 14. Let ax2 + bx + c = 0 If the sum of the equal roots is equal to the product of the same roots.Then which of the following hold true (a) a + b = 0 (b) a = 0 (c) c = 0 (d) a + c = 0 15. Find the value of (1.99)2 Ans. 3.9601(dont calc - should be near to 4) 16. There is a room with 6' x 8'. A 1' tile is fixed along the 4 walls in one row. How many 1" tiles require to finish the work. Ans. 24 17. 2 persons can finish a job in 8 days. First person alone can finish the work in 24 days. How many days does the second person take to finish the job? Ans. 12 days 18. In a car wheel, two spokes cover 15 degree. Then for the entire car,how many spokes are there? Ans. 24. 19. What is the angle of degree suspended when two hands of clock showing the time 2.30. Ans. 105 degrees 20. A person's salary is getting reduced by 20%. What percentage should be added to get back his original salary? Ans. 25% 21. Two persons start at the same point, walk in opposite directions with 5km/hr and 5.5km/hr respectively. What is the distance separated after 2 and half hrs? Ans. 26.25 (approx) 22. A person starts walking at a speed of 5km/hr through half the distance, rest of the distance he covers with a speed 4km/hr. Total time of travel is 9 hours. What is the maximum distance he can cover? Ans. 40km. 23. In a rectangle the length is increased by of the original length . By what proportion should the

width be reduced so that the area will be the same? Ans. 33 24. Find the nth number in the series is 1, -3, 5, -7.___ Ans. (-1)*(2n-1) 25. If a square is formed by the diagonal of the square as an edge, what is the ratio between the area? Ans. 2 26. The perimeter of a rhombus is 52 units. One of its diagonal is 24 units.What is its second diagonals length? Ans. 10 27. A cubical rectangular bar has the dimensions with the ratio 5 : 4 : 3. Its volume is 7500. What is the surface area of the bar? Ans. 2350 28. A persons salary iis decreased by steps of 20%, 15% and 10%. What will be the percentage decrease, if the salary is decreased in a single shot? 29. 3 persons started placementpapers with a capital of Rs.3000 . B invest Rs.600 less than A, C invest Rs.300 less than B. Then what is the share amount of B in a profit of Rs.886 ? , 30. 178^2 -22^2 (USE (A+B)(A-B)) 31. f(x) = x! g(x)=x^10 h(x)= 10^x for large values of x which is greater? 32. f(x) = 1/(1-x) find(f(f(f(x))) 33. x+(1/x) = 2.5 find x^2 + (1/x^2) 34. 2^(2^n) = 256 find 2^(2n) 35. x=0 y=0 6x+7y=42 find position(lies inside/out/on) of (4,5) in the triangle 36. 22 Jun 2003 is sunday which year previously did 22 jun became sunday 37. A man walked 30m west 5m south 10m west 35m north to reach tollgate find the dist from start to gate. ans. 50m (use pythogoras) 38. area of square = sum of areas of rect1(41*27) and rect2(41*14). find square side. ans. 41 39. find next in series 31,29,31,30,31,?

ans. 30 he!he!he!(they rep jan,feb,mar,.... hit your head) Walk/Swim thro Coding 40. Input a,b s=0; c=a; d=b; while(d<>0) { s=s+c; d=d-1; } Print s for given values of a,b find the output 5questions for different values of a,b 41. Input a,b while(3a+2b<100) { s=s+a; a=a+2; b=b-1; } for given values of a,b (i remember one 29,5) find the output 250m away from a church tower of height 150m there is a tower of height 200m... there are two pigeon above each tower...they travel at a constant speed and reach the ground at same time to pick the grain. Say how far is grain from the second tower.... ans----i think 90. ____________________________________________________ 20 m rope is divided into 2 parts .each part square. A) sq.m B) sq.m find find forms perimeter of

the probability that the largest squares area is > 9.08 the probability that the largest squares area is >20.08

____________________________________________________ a town members decide to hire a band for a concert. They decide to pay rs10,000 and 20% of gate receipt. They expect about 10,000 people to attend a) b) find the admission fare to get breakeven. Toget profit of 20,000

____________________________________________________

there were 2 logical sentences each 8 marks...............4 conditions were given and then sequence of questions..they were easy.... ____________________________________________________ sita buys 3 silk and 4 rotten and pays rs 31cents........she comes back and ask the shopkeepertoexchange it for 4 silk and 3 rotten...shopkeeper says she is 1 cents shy.....find the cost of silk and rotten.... ____________________________________________________ there are 3 towns in syria.attacked by 3 dragons-x,y,z.......number of days x attack a town is equal to number of days y attacking another town. Number of days x attack is equal to half the squareroot of number of days z attacking a town........number of days y attacking the town is twice the square root of z......calculate howmuch days the curse of each dragon be. ____________________________________________________ one probability sum which i dont remember. ____________________________________________________ 1) 1. value of N and! K is100 2.N is divided by K and the integer part of the quotient is stored in I. 3. N is divided by K and the reminder is stored in K 4. I is multiplied by itself to get the new value of I 5. if the value of I and J are equal then N is printed 6. add 1 to N 7. if the value of N exceeds 999 then stop. 8. go to step 2 The result of the above logic is a) 124, 248, 369 ....b) 101 , 204, 309, .....c) 111, 222, 333, ...d) none ans. (b) 2) 1. Sony =0 2.jimmy =1 3.Add jimmy to sony to get new jimmy 4. add sony to jimmy to get new jimmy. 5. if jimmy is equal to 34 then stop 6. go to step 3 What is the value of Sony when the logic terminates. a) 15 b) 18 c) 21 d) none ans:d(check) 3) get the cost of the iteam (C)

1.get the no of iteams (N) 2.multiply N by C to get total cost (TC) 3.if Tc is above 10000 then discount (d)= 5%4. if TC is less than 10000 but more then 5000 and C is above 85 then D=4.5 % 5. if Tc is less than 5000 and C>65 and N>50 then D=4% What will be te discount if 40 iteams were purchased each costing Rs 58' a)5% b) 4.5% c) 4% d)none ans:d(check) 4) the average age of a class of 45 is 20 years. the age of the teacher is 30 years. the the average age of the 45 peaple together is 1.j=z * l /* here * means multiplication*/ 2.m=j+ 30 3.z= 20 4.r=m/(l+1) 5.l=45 the above logic represents the solution of the given problem. the correct sequence is a)5-3-1-4-2 b)5-3-1-2-4 c)5-3-2-1-4 d)none ans:b(check) 5) 1.k=10 2.add 1 to k to get the value of z 3.p= quotient of z divided by 10 and some statements are there... 8. Go to 2 the result of the above steps is a)11,22,33,.....b)21,32,33,..c)12,23,24...d)none ans! ...(b) 6) A long flow chat FN=10-----> k=10-----> divide FN by K and ...... ..... the number of values that are printed are a) 9 b) 19 c) 29 d)none ans (a) 7) 1. cost of iteam C= Rs 50 2. total cost TC= 0 3.number of iteams purchased =1 4.TC= TC+C 5.if the reminder of N divided by 5 is 0 then reduce Rs. 10 from C 6.increase the value of N by 1 using the above rule, how much money should one pay after buying 12 iteams a)480 b) 510c)540 d) none ans.....(d) check the answer. 8)flow chat every applicants takes 3 tests. the candidate is selected if he/she secures 80 % above in the aggregate and 75% and above atleast two testes.

all tests are evaluated to 100 marks. C=0------> Get T1,T2,T3------> Is T1>74......... ............................... if a candidate scores 96, 78, 68 in the three testes going by the above logic, his result will be a! ns...(b) selected 9)flow chat R=8 and J= 5 ---------> print R------L= R+j .......... the out put of the above logic is ans .... (d) none 10)flow chat N= 100 -------> D=INT(N/100) ------> K=reminder (N/100) .............> print N if the no like 244, 514,136 etc are to printed from the above logic , the ? should be replaced with ... ans.d(check)..... ***11) 1. let the value of FN be 1 2. let the value of A be 2 3.Let the new value of FN be FN multiplied by A and add 1 to it. 4.Write the value of FN 5.If the value written is more than 777 then stop 6.--------------------------7.go to step 3 if the expected out put from the above logic is 3,10,41,206,1237, then the statement in the line 6 should be (a)let the value of A be A multiplied by A (b) add A to A (c)add 1 to A (d) ! none ans.......(c) add 1 to a **12)flow chat N=2----> R= N+1, J=N x R . | ^ no | --------> print J----> N=N+1----.is N>9 ?

&n! bsp;

|------------------------------------------------the result of the above logic is (a)6,8,14... (b)6,12,20,30 ...... (c)6,12,18,24...(d)none ans.....(b) ***13) the followig qualification are necessary for recruitment of a librarian . the candidate must 1. have master degree in library science with at least 55% mark.

2.have one year specialization in IT 3.have at least 10 yrs experience in university In case the candidate 4.has 15 yrs experience in college ,the case may be reffered to vice chancellor 5.has obtained less than 55% marks but has 13 yrs expr.in university ,the case may be to resistrar 6.has PHD in library sc./documentation,the conditin 1 may be waived. gopal brahma has been deputy librarian of SLC university since 1982 and has a master degree! in library sc.with 53 % . given the above rules and Gopal's information,Gopal should be (a)selected (b) rejected (C) referred to V-C (d) none ans.....d:none(chek) ***14).flow chart ans:a ***15> 1.R=2 2.J=R*R-R 3.R=R+1 4.K=R*R-R 5.print K-J 6.let J=K 7.if R.8 then stop 8.go to 3 the result of the above logic is (a)4,9,16,25.... (b)4,6,8,10.... (c)4,4,4,4.... (d)none ans (b)(check) **16>flow chart is remainder of N=66---->D=N/2--->N=D+1------> N/2=0 ------>stop ^ ! | | |no |- ---------------| after how much iteration will the above logic terminate (a)8 (b)6 (c)7 (d) none ans.(d) check **17. 1.T=1 2.T1=T*1000 3.T2=T+1 4.T3=T2*100 5.T4=T2+1 6.T5=T4*10 7.T6=T4+1 8.T7=T1+T3+T6+T5 9.print T7 10.T=T+1 11.if T<8 then 2 12.stop which of the following value will not ptinted by above logic (a)6789 (b)7890 (c)7900 (d) none

! ;

ans (b) 7890 q18)) ans:c is j>10 Q19) flowchart ans:a)200 Q20) flowchart ans:a)21 Q21) flowchart ans:a)10 (confirm the question) q22) flowchart ans:c(4x+8y) q23) ans:c)4-6-5-1-3-2 q24)flowchart ans:b(17) q25)a person distribute his pen among four friends in the ratio 1/3,1/4,1/5,1/6..... ans:b(minimum he should have) Q26) 1.let the value of N be 10 2.divide N by 10 to give quotient (Q) remainder(R) 3.diff=R-Q 4.if diff=1 then 6 5.go to step 7 6.display the value of N 7.add 1 to N 8.if N <100 go to step 2 the above logic will produce (a)11,12,13,,14..(b)12,23,34,.....(C)21,32,4,354......(d)none ans:d(none) Q27)flowchart ans:b(36,45,25) Q28)flowchart hint: k=3,j=2......

ans:b(375) Q29)a man has Rs. 128 in currency of one rupee........ ans:a(24) Q30)when A is devided by B,it is reqd tro round of the result to the nereast whole no,the logic is giv en below .give the correct sequence 1. divide A by B giving correct s! equence 2.substract INTERGER(A/B) from C and store theresult in D 3.if reminder (A/B) is 0 then stop 4.print INTERGER (A/B) 5.if D>.49,increase the value of INTRGER (A/B) by 1 (A)1,3,2,5,4(B)1,3,2,4,5(C)1,2,3,5,4(D) none ans:a(1,3,2,5,4) Q31)A cistren is filled by 9 hours....... ans:a)90 hours Q32) ans:c(42) Q33) flowchart ans:d(none) **34> When madan said that he was burn on 31-9-83,everybody laughed.the logical sequence of steps involved in reduculing madan's statement r given below 1.if the day of birth is >Y(month of birth)then wrong 2.since 1983 is not a leap year february has 28 days 3. store 31,28,31,....31 in an array y(12) 4.month of birth is not less than 1 and not greater than 12, therefore month of birth is valid the right sequence of steps is a)3-1-4-2 b)3-2-4-1 c)3-2-1-4 d)none ans:b(check) 35). the logic to! convert a 3 digit number to a word is given, Ex 123 should be written as One Two Three

1. accept the number N 2. if N < 100 or N > 999 go to step 1 3. Split N into hundreds(H), Tens (T), & Units(U) 4. Store words zero,one,two,....... in array W(10) 5. print W(H),W(T),W(U) 6. Stop The above logic is a) incomplete b) complete c) complete but inefficient d)none ans: b 36) given a number which is more than 99 but less than 1000, the logical sequence of steps to say how many hundreds, tens & units are in the given number is given below.identify the logical order. 1.get the number N 2.write quotient and the words 'tens' 3.write the remainder and the words "units' 4.write the quotient and the word "hundred" 5.divide the number N by 100 and get the quotiient and remainder 6.divide the remainder by 10 giving the quitient and remainder (a) 1,5,6,2,3,4 (b)1,5,4,3,2,6 (c) 1,5,4,6! ,2,3(d) none ans.c **37. step 1.no of toffees in hand (TIH)=2 2.no of picks (NOP) = 1 3. Multiply TIH by 2 4. Add 1 to NOP 5. If NOP is less than 4 then step 3 6. Print TIH 7. Stop The value of TIH at the end of execution of logic is a) 16 ans: a Q38) flowchart ans:b (1,2,4,8,16,32,32,16,8,4,2,1) Q39) flowchart ans:b(2) Q40) ans:a)17 Q41) ans:b(6 days) b) 24 c)30 d)None

Q42) flowchart ans:b(I=468 , II=468) Q43) the logical sequence of steps to calculate the average of N nos is given below 1.let the sum(S) of N nos be zero 2.get the value of N 3.let the value of A be N 4.add N to S 5.add 1 to A 6.if A is not = N then go to 4 7. compute average by dividingS by N the abouve logic is (a)correct(b)incorrect(c)inadequate data(d)none ans:b(incorrect) Q44) indet! ify the correct logical order to write ur name in reverse order 1.count the no of alphabets in ur name(L) 2.if L=0 stop 3.rerduce the value of L by 1 4.write the lth character 5.transfer control to the step where the value of L is reduce (a)1,4,3,5,2(B) 1,4,3,2,5 (c)1,3,4,2,5, (d) none ans:a(1,4,3,5,2) **45. six girls A,B,C,D,E & F have 10 marbles each. a & B take 4 marbles each from D & E respectively. C gives 3 marbles to F who in turn gives 2 marbles to D & 4 marbles to E. E takes back the marbles she had given to B and gives them to C. The equations r given below 1. A=A+4 2. B=B+4 3. D=D-4 4. E=E-4 5. F=F+3 6. F=F-2 7. F=F-4 8. C=C-3 9. D=D+2 10. E=E+4 11. B=B-4 1! 2. C=C+4 The above sequence of steps are a)Complete b)Incomplete c)Incorrect d)none

ans:b 46. Subtract 3 from the number X. Multiply the result by 3, Add 3 & then divide this result by 3. The final result is ans:a a) X-2 b)X+2 c)X+1 d) X 47. Given a sentence which has only lower case letters and which ends with a period (.), you r asked to count the number of vowels & consonants. The logical sequence of steps r given below. Arrange them in correct order. 1. If character = a,e,i,o or u increse VC by 1 else increase CC by 1. 2. Print VC,CC 3. Read a character 4. If character read is . then go to print VC,CC 5. Go to read a character 6. Stop 7. Initialise VC & CC to 0 ur logical sequence of steps r a)7,3,4,1,5,2,6 b)7,4,3,1,2,5,6 c)7,3,4,! 5,1,2,6 d)none ans:c 48. 1.number of units consumed(NUC) = Closing reading - Opening reading 2. if NUC is less than 100 units then 5 3. if NUC is less than 200 units then 6 4. if NUC is less than 300 units then 7 5. CHARGE=NUC 6.CHARGE = 100+(NUC-100)*2 7.CHARGE=300 +(NUC -200)*3 8. Print CHARGE if the opening reading is 785 and the Closing reading is 865, CHARGE is a)80 b)60 c)-60 d)None ans: d Q49) ans: c)3,7,1,2,5,6,4 Q50) flowchart ans:b(prints multiplication table from 1 to 10) 1)two clocks meet at 12.Next time they meet again after Ans 65.45 2)Soldiers form a square .Initially 32 soldiers left.Then 8 .After they could not form a square.Total at the beginning

options 100,81,49,67 3)Age of Grandfather is the ages of 4 grandchildren where their ages are in consective.Find the age of GF I think 72 4)3 person waiting for the train Some conditions given.Finally they ask the time.Ans is 4.37 5)Some Squirrel qs where it covers 1/4 th distance then 1/2 i couldnt remember 6)if mpown=121.Then m-1pown+1 is 1000 7)if speed in the ratio 5:4:3 then the time taken to cover the same distance is 8)20men construct a wall 56m in some days similar one ans 49 (check aggarwal)1. Father's age is three years more than three times the son's age. After three years, father's age will be ten years more than twice the son's age. What is the father's present age. Ans: 33 years. (2 marks)

2. Find the values of each of the alphabets. NOON SOON +MOON ---------JUNE Ans: 9326 (2 marks)

3. There are 20 poles with a constant distance between each pole A car takes 24 second to reach the 12th pole. How much will it take to reach the last pole. Ans: 41.45 seconds (2 marks) Let the distance between two poles = x Hence 11x:24::19x:? 4. A car is travelling at a uniform speed. The driver sees a milestone showing a 2-digit number. After travelling for an hour the driver sees another milestone with the same digits in reverse order. After another hour the driver sees another milestone containing the same two digits. What is the average speed of the driver. Ans: 45 kmph (4 marks)

5. The minute and the hour hand of a watch meet every 65 minutes. How much does the watch lose or gain time and by how much? Ans: Gains; 5/11 minutes (4 marks)

6. Ram, Shyam and Gumnaam are friends. Ram is a widower and lives alone and his sister takes care of him. Shyam is a bachelor and his neice cooks his food and looks after his house. Gumnaam is married to Gita and lives in large house in the same town. Gita gives the idea that all of them could stay together in the house and share monthly expenses equally. During their first month of living together, each person contributed Rs.25. At the end of the month, it was found that Rs 92 was the expense so the remaining amount was distributed equally among everyone. The distribution was such that everyone recieved a whole number of Rupees. How much did each person recieve? Ans. Rs 2 (4 marks) (Hint: Ram's sister, Shyam's neice and Gumnaam's wife are the same person) 7. Four persons A, B, C and D are playing cards. Each person has one card, laid down on the table below him, which has two different colours on either side. The colours visible on the table are Red, Green, Red and Blue. They see the color on the reverse side and give the following comment. A: Yellow or Green B: Neither Blue nor Green C: Blue or Yellow D: Blue or Yellow Given that out of the 4 people 2 always lie find out the colours on the cards each person. 1. At 6'o a clock ticks 6 times. The time between first and last ticks is 30 seconds. How long does it tick at 12'o clock. Ans: 66 sec. (2 marks)

2. Three friends divided some bullets equally. After all of them shot 4 bullets the total number of bullets remaining is equal to the bullets each had after division. Find the original number divided. Ans: 18 (2 marks)

Initially . x x Now x-4 x-4 Equation is 3x-12 = x

x x-4

3. A ship went on a voyage. After it had travelled 180 miles a plane statrted with 10 times the speed of the ship. Find the distance when they meet from starting point. Ans: 200miles. (2 marks) Distance travelled by plane = 1/10 distance travelled by ship + 180 4. Complete the Table given below: Three football teams are there. Given below is the group table. Fill in the x's Played Won Lost Draw Goals For Goals Against 2 2 x x x B 2 x x 1 C 2 x x x 3 7 (4 2 4 1

Ans: The filled table is given below marks) Played Won Lost Draw

A 2 2 0 0

Goals For Goals Against

7 B 2 0 1 1 C 2 0 1 1 3 2

5. There are 3 societies A, B, C. A lent cars to B and C as many as they had already. After some time B gave as many tractors to A and C as many as they have. After sometime c did the same thing. At the end of this transaction each one of them had 24. Find the cars each orginally had. Ans: A had 39 cars, B had 21 cars & C had 12 cars marks) 6. There N stations on a railroad. After adding X stations on the rail route 46 additional tickets have to be printed. Find N and X. Ans. x=2 and N=11 Let initially, N(N-1) = t After adding, (N+X)(N+X-1) = t+46 By trail and error method marks) (4

(4

7. Given that April 1 is tuesday. A, B, C are 3 persons told that their farewell party was on A - May 8, thursday B - May 10,tuesday

C - June 5, friday Out of A, B, C only one made a completetly true statement concerning date,day and month The other told two one told the day right and the other the date right.. What is correct date, month, day. Ans: B - (May 10) SUNDAY C - June 6 (Friday). marks) (5

8. The Bulls, Pacers, Lakers and Jazz ran for a contest. Anup, Sujit, John made the following statements regarding results. Anup said either Bulls or Jazz will definitely win Sujit said he is confident that Bulls will not win John said he is confident that neither Jazz nor Lakers will win When the result cameit was found that only one of the above three had made a correct statement. Who has made the correct statement and who has won the contest. Ans: Sujith; Lakers (5marks )

9. Five people A ,B ,C ,D ,E are related to each other. Four of them make one true statement each as follows. (i) B is my father's brother. (ii) E is my mother-in-law. (iii)C is my son-in-law's brother (iv)A is my brother's wife. Ans: (i) D (ii) B (iii) E (iv) C

(10 marks)

10. Some statements are given below: L says all of my other four friends have money M says that P said that exactly one among them has money N says that L said that precisely two among them have money O says that M said that three of the others have money P, L and N said that they have money All the above statement are false.. Who has money & who doesn't have any money? (5 marks) 1. Mr.Mathurs jewels have been stolen from his bank locker . The bank has lockers of 12 people which are arranged in an array of 3

rows and 4 columns like: 1 2 5 6 7 3 4

8 9 10 11 12 The locker belonging to JONES was to the right of BLACK'S locker and directly above MILLAR'S. BOOTH'S locker was directly above MILLAR'S. SMITH'S locker was also above GRAY's (though not directly). GREEN'S locker was directly below SMITH'S. WILSON'S locker was between that of DAVIS and BOOTH. MILLAR'S locker was on the bottom row directly to the right of HERD'S. WHITE'S locker was on the bottom right hand corner in the same column as BOOTH'S. Which box belonged to Mr.Mathurs? Ans: Box number 9 belongs to Mr.Mathurs. 2. Fifty minutes ago if it was four times as many minutes past three o'clock,how many minutes is it to six o'clock? Ans: Twenty six minutes. 3. If a clock takes 7seconds to strike 7, how long will the same clock take to strike 10? Ans: The clock strikes for the first time at the start and takes 7 seconds for 6 intervals-thus for one interval time taken=7/6. Therefore, for 10 seconds there are 9 intervals and time taken is 9*7/6=10 and 1/2 seconds. 4. Three criminals were arrested for shop lifting. However, when interrogated only one told the truth in both his statements, while the other two each told one true statement and one lie. The statements were:

ALBERT :(a)Chander passed the merchandise. (b)Bruce created the diversion. BRUCE :(a)Albert passed the merchandise. (b)I created the diversion. CLIVE :(a)I took the goods out of the shop. (b)Bruce passed them over. Ans: Albert passed the goods.Bruce created the diversion..Clive took the goods out of the shop. 5. Everyday in his business a merchant had to weigh amounts from 1 kg to 121 kgs, to the nearest kg. What are the minimum number of weight required and how heavy should they be? Ans: .The minimum number is 5 and they should weigh 1,3,9,27 and 81 kgs. 6. A hotel has 10 storeys.Which floor is above the floor below the floor, below the floor above the floor, below the floor above the fifth. Ans: The sixth floor. 7. Seven members sat around a table for three days for a conference. The member's names were Abhishek, Amol, Ankur, Anurag,Bhuwan ,Vasu and Vikram. The meetings were chaired by Vikram. On the first evening members sat around the table alphabetically. On the following two nights, Vikram arranged the seatings so that he could have Abhishek as near to him as possible and abesent minded Vasu as far away as he could. On no evening did any person have sitting next to him a person who had previously been his neighbour. How did Vikram manage to seat everybody to the best advantage on the second and third evenings? Ans: Second evening:Vikram,Ankur,Abhishek,Amol,Vasu,Anurag and Bhuwan. Third evening :Vikram,Anurag,Abhishek,Vasu,Bhuwan,Ankur,Amol. 8. Two trains start from stations A and B spaced 50 kms apart at the same time and speed.

As the trains start, a bird flies from one train towards the other and on reaching the second train, it flies back to the first train.This is repeated till the trains collide. If the speed of the trains is 25 km/h and that of the bird is 100km/h. How much did the bird travel till the collision. Ans: 100 kms. 9. Four prisoners escape from a prison. The prisoners, Mr East, Mr West, Mr South, Mr North head towards different directions after escaping. The following information of their escape was supplied: The escape routes were The North Road, South Road, East Road and West Road. None of the prisoners took the road which was their namesake. Mr.East did not take the South Road Mr.West did not the South Road. The West Road was not taken by Mr.East What road did each of the prisoners take to make their escape? Ans: Mr.East took the North Road Mr.West took the East Road Mr.North took the South Road Mr.South took the West Road. 10. Complete the series: 5, 20, 24, 6, 2, 8, ? Ans: 12 (as 5*4=20, 20+4=24, 24/4=6, 6-4=2, 2*4=8, 8+4=12). 1. Replace each letter by a digit. Each letter must be represented by the same digit and no beginning letter of a word can be 0. ONE ONE ONE ONE ------TEN Ans: 0 =1, N = 8 ,E = 2, T = 7

2. Ann, Boobie, Cathy and Dave are at their monthly business meeting. Their occupations are author, biologist, chemist and doctor, but not necessarily in that order. Dave just told the biologist that Cathy was on her way with doughnuts.

to choose,but didn't say anything. What is each person's occupation?

Ann is sitting across from the doctor and next to the chemist. The doctor was thinking that Boobie was a goofy name for parent's

Ans: Since Dave spoke to the biologist and Ann sat next to the chemist and across the doctor, Cathy must be the author and Ann the biologist. The doctor didn't speak, but David did, so Bobbie is the doctor and Dave the chemist.

3. Sometime after 10:00 PM a murder took place. A witness claimed that the clock must have stopped at the time of the shooting. It was later found that the postion of both the hands were the same but their positions had interchanged. Tell the time of the shooting (both actual and claimed). Ans: Time of shooting = 11:54 PM Claimed Time = 10:59 PM

4. Next number in the series is 1 , 2 , 4 , 13 , 31 , 112 , ? Ans: 224. No number has digits more than 4. All of them are 1 , 2, 4, 8 , 16 , 32 , 64 converted to numbers in base 5

5. Shahrukh speaks truth only in the morning and lies in the afternoon, whereas Salman speaks truth only in the afternoon. A says that B is Shahrukh. Is it morning or afternoon and who is A Shahrukh or Salman. Ans: Afternoon ; A is Salman.

6. Two trains starting at same time, one from Bangalore to Mysore and other in opposite direction arrive at their destination 1 hr and 4 hours respectively after passing each other. How nuch faster is one train from other? Ans: Twice

7. There are 6 volumes of books on a rack kept in order ( ie vol.1, vol. 2 and so on ). Give the position after the following changes were noticed. All books have been changed Vol.5 was directly to the right of Vol.2 Vol.4 has Vol.6 to its left and both weren't at Vol.3's place Vol.1 has Vol.3 on right and Vol.5 on left An even numbered volume is at Vol.5's place Find the order in which the books are kept now. Ans: 2 , 5 , 1 , 3 , 6 , 4

8. I bought a car with a peculiar 5 digit numbered licence plate which on reversing could still be read. On reversing value is increased by 78633.Whats the original number if all digits were different? Ans: Only 0 1 6 8 and 9 can be readupside down.So on rearranging these digits we get the answer as 10968

9. The shape in the sketch below is that of a square attached to half of a similar square.Divide it into four equal pieces

Ans: Hint : the figure can be divided into 12 equal triangles

10. Supposing a clock takes 7 seconds to strike 7. How mlong will it take to strike 10? Ans: 10 1/2 seconds. 1) A man collects cigarette stubs and makes one full cigarette with every 8 stubs. If he gets 64 stubs how many full cigarettes can he smoke. Ans: 8+1=9 2) A soldier looses his way in a thick jungle. At random he walks from his

camp but mathematically in an interesting fashion. First he walks one mile East then half mile to North. Then 1/4 mile to West, then 1/8 mile to South and so on making a loop. Finally how far he is from his camp and in which direction. Ans: Distance travelled in north and south directions 1/2 - 1/8 + 1/32 - 1/128 + 1/512 - and so on = 1/2/((1-(-1/4)) Similarly in east and west directions 1- 1/4 + 1/16 - 1/64 + 1/256 - and so on = 1/(( 1- ( - 1/4)) Add both the answers 3) How can 1000000000 be written as a product of two factors neither of them containing zeros Ans: 2 power 9 x 5 power 9 4) Conversation between two mathematcians: First : I have three childern. The product of their ages is 36. If you sum their ages, it is exactly same as my neighbour's door number on my left. The second mathematician verfies the door number and says that it is not sufficient. Then the first says " Ok one more clue is that my youngest is really the youngest". Immmediately the second mathematician answers . Can you answer the question asked by the first mathematician? What are the childeren ages? Ans 1,6 and 6 5) Light glows for every 13 seconds . How many times did it glow between 1:57:58 and 3:20:47 am. Ans : 383 + 1 = 384 6) 500 men are arranged in an array of 10 rows and 50 columns according to their heights. Tallest among each row of all are asked to fall out. And the shortest among them is A. Similarly after resuming that to their original podsitions that the shortest among each column are asked to fall out. And the tallest among them is B . Now who is taller among A and B ?

Ans A 7) A person with some money spends1/3 for cloths, 1/5 of the remaining for food and 1/4 of the remaining for travel. He is left with Rs 100/- . How much did he have with him in the begining ? Ans: Rs 250/8) There are six boxes containing 5 , 7 , 14 , 16 , 18 , 29 balls of either red or blue in colour. Some boxes contain only red balls and others contain only blue. One sales man sold one box out of them and then he says " I have the same number of red balls left out as that of blue ". Which box is the one he solds out ? Ans: Total no of balls = 89 and (89-29 /2) = 60/2 = 30 and also 14 + 16 = 5 + 7 + 18 = 30 9) A chain is broken into three pieces of equal lenths containing 3 links each. It is taken to a backsmith to join into a single continuous one . How many links are to to be opened to make it ? Ans : 2. 10) Grass in lawn grows equally thick and in a uniform rate. It takes 24 days for 70 cows and 60 days for 30 cows to eat the whole of the grass. How many cows are needed to eat the grass in 96 days.? Ans : 20 g - grass at the beginning r - rate at which grass grows, per day y - rate at which one cow eats grass, per day n - no of cows to eat the grass in 96 days g + 24*r = 70 * 24 * y g + 60*r = 30 * 60 * y g + 96*r = n * 96 * y Solving, n = 20. 1. From a vessel, 1/3rd of the liquid evaporates on the first day. On the second day 3/4th of the remaining liquid evaporates. What fraction of the volume is present at the end of the second day. Ans: 50% 2. An orange glass has orange juice and white glass has apple juice both of equal volumes.

50ml of the orange juice is taken and poured into the apple juice. 50ml from the white glass is poured into the orange glass. Of the two quantities, the amount of apple juice in the orange glass and the amount of orange juice in the white glass, which one is greater and by how much? Ans: The two quantities are equal 3. There is a 4 inch cube painted on all sides. This is cut down into of 1 inch cubes. What is the no of cubes which have no pointed sides. Ans: 8 4. Sam and Mala have a conversation. Sam says I am certainly not over 40 Mala says I am 38 and you are atleast 5 years older than me Now Sam says you are atleast 39 All the statements by the two are false. How old are they really? Ans: Mala = 38 yrs Sam = 41 yrs. 5. Ram Singh goes to his office in the city, every day from his suburban house. His driver Gangaram drops him at the railway station in the morning and picks him up in the evening. Every evening Ram Singh reaches the station at 5 O' Clock. Gangaram also reaches at the same time. One day Ram Singh started early from his office and came to the station at 4 O' Clock. Not wanting to wait for the car he starts walking home. Mangaram starts at normal time, picks him up on the way and takes him back house, half an hour early. How much time did Ram Singh walk? 6. In a railway station, there are two trains going. One in the harbour line and one in the main line, each having a frequency of 10 minutes. The main line service starts at 5 o'clock and the harbour line starts at 5.02A.M. A man goes to the station every day to catch the first train that comes. What is the probability of the man catching the first train? Ans: 0.8

7. A family X went for a vacation. Unfortunately it rained for 13 days when they were there. But whenever it rained in the mornings, they had clear afternoons and vice versa. In all they enjoyed 11 mornings and 12 afternoons. How many days did they stay there totally? Ans: 18 8. A survey was taken among 100 people to find their preference of watching T.V. programmes. There are 3 channels. Given the no of people who watch at least channel 1 at least channel 2 at least channel 3 no channels at all atleast channels 1and 3 atleast channels 1 and 2 atleast channels 2 and 3 Find the no of people who watched all three. 9. Albert and Fernandes have two leg swimming race. Both start from opposite ends of the pool. On the first leg, the boys pass each other at 18 m from the deep end of the pool. During the second leg they pass at 10 m from the shallow end of the pool. Both go at constant speed but one of them is faster. Each boy rests for 4 seconds at the end of the first leg. What is the length of the pool? 10. Each alphabet stands for one digit in the following multiplication. THIS x IS --------XFXX XXUX -----------XXNXX -----------What is the maximum value T can take? Ans: T max value = 4 1.An escalator is descending at constant speed. A walks down and takes 50 steps to reach the bottom. B runs down and takes 90 steps in the same time as A takes 10 steps. How many steps are visible when the escalator is not operating? Ans: 150 steps

2. Every day a cyclist meets a train at a particular crossing. The road is straight before the crossing and both are travelling in the same direction. The cyclist travels with a speed of 10 Kmph. One day the cyclist comes late by 25 min. and meets the train 5km before the crossing. What is the speed of the train? Ans: 60 kmph 3. There are five persons with surnames Mukherjee, Misra, Iyer, Patil and Sharma. There are 4 persons having first or middle name of Kumar, 3 persons with Mohan, 2 persons with Dev and 1 Anil. Either Mukherjee and Patil have a first or middle name of Dev or Misra and Iyer have their first or middle name of Dev. Of Mukherkjee and Misra, either both of them have a first or middle name of Mohan or neither have a first or middle name of Mohan. Either Iyer or Sharma has a first or middle name of Kumar but not both. Who has the first or middle name of Anil? Ans: Kumar Misra Dev Mohan Iyer Dev Kumar Patil Mohan Mohan Sharma Kumar 4. Boys are allowed to watch football at C.V.Raman auditorium subjected to conditions. The boy over age 16 can wear overcoat No boy over age 15 can wear cap To watch the football either he has to wear overcoat or cap or both A boy with an umberella or above 16 or both cannot wear sweater. Boys must either not watch football or wear sweater. What is the appearence of the boy who is watching football. 5. A bird keeper has got P pigeons, M mynas and S sparrows. The keeper goes for lunch leaving his assistant to watch the birds. Suppose p=10, m=5, s=8 when the bird keeper comes back, the assistant informs the x birds have escaped. The bird keeper exclaims: "Oh no! All my sparrows are gone." How many birds flew away? When the bird keeper comes back, the assistant told him that x birds

have escaped. The keeper realised that atleast 2 sparrows have escaped. What is minimum no of birds that can escape? 6. Answer the following questions based on the conditions from the choices A, B, C, D, E as described below: (A) if a definete conclusion can be drawn from condition 1 (B) if a definete conclusion can be drawn from condition 2 (C) if a definete conclusion can be drawn from condition 1 and 2 (D) if a definete conclusion can be drawn from condition 1 or 2 (E) no conclusion can be drawn using both conditions person 1 says N<5 person says N>5 person 3 says 3N>20 person 4 says 3N>10 person 5 says N<8 What is the value of N? a) 1. No of persons who speak false being less than no of persons who tells the truth. 2. Person 2 is telling the truth. b) 1. no of persong telling the truth is greater than no of persons telling lies 2. person 5 is telling the truth. 7. There are N coins on a table and there are two players A & B. You can take 1 or 2 coins at a time. The person who takes the last coin is the loser. A always starts first. If N=7 (a) A can always win by taking two coins in his first chanse (b) B can win only if A takes two coins in his first chance. (c) B can always win by proper play (d) none of the above 2. A can win by proper play if N is equal to (a) 13 (b) 37 (c) 22 (d) 34 (e) 48 Ans: (e.) 3. B can win by proper play if N is equal to (a) 25 (b)26 (c) 32 (d) 41 (e) none 4. if N<4, can A win by proper play always? (a) Yes (b) No 8. Two twins have vertain peculiar charcteristics. One of them always lies on Monday, Wednesday, Friday. The other always lies on Tuesdays, Thursdays and Saturdays. On the other days they tell the truth. You are given a conversation.

Person A-- today is Sunday my name is Anil Person B -- today is Tuesday, my name is Bill What day is today? Ans: Today is Tuesday. 9. There is a safe with a 5 digit number as the key. The 4th digit is 4 greater than second digit, while 3rd digit is 3 less than 2nd digit. The 1st digit is thrice the last digit. There are 3 pairs whose sum is 11. Find the number. Ans: 65292 10. A hotel has two wings,the east wing and the west wing. Some east wing rooms but not all have an ocean view. All west wing rooms have a harbour view. The charge for all rooms is identical, except as follows : Extra charge for all harbour view rooms on or above the 3rd floor Extra charge for all ocean view rooms except those without balcony Extra charge for some harbour rooms on the first two floor & some east wing rooms without ocean view but having kitchen facilities. Which of the following cannot be determined on the basis of the nformation given: I. Whether there are any rooms without a balcony for which an extra charge is imposed. II. Whether any room without a kitchen or a view involves an extra charge. III. Whether two extra charges are impsed for any room. (A) I only (B) II only (C) III only (D) II and III only (E) I, II and III (This question is from 1999 Barrons GRE Guide model Test 3 - section 6, question 22) Ans: (A)1. If a date oct,02,2001 is written as 10022001(mmddyy), Then which date before 01,oct,2000 gives the maximum valued palindrome(the digits must be same when reversed) ? 6 marks 2 negative Ans:12311321

2. There are eight differen coloured eggs in a basket, among them 23 are in red, 17 blue, 21 green, 18 yellow, 16 black, 12 white, 25 voilet.What is the probability of getting atleast one pair of the same colour, when you draw a least number? 7 marks 2 negative Ans:................... 3. What is the number whose positive integer digits when multiplied gives the maximum value and added gives the exact 100. 5 marks 2 negative Ans:(2+2+........+50 times)=100, i.e 2 power 50. 4. In my garden the gross grows equally in every year. It takes 24 days for the 70 cows to eat and it takes 60 days for the 30 cows to eat. Then how many number of cows can eat in 96 days? 6 marks 2 negative Ans:17.5 so considered as 18 5. In the investigation regarding one theft among the suspects A,B,C one who is the thief always tells lies, the helper to the thief tells both truth and lies , the other fellow who is innocent always tell the truth. They made the following statements A: I'am a doctor B is a barber and C is the taxi driver. B: I'am an employee of water works dept, A is the taxi driver and C is the doctor. C: I'am a lawyer A is a taxi driver and B is the doctor. Who is the thief? 7 marks 2 negative Ans:....................... 6. In one country they prefer only boys to have. If it is a girl in the first birth then there must be the other child to have, if it is also a girl then they will go for the next birth and stops when they got the boy. Then what is the praportion of girls to boys in that country? 8 marks 2 negative Ans: (n+1)/2:1 7. While three friends Abe, Bob, Cort in a restaurent taking thier snax one lady came and she asked them how many children you have. One of them replied "By looking at the total number of children we have you can uniquely identify the no of children I'am having" but she failed to guess then he made the following statements Abe is having atleast one girl and twice as many boys as girls. Bob is having atleast one girl and thice as many boys as girls. Cort is having three more than boys that he have girls. The no of children they have is less than 25.

Who has replied to that lady and how many children he have. How many total no of children they have? 11 marks 2 negative Ans: Abe, and is having 3children (1G+2B),total is 12 CTS_BLACK vocabulary,strings,dominoes,functions,coding (each section 8 ques) CTS_BROWN word series,numerical series,functions,figures,verbal (each section 8 ques) CTS_VIOLET functions,strings,bricks,jigsaw puzzle,cryptic clues (each section 8 ques) CTS_RED 1. 8 functions 2. 4 cryptic clues ,4 anagrams 3. 4 tetris figures, 4 bricks 4. 8 strings 5. 4 jigsaw puzzles 4 number series Instructions 1. All answers provided in these sets may not be correct. So Please check. 2. do read old papers. few ques of 2002 came from these old sets INTERVIEW * puzzles * technical BROWN 2002 There were different papers for different sessions. The paper had 5 sections, 5 * 8 = 40 Q's. totally. Section 1 : Functions. Q: 1 - 8 Certain functions were given & based upon the rules & the choices had to be made based on recursion. This is time consuming, but u can do it. Try to do it at the end. start from the last section. L(x) is a function defined. functions can be defined as L(x)=(a,b,ab) or (a,b,(a,b),(a,(b,b)),a,(b,b)).... two functions were given A(x) & B(x) like if l(x)=(a,b,c) then A(x)=(a) & B(x)=(b,c) i.e., A(x) contains the first element of the function only. & B(x) contains the remaining, except the first element.

then the other two functions were defined as C(x) = * if L(x) = () A(x) if L(x) = () & B(x) != () C(B(x)) otherwise D(x) = * if L(x) = () ** if B(x) = () A(x) if L(x) != () & B(x) != () D(D(x)) otherwise now the Questions are, 1 : if L(x) = (a,b,(a,b)) then C(x) is ? (a): a (b): b (c): c (d): none 2 : if L(x) = (a,b,(a,b)) then find D(x) same options as above 3 : if L(x) = (a,b,(a,b),(b,(b))) find C(x) 4 : -----------~~~~~~~~---------- find D(x) 5 : if L(x) = (a,(a,b),(a,b,(a,(b))),b) then find c(x) 6 : -----------~~~~~~~~---------- find D(x) 7 : if L(x) = (a,b,(a,b)) then find C(D(x)) 8 : -----------~~~~~~~~---------- find D(C(x)) Section 2 : Word series Q's : 9 - 16 This is one of the easiest section. Try to do it at first. if S is a string then p,q,r form the substrings of S. for eg, if S=aaababc & p=aa q=ab r=bc then on applying p->q on S is that ababaabc only the first occurance of S has to be substituted. if there is no substring of p,q,r on s then it should not be substituted. If S=aabbcc, R=ab, Q=bc. Now we define an operator R? Q when operated on S, R is replaced by Q, provided Q is a subset of S, otherwise R will be unchanged. Given a set S= ., when R? Q, P&#61= 672; R, Q ? P operated successively on S, what will be new S? There will be 4 = : if s=aaababc & p= aa q=ab r=bc then applying p->q, q->r & r->p will give, (a): aaababc (b): abaabbc (c): abcbaac (d): none of the a,b,c 10: if s=aaababc & p= aa q=ab r=bc then applying q->r & r->p will give, 11: if s=abababc & p= aa q=ab r=bc then applying p->q, q->r & r->p will give, 12: if s=abababc & p= aa q=ab r=bc then applying q->r & r->p will give, 13: if s=aabc & p=aa q=ab r=ac then applying p->q(2) q->r(2) r->p will give, (2) means applying the same thing twice. 14: similiar type of prob. 15: if s=abbabc p=ab q=bb r=bc then to get s=abbabc which one should be applied. (a): p->q,q->r,r->p

16: if s=abbabc p=ab q=bb r=bc then to get s=bbbcbabc which one should be applied. Let us consider a set of strings such as S=aabcab. We now consider two more sets P and Q which also contain strings. An operation P->Q is defined in such a manner that if P is a subset of S, then P is to be replaced by Q. In the following questions, you are given various sets of strings on which you have to perform certain operations as defined above. Choose the correct alternative as your answer. (the below are some ques from old ques papers) 21. Let S=abcabc, P=bc, Q=bb and R=ba. Then P->Q, Q->R, R>P changes S to (A) ............ (B) abcabc (C) ............ (D) none of A,B,C 22. Let S=aabbcc, P=ab, Q=bc and R=cc. Then P->Q, Q->R, R>P changes S to (A) ababab (B) ............ (C) ............ (D) none of A,B,C 23. Let S=bcacbc, P=ac, Q=ca and R=ba. Then P->Q, Q->R, P>R changes S to (A) ............ (B) ............ (C) bcbabc (D) none of A,B,C 24. Let S=caabcb, P=aa, Q=ca and R=bcb. Then P->Q, P->R, R>Q changes S to (A) ............ (B) ............ (C) ............ (D) none of A,B,C

Section 3 : numerical series Q's : 17 - 24 This is little bit tough. proper guesses should be made. find these probs in r.s.aggarval's verbal & non verbal reasoning. 17: 2,20,80,100, ?? (a): 121, (b): 116 (c): 18: 10,16,2146,2218, ?? (d):none

like these other series were given. section 3 : series (from other booklet) transformations 17: 1 1 0 2 2 1 1 ---> 0 0 1 0 0 2 2 1 0 1 1 0 0 1 ---> 2 1 2 2 1 1 2 then 2 2 1 1 0 1 1 ---> ???? ans may be 0 0 2 2 1 2 2

18: 1 1 0 0 2 2 ---> 2 2 0 0 1 1 1 0 1 1 2 1 ---> 1 2 1 1 0 1

Section 4 : figures 19: ^ ^ | -> <- | -> | ^ : ^ :: ^ : ? | -> <- | <- | ans is : ^ | <^ | -> all probems are very easy.(see cts_old\cts13 file) some are mirror images, some r rotated clockwise/anti Section 5 : Verbal if u have a very good vocab. then this section is managable. two words together forming a compound words were given. the q's contained the second part of the compound word. the first word of the compuond word had to be guessed. then its meaning had to be matched with the choices. if the word is "body" then its meaning of its first part is.. its really tough to guess.. the words were however very simple some words which i can remember are, head, god, (see old papers) like block head main stream star dust Eg: OLD PAPERS (1) -(head)- (a) purpose (b) man (c)obstacle (d)(ans:c for blockhead) >(2) (dust)- (a) container(b)celestial body (c)groom(d)(ans: c for star dust) >(3) (stream )-(a) mountain (b) straight (c) (d) (ans:a) >(4) (crash)- (a) course (b) stock3 anagram >first find the anagram of the given word & then >choose the meaning of the anagram from the options. >1. latter ->rattle 2..spread 3.risque 4.dangled(ansjogged) ^

Quest of red set. i) Series Transformation 1) If 102101->210212 then 112112->? a) b) c) d) 2) if 102101-> 200111 then 112112->? Again there r 4 choices. 3) If 102101->101201 then 112112->? Again there r 4 choices. Tips:The 1st one all change 0->1, 1->2, 2->1 The 2nd on alternate do not change The 3rd it is just reverse of the original string _______________________________________________________ ii) Target=127: Brick=24,17,13: Operation available= +,/,*,Again there r 4 choices.For ex choice b)20,6,7 Tips:Answer is b one bcos 20*6+7=127.Hence it is the answer Q:1)U HAVE TO MAKE A TARGET =102; THE ANSWER FROM THE OPTION IS (6,17,2,1) 2)TARGET=41;FIVE NO.S WERE GIVEN;25 22 16 5 1 U CAN USE THE NO.S ONLY ONCE&CAN PERFORM OPERATION +,MULTIPLY,-,/,()ONCE; OPTIONS WERE; A)25 22 16 5 B)25 22 16 1 C)25 22 5 1 D)25 16 5 1) 4 SUCH QUESTINS ARE THERE. 2)87 3)146 4)127 THERE ARE SOME FIGURATIVE QUESTION;SEE FROM COMPETITION MASTER,I CANT REMEBER THE FIGURE.4 QUESTIONS ARE THERE __________________________________________________ iii) Cryptic Sentence. Form word A sentence is there .a cryptical clue is hidden in the sentence. Find out answer from the opticn. 1)a friend in rome a)aerodrome b)palindine c)palindrome d)condome ans:palindrome 2)Rowed them across a)crosswiz b)acropolis c)acroword d)crossword Ans:crossword/crossover 3)cuticle cutting the filly glass a)cubicle b)uphilly c)cutglass d)cutlass Ans:cutlass

4)hat jumps upward in a water closet a)watch b)witch ans:watch/whatever Tips:The 1st oneJumble out the word SHORE to get the word HORSE and then get the adjective of the word HORSE as TROJAN The 2nd one lips->slip->freudian/french _______________________________________________________________ iv) Anagram noun form the correesponding adjectives There re options. Q:some nouns are jumbled on ,you have to rearrange, look for a suitable adjective: Make a phrase then. 1)shore a)aegean b)Indian c)trojan d)Spartan ans:trojan 2)sire a)dutch b)rome c)herculean d)mercurial ans:mercurial 3)ourcage a)english b)rome c)dutch d)Spartan ans:Spartan 4)lips Again there r 4 choices. Ans:freudian/french _______________________________________________________________ v) Jigsaw puzzle as given in the book by Edgar Thorpe, of TMH Publications _____________________________________________________________________ vi) FUNCTIONS same as CTS_BLACK\fun ____________________________________________________________________ vii) x , y -> strings of G st there is at least one G in x and y xoxy valid xoy->xoxy invalid Find valid & invalid strings

____________________________________________________________________ viii)there were a couple of ( seven to be precise)figures ( tetris type if u remember that game) given in the main theme. The 10 questions that followed showed patterns which were formed due to combination of the 7basic figs. NOTE: the intersecting part of the combined fig. always gets subtracted from the total combination Hello Shivesh CTS paper was of diff pattern this time and there were ateast 5 different sets of question papers given to students. Of the type i recvd, as i told there wer 10x4 questions for 60 mins. section: 4) last section( thats bcoz i remeber it well) had meaningful words whose anagrams are nouns and we hav to choose the best adjective from the list to describe this noun: ex: shore ( word given) choices: a) roman b) spanish c) trojan d).... ans: c) trojan shore is anagram(jumbled form of) 'horse' and trojan-horse is the best match 3) there were a couple of ( seven to be precise)figures ( tetris type if u remember that game) given in the main theme. The 10 questions that followed showed patterns which were formed due to combination of the 7basic figs. NOTE: the intersecting part of the combined fig. always gets subtracted from the total combination 2) This section had the funda of xOy where x and y represented strings of Gs . The test was to find the valid or invalid patterns with ref. to the rules 1) L=list of objects ex:L={a,b,c,d} where a,b,c,d are objects P(L) was a function( dont remembr xatly) M(L) was another function defined etc in the following questions P(x) etc were given to be found out. Note : this may take considerable amnt of time. so take intelligent guesses13) A can do a work three times better as that of B.If B can finish a work in 10 days how long A will take to finish the work. 14) Two Years ago Puspha is twice as old as Rita.If the difference of their current age is 2,What is Puspa 's age now?. 15)A/B= 5/7 B/C = 6/11 what is A:B:C ? 16)A/B= 3/4 ;The Sum of A,B is 420. what is the larger value of these two.

Ans 240 (B). 17)A train 300m long crosses a pole 600m long in 5 minutes.What is its Velocity. Similar Question. 18) A train of 120m crosses a pole of 130m in 54 km/Hr.I forget the question But.. 19)Find the odd one in the series. 385,462,572,396,427,671. (I wrote these no inhand to tell it to you.) 20)When will the needles of the Clock co incide between 5 O clock and 5.30 ?. 5.22.A, B, C are three persons. among these 3 persons only one is having a key to open a treasure. Each person always either speaks truth or lies. The three persons made the following statements. A: I have the key B: I dont have the key C: B dont have the key. Among the three, atleast one always speak truth. Find out who got the key? (4 Marks) -------------------------------------------------------------------------------A train is travelling from A to B. During its travel it met with an accident after travelling for half an hour. After the accident it travels with 3/5th of its original speed and arrives the destination two hours late. If accident occurs 50 km farther it will arrive the destination by 40 mins late. Then find the distance between A and B? (6 Marks) -------------------------------------------------------------------------------A starts from x to y at noon. his friend B, starts from y to x at 2 pm and met him at 4:05 pm and then both of them reached their destinations at the same time. Find arrival time. (8 Marks) -------------------------------------------------------------------------------There are innumerable packets of neggets each of which contain either 3 or 6 or 9 nuggets. You cannot break a packet ( i.e., u can not have 1 or 2 nuggets, they are available only in packets of 3, 6 and 9). Problem is to find a number N starting from which it is possible to get any number of nuggets without breaking any of the packets.

(12 Marks) Ans : There is no such number exists. It's not possible to find out. So, answer should be "Infinite". -------------------------------------------------------------------------------An item x is available in 6,9,20 peices of packets. A customer purchased least N peices such that N+1, N+2, N+3.......peices also can be purchased. Find N. (12 Marks) Ans : 44 (This answer is absolutely right. Find out a method how to solve it. If you have a method which works for all such kinds of problems, mail to me at : http://us.f109.mail.yahoo.com/ym/Compose? To=nchetana123@yahoo.com) -------------------------------------------------------------------------------Three ants are present on the 3 vertices of triangle.They need to change their positions. what is probability of getting their positions changed with the condition that no two ants collide each other while exchanging their positions. -------------------------------------------------------------------------------You need to chose "n" numbers such that their sum is 100 and they give the maximum product when multiplied. -------------------------------------------------------------------------------A business guy got 4 weigths and he can weight up to 40 kgs. What are denominations ? Ans : 1,3,9,27 -------------------------------------------------------------------------------In a game where three dice are thrown, a player guesses a number. If the guessed number appears on only one die, he gets double amount. If the guessed number appears on two dies, he gets thrice the amount. If the guessed number appears on three dies, he quadraples his money. Find the probability to win triple or quadraple amount of the bet. (8 Marks) -------------------------------------------------------------------------------Tom, Dolly and Mouly are living in a street having door numbers 1-99. But they dont know each others house numbers. Mouly asked Tom, "is your dno a perfect square?". He replied. Again she asked, "is it greater than 50?". He replied.

With this information she went to Tom house, but she could not find out his address. Then she realizes that the second answer is only the correct one. Then dolly, who is unaware of the above conversation, asked Tom, "Is ur dno a perfect cube?". He replied. Again she asked, "is it greater than 25?". He replied. With this information she went to Tom house, but she could not find out his address. Then she realizes that the second answer is only the correct one. If sum of door numbers of the three is equal to a perfect square multiplied with 2. Tom's door number is less than that of Dolly. Also Tom's door number is less than that of Mouly. Then find out the door numbers of all the three people. (12 Marks) --------------------------------------------------------------------------------

Question Paper of the Written Test held on : 2 nd March 2003 at Bangalore Note: The questions may not be complete. They are just to give you an idea of how the test will be. -------------------------------------------------------------------------------How to weigh 1 to 40kg (whole numbers) with only 4 different weighing stones on a simple balance. What are denominations ? Ans : 1,3,9,27 -------------------------------------------------------------------------------A train travels from A to B .on the way from A to B,after traveling for 1 hr the train has an accident and the rest of the journey it travels with 3/5 of its original speed and reaches 2 hrs late. If the accident has occurred 50 km further it would have reached 40 minutes earlier. What is the distance between A and B ? -------------------------------------------------------------------------------A murder occurred in atrain.The murdered is Mr X. The people investigated gave the following statements. Old Man : Im innocent . I saw Blonde woman talking to Mr X Blonde Woman : Im innocent . I didnt talk with Mr X. Young Man : Im innocent . The brunette woman killed Mr X. Brunette Woman : Im innocent . One of the men is the murderer. There are 4 true and four false statements .

Who is the murderer? -------------------------------------------------------------------------------Ajay and Vijay divided some apples between themselves. Vijay says : You have more apples than me . Then Ajay gives Vijay one apple each for Vijay's age. Then Vijay says : You have 2 times the apples that I have. And Ajay replies that : it is fair since my age is twice your age. When Ajay leaves, Vijay takes as many apples equal to Ajay's age. Who has more number of apples ? -------------------------------------------------------------------------------3 dice each with a number from 1 to 6 are rolled one after another. A person can bet mony on any one of the numbers If the number turns out on only die then he receives his amount along with the same amount as he invested. If the number turns out on any 2 dice then he gets his money back along with twice the money he had bet. If the number turns out on all 3 dice then he gets his money back along with trice the same money. What is the probability of tripling and quadrupling his money ? -------------------------------------------------------------------------------X puts some black and white balls in a bag. Y picks a ball from the bag and finds it to be black. He picks out another ball and finds it also to be black. He asks what would be probability of a third black ball for which X replies that it is 9/10 of the probability of the original trail. Y knows that there are atleast 7 balls in the bag originally. How many balls of each color are there originally ? >A person travels in a car with uniform speed. He observes the milestone, >>which has 2 digits. After one hour he observes another milestone with same >>digits reversed. After another hour he observes another milestone with >>same 2 digits containing a 0. Find the speed of the car? >> Ans : 45 km/h >>One quarter of the time till now from midnight and half of the time >>remaining from now up to midnight adds to the present time. What is the >>present time? Ans: 9:36AM >> After A man was on his way to a marriage in a car with a constant speed.

>>2 hours one of the tier is punctured and it took 10 minutes to replace it. >>After that they traveled with a speed of 30 miles/hr and reached the >>marriage 30 minutes late to the scheduled time. The driver told that they >>would be late by 15 minutes only if the 10 minutes was not waste. Find the >>distance between the two towns? Ans:speed =40 miles/hr distance= 120 miles

>>Three persons A, B &C went for a robbery in different directions and they >>theft one horse, one mule and one camel. They were caught by the police >>and when interrogated gave the following statements A: B has stolen the horse B: I didnt rob anything. C: both A & B are false and B has stolen the mule. The person who has stolen the horse always tell the truth and The person who has stolen the camel always tell the lie. Find who has stolen which animal? Ans: A- camel B- mule C- horse

>>One quarter of the time till now from midnight and half of the time >>remaining from now up to midnight adds to the present time. What is the >>present time? Ans: 9:36AM >>After world war II three departments did as follows First department gave some tanks to 2nd &3rd departments equal to the number they are having. Then 2nd department gave some tanks to 1st &3rd departments equal to the number they are having. Then 3rd department gave some tanks to 2nd &1st departments equal to the number they are having. Then each department has 24 tanks. Find the initial number of tanks of each department? Ans ; A-39 B-21 C-12 >>A, B, C, D&E are having their birthdays on consecutive days of the week >>not necessarily in the same order. A s birthday comes before Gs as many

>>days as Bs birthday comes after Es. D is older than E by 2 days. This >>time Gs birthday came on Wednesday. Then find the day of each of their >>birthdays? Ans: D SUNDAY A MONDAY E TUESDAY G WEDNESDAY B THURSDAY >>A girl A told to her friend about the size and color of a snake she has >>seen in the beach. It is one of the colors brown/black/green and one of >>the sizes 35/45/55. If it were not green or if it were not of length 35 it is 55. If it were not black or if it were not of length 45 it is 55. If it were not black or if it were not of length 35 it is 55. a) What is the color of the snake? b) What is the length of the snake? Ans: a) brown b) 55 if a clock runs 5 minutes slowly.after how many days does it show the correc time a.5 days b.6 days c.7 days d.8.5 ays 2.venn diagram (12 study french,10 study english,9 study german,7 study both eng & fren,etc...) 3.Work based problem (like 5 women can finish a work in 36 days.after some days a women is replaced by a man and a boy. a man is twice faster than women and slow than boy .find out how many days does it take to complete the work) 1) A man is going to a wedding party. He travels for 2hrs when he gets a puncture. Changing tyres takes 10mins. The rest of the journey he travels at 30 miles/hr. He reaches 30mins behind schedule. He thinks to himself that if the puncture had occurred 30miles later, he would have been only 15mins late. Find the total distance traveled by the man 2) A car is traveling at a uniform speed. The driver sees a milestone showing a 2-digit number. After traveling for an hour the driver sees another milestone with the same digits in reverse order. After another hour the driver sees another milestone containing the same two digits separated by a zero. What is the average speed of the driver? (ans : 45 kmph) 3) There are 3 societies A, B, C. A lent cars to B and C as many as they had already. After some time B gave as many tractors to A and C as many as they have. After sometime c did the same thing. At the end of this transaction each one of them had 24. Find the cars each orginally had.

Ans: A had 39 cars, B had 21 cars & C had 12 cars 4) Alpha, Beta Gamma, Delta and Epilson had birthhdays consecutively. Delta was older than Epilson by 2 days. The difference in the number of days between Alpha and Gamma was equal to Beta and Delta. (Alpha was older than Gamma). Gamma's birthday was on a Wednesday. Find on which day the others have their birthdate. 5) Adding 1/4 of the time from midnight to the present time, to 1/2 of the time from present until midnight, gives the present time. what is the present time. (ans 9.36 plz check) 6) Passage followed by 4 questions (passage on some men being lean, tall, muscular, fair, rich, employed) 7) another complex question 8) Find length and colour of serpent if not green or 35m long, then it is 55m long if not black or 45m long, then it is brown if not black or 35m long, then it is 55m long 9) Three man A,B,C went in three direction and had stolen a mule , a horse and a camel they ware caught by C.B.I and arrested . During their interrogation they gave the following statements. A: B had stolen a horse B: A and C are both lying and I had stolen nothing. C: A is lying and B had stolen a mule One who had stolen a camel is telling lie and one who had stolen a horse is telling truth. Among A,B,C who had stolen which animal ? (ans : A-Camel, B-Mule, C-Horse) . A car is traveling at a uniform speed. The driver sees a milestone showing a 2-digit number. After traveling for an hour the driver sees another milestone with the same digits in reverse order. After another hour the driver sees another milestone containing the same two digits with a zero in between(0). What is the average speed of the driver. Ans: 45 kmph 2. Mr. ANYMAN left ANYTOWN by car to attend a wedding at ANYCITY. He had been driving for exactly two hours when the car got punctured. It took his driver exactly ten minutes to change the wheel. In order to play safe they covered the remaining distance at a speed of 30 mph. consequently, Mr. ANYMAN was at wedding half anhour behind schedule. Had the car got the puncture only 30 miles later , I would have been only FIFTEEN minutes late he told the driver . How Far is ANYCITY from ANYTOWN. Ans: 120 miles 3. There are three persons A,B,C. one day they set out in different directions and each one steals an animal. The animals are camel, horse and a mule though not in

the order. A CBI officer catches them and on interrogation he has the following statements: A says: B has stolen a horse C says: A is telling wrong, B has stolen a mule. B says: A and C are both telling wrong. I have stolen Nothing. The person stealing a camel tells wrong while the person with a horse tells right. So tell which person steal which animal. Ans: A: Camel B: Mule c: Horse 4 There are 3 societies A, B, C. A Lent Tractors to B and C as many as they had already. After some time B gave as many tractors to A and C as many as they have. After sometime C did the same thing. At the end of this transaction each one of them had 24. Find the cars each originally had. This question was bit differently asked with the same thing. Ans: A had 39 cars, B had 21 cars & C had 12 cars 5: Alpha, Beta , gamma, delta and epsilon are friends and have birthdays on consecutive days though may not be in order. Gamma is as many days old to Alpha as Beta is younger to Epsilon. Delta is two days older then Epsilon. Gammas Birthday is on Wednesday. Tell whose birthday is when. Ans: Alpha: Friday Beta: Saturday Gamma: Wednesday Delta: Tuesday Epsilon: Thursday 6. The quarter of the time from midnight to present time added to the half of the time from the present to midnight gives the present time. What is the present time? Ans: 9hrs past 36 minutes AM 7. A girl was on a vacation when she happened to notice a snake. The snake was either black, brown or yellow in colour and 35, 45 or 55 m long. She makes the following remarks: Dont remember exactly but the answer I wrote was 55 m and brown colour snake. The remaining two questions were logical and lengthy . I could not get time. Anyways one was relating to six persons are from six places and professions are given . u have to match each person along with the profession and his place. And one question was relating to various characteristics such as a tall men, lean men, muscular men, rich man, employed , fair skinned and not fair skin. The relation between the various characteristics are given and based on that u are to find answers to a subset of question such as A: if the unfair men is muscular whether he is employed or not. Like this 4 questions are their. And options are given. U are to tick the answer which can be derived

from above relationships. 1. are there Three ststions . 1st one lends 2nd and 3rd that many no. of tractors that then already each had. After few months, 2nd lends to 1st and 3rd that many tractors then they had. After a few months 3rd lends to 1st and 2nd that many tractors then they had. Now each of them got 24. Find how many they had initially? - 4 2. the time now is 1/4 the time from midnight and 1/2 the time from now to midnight what is the time now? - 4 3. there are 6 men a,b,c,d,e,f from leningard, kiev, koshrv and some other 6 towns, and were in differnt proffessions some 5 conditions were given you had to match the men with their proffessions and towns .... - 8 marks 4. Question on Venn diagram. All handsome are also fair skinned Some muscular are fair skinned Some muscular are also handsome All lean are also muscular Some lean are also fair skinned. All rich man is not fair skinned but all rich man are handsome 4 qustions follow......... - 8 marks 5. alpha,beta,delta,epsilon and gamma are 5 guys problem, it is in the question bank provided by arun kumar - 6 marks 6. a girl saw a serpant and comments on the color(blue,green or black) and its length (35,45,55m), a. if its not black or if its not 35 then it is 55m b. c. i dont exactly remember the choices, one of them is always correct and other is false. find the color and length - 6 marks 7. punchered tyre question. A guy is going on to a city at a constsnt speed for 2 hrs, his tyre gets punchered aftered it takes him 10mins to change the tyre, he then goes at a constant speed @30mph for rest of the distance, on reaching the destination he finds himself 30mins late and comments " if the tyre hat punchered 30miles later(i think) i would have been only 15mins late " find the distance between the two cities. - 6marks 8. a man will be going with a certain speed, he sees a mile stone after sometime he sees another milestone that has the same digits as the forst one but in reverse order, after somemore time he sees another mile stone that contains the same numbers(either forwards/backwards) separated by a zero. find the speed. 6 - marks. The highest Score in an innings was 3/11 of the total and the next highest was 3/11 of the reminder. If the scores differed by 9, find the total score. a) 151 b) 161 c) 121 a d) 101

e) 137

Ans: c A boy was asked to multiply a certain number by 53. He multiplied it by 35 and got his answer less than the correct one by 1206. Find the number to be multiplied. a) 37 b) 67 a c) 87 d) 97 e) 107 Ans: 67 A Problem like this not exactly the same but on same model. If the manufacturer gains 10 %, the wholesale dealer 15 % and the retailer 25 % then the cost of Production of a table, if the retail price is Rs.1265 a) 632.50 b) 800 c) 814 d) 834.24 Ans: b A trader marks his goods up by 50% and declares two successive discounts of 20% each. What is his overall gain? a) 10% gain b) 4% gain c) 4% loss (A) d) 10% loss e) No loss No gain Train Problems - 3 as finding speed of train, how many secs it will cross each other etc. Three friends, one Blind, One Deaf and One dumb stays together. Only Blind is married. One day The Dumb person sees Blinds wife with Deaf in not a comprmising situation. This he want to convay to Blind without taking any external aid. How can he convay this to Blind.Q:1/3th of the passengers gotdown at first stop and 30 people got into the bus, 1/4th of the passengers gotdown at second stop and 12 people got into the bus. in last stop 84 people got down from the bus. What is the initial capacity of the bus? Q:one person is running around a 180 sq yard filed, first 1/4 his speed was 10KMPH second 1/4 20KMPH third 1/4 30 KMPH and the fourth 1/4 40 KMPH. what is the average speed? NOTE: there was a question on milk ,water ratio.. logical deductions.. Q: is x+y a prime? a. x is prime b. y is prime options: 1. only "a" correct 2. only "b" correct 3. both are correct 4. both are wrong. C,C++,JAVA Q: There were 2 or 3 C programs and the question is to point error in the program or to predict the answer Q:Which one of the following is not a storage type: 1. static 2. register 3. stack 4.auto answer: stack Q:what is the type of the second parameter of fopen() function? Q:Which of the following is correct about constructor?

OPTIONS: 1.void type 2. return nothing 3..... 4..... answer 2 Q:how do we store a constant value in java? multiple choices given.. answer: using "final" SQL: please be thorough with SQL, PL/SQL There were some questions where they gave query and asked whether it is correct or wrong if wrong what is wrong... Q: create table...........; {table creation query) Roll back; what will be the output? options .... *_ARITHMETIC SECTION This section consists of 29 problems. The questions are simple though time consuming.

1. If a boat is moving in upstream with velocity of 14 km/hr and goes downstream with a velocity of 40 km/hr, then what is the speed of the stream ? (a) 13 km/hr (b) 26 km/hr (c) 34 km/hr (d) none of these Ans. A 2. Find the value of ( 0.75 * 0.75 * 0.75 - 0.001 ) / ( 0.75 * 0.75 0.075 + 0.01) (a) 0.845 (b) 1.908 (c) 2.312 (d) 0.001 Ans. A 3. A can have a piece of work done in 8 days, B can work three times faster than the A, C can work five times faster than A. How many days will they take to do the work together ? (a) 3 days (b) 8/9 days (c) 4 days

(d) can't say Ans. B 4. A car travels a certain distance taking 7 hrs in forward journey, during the return journey increased speed 12km/hr takes the times 5 hrs.What is the distance travelled (a) 210 kms (b) 30 kms (c) 20 kms (c) none of these Ans. B 5. Instead of multiplying a number by 7, the number is divided by 7. What is the percentage of error obtained ? 6. Find (7x + 4y ) / (x-2y) if x/2y = 3/2 ? (a) 6 (b) 8 (c) 7 (d) data insufficient Ans. C 7. A man buys 12 lts of liquid which contains 20% of the liquid and the rest is water. He then mixes it with 10 lts of another mixture with 30% of liquid.What is the % of water in the new mixture? 8. If a man buys 1 lt of milk for Rs.12 and mixes it with 20% water and sells it for Rs.15, then what is the percentage of gain? 9. Pipe A can fill a tank in 30 mins and Pipe B can fill it in 28 mins.If 3/4th of the tank is filled by Pipe B alone and both are opened, how much time is required by both the pipes to fill the tank completely ? 10. If on an item a company gives 25% discount, they earn 25% profit. If they now give 10% discount then what is the profit percentage. (a) 40% (b) 55% (c) 35%

(d) 30% Ans. D 11. A certain number of men can finish a piece of work in 10 days. If however there were 10 men less it will take 10 days more for the work to be finished. How many men were there originally? (a) 110 men (b) 130 men (c) 100 men (d) none of these Ans. A 12. In simple interest what sum amounts of Rs.1120/in 4 years and Rs.1200/- in 5 years ? (a) Rs. 500 (b) Rs. 600 (c) Rs. 800 (d) Rs. 900 Ans. C 13. If a sum of money compound annually amounts of thrice itself in 3 years. In how many years will it become 9 times itself. (a) 6 (b) 8 (c) 10 (d) 12 Ans A 14. Two trains move in the same direction at 50 kmph and 32 kmph respectively. A man in the slower train observes the 15 seconds elapse before the faster train completely passes by him. What is the length of faster train ? (a) 100m (b) 75m (c) 120m (d) 50m Ans B 15. How many mashes are there in 1 squrare meter of wire gauge if each mesh is 8mm long and 5mm wide ?

(a) 2500 (b) 25000 (c) 250 (d) 250000 Ans B 16. x% of y is y% of ? (a) x/y (b) 2y (c) x (d) can't be determined Ans. C 17. The price of sugar increases by 20%, by what % should a housewife reduce the consumption of sugar so that expenditure on sugar can be same as before ? (a) 15% (b) 16.66% (c) 12% (d) 9% Ans B 18. A man spends half of his salary on household expenses, 1/4th for rent, 1/5th for travel expenses, the man deposits the rest in a bank. If his monthly deposits in the bank amount 50, what is his monthly salary ? (a) Rs.500 (b) Rs.1500 (c) Rs.1000 (d) Rs. 900 1. Mrs. Jones, Mrs. Davis and Mrs. Smith are Friends. Their first names are Mary, Helen and Dorothy though not in order. All three of them go on shopping, where Marry spends twice of what Helen spent and Helen spends thrice of what Dorothy Spends. Mrs. Davis spends 3.85 more than Mrs. Smith. Identify the Families of these three women. 2. A cube of size 10 units, painted black, is cut into 1000 pieces of 1 unit size. How many of the cubes are painted (At least on one side) 3. A Person gets his old book binded. But he finds that the page numbers are cut off. So he starts numbering the pages. During the process he finds that 3 is encountered 61 times. Can u tell how many pages were

there in the book. 4. A child questions his father, "What is your father's age", to which the father answers "He was aged X in year X2", If the context is 20th Century, what is his Date of Birth. 5. A Clock loses exactly 24 minutes per hour. The Clock is now showing 3.00 A.M. If the clock was corrected a midnight and it stooped 1 hour ago, what is the correct time now. 6. A boy will cycle his way to his grandfather's house. On the first day he cycles of the distance. On the second day he cycles of he remaining distance. On the third day he cycles three quarter of the remaining. On the fourth day he cycles 10 miles. On the fourth day he cycles of the remaining distance and on the final day he cycles the remaining 5 miles. What is total distance he traveled? 7. A rich man wanted to distribute his collection of gold coins among his ten children comprising of 5 sons and 5 daughters. He also wanted to give some coins to his car driver. He proceeds on the following basis. The first coin went to the car driver and then 1/5th of the remaining coins to his first son. He then gave another coin to the car driver and distributed 1/5th of the remaining coins to his second son. This procedure went on till all the five sons received their share. Once all the sons received their shares, he distributed the remaining coins equally among his daughters. Can u find how many gold coins the rich man had? 8. ( A Lengthy Question About 6 Football teams each making 2 predictions each. The team finishing last has both its predictions true and the one finishing 5th has one of its predictions true and the other false. The rest of the teams have their predictions false. If one of the teams cannot finish last give the team positions. The predictions are something like that team will finish in that position this position) 9. L, M, N, R, S are single digit numbers which satisfy the arithmetic shown below. Find the values of L M N X. LM N LMN

+ R S SXR

RS TUVW

1. Ava, Fanny, Eartha, Cynthia, Isabelle and Rick, Smith, Vic, Steve, Willy are 5 Couples though not in order. Each were married on the different day of the week from Monday to Friday. Ava was married on Monday but not to willy Steve was married on Thursday and Rick on Friday

(but not to Isabelle) Fanny married Vic on the day after Eartha was married. Identify the Couples. 1. Five people A ,B ,C ,D ,E are related to each other. Four of them make one true statement each as follows. ( 8 marks) (i) B is my father's brother. (ii) E is my mother-in-law. (iii)C is my son-in-law's brother (iv)A is my brother's wife. who made these statemens and what are the relationships among them? Ans: (i) D (ii) B (iii) E (iv) C 2. fathers wife is reverse of son`s age . one year back faters age was twice of son`s age . what`s the fathers current age ans : 73 3. An escalator is descending at constant speed. A walks down and takes 50 steps to reach the bottom. B runs down and takes 75 steps in the same time as A takes 3 steps. How many steps are visible when the escalator is not operating? Ans: ???? 4. a man asks a weatherman what`s the past five days temp? he says i didn`t remember but i can say their product is 12 and all are diff temperatures. what are the five temperatures? ans: -2,-1,1,2,3

5: there are 100 men among them 80 have telephones and 70 have cars and some 75 have houses and 85 have mobiles.(data is not exact) . what is the minimum number of men that have all these things.? 6: there ia truck which should reach some place at 11`o clock , if it travels with 30 mph it reaches i hour before , if it travles with 20 mph it reaches 1 hour late. what is the distance it must be travlled and what is the speed it must maintain to rech at exact time? ans: 120 miles and 24 mph 7: there are some stones (may be 6231) with eqaul weight, but one of them having more weight than others,how many times we need to weigh to find that overweigheted stone? ans: 12 ( as per the number given there, that no of stones i didn`t remember exactly). 8:two and half artists with two and half canvases and two in two and half hours cvan paint two and half drawings. how many artists are needed for 24 canvases and 24 drawings in 20 hours? The highest Score in an innings was 3/11 of the total and the next highest was 3/11 of the reminder. If the scores differed by 9, find the total score. a) 151 b) 161 c) 121 a d) 101 e) 137 Ans: c A boy was asked to multiply a certain number by 53. He multiplied it by 35 and got his answer less than the correct one by 1206. Find the number to be multiplied. a) 37 b) 67 a c) 87 d) 97 e) 107 Ans: 67 A Problem like this not exactly the same but on same model. If the manufacturer gains 10 %, the wholesale dealer 15 % and the retailer 25 % then the cost of Production of a table, if the retail price is Rs.1265 a) 632.50 b) 800 c) 814 d) 834.24 Ans: b A trader marks his goods up by 50% and declares two successive discounts of 20% each. What is his overall gain? a) 10% gain b) 4% gain c) 4% loss (A) d) 10% loss e) No loss No gain Train Problems - 3 as finding speed of train, how many secs it will cross each other etc. Age problems -3 , find mothers age,fathers age etc. Statistics Problems(probability) as A-this much , B-This Much, A n B-this much and A u B - ? Angle Problems which deals with triangle - 3 problems. Section III - Logical reasoning - 30 qns in 30 mins.

Tower of Hanoi Problem - Ans : 7 problem on fig. find the shaded area, square of size 14cm.. ans: 42 problem on symmetric fig ;; ans less than the 1172(check) . Blood Relation Problems - 3 Then a passage is given on blood relation and 4 qns asked how many male: ans -4 and few etc. Find odd man out of the given series - 3 problems pretty simple put A-Z and number them 1-26 and then u can find easily odd man out. Find the missing letter - was a bit tough to establish relation - 3 problems u have 3 figure -based on the figures ques asked.. easy one.. eg: Fig Triangle means Strong Fig Square means Tall Fig Circle means Fair. Ques asked is find the number that represents Strong Tall and Fair persons.. easy one .. ANs : 4 ANother question Women,Sub inspector,graduates - Ans: 3 An Flow Chart - GIven Units - 250 find final Value with formulae - (units*1.25) and few calculations like this Ans:506.25 And two tedious flow charts couldn't remember them and the answers.

1. if M person r buying a thing costing D$ each,, if 3 person get away,, how much each person has to spend so that total expenditure is same ??? 2. which is smaleest? a. 1/7 b 1/8 c 2/9 d 3/13 ...or something similar having denom. as 11 and 13 in option "d" and "e" 3. if Rix can collect 45 pieces in 1 min.. and Rax take 1 and half minute for same,, what is time require d to colloect 300 pieces when both working togetthre?? 4. one long quiz followed by 5 quest... 3 cages having 3 tags on them, sum of digit of cage num not to exceed 10, and other cond... 5. one quest of grandfather-father-son type quest.. 6. A is shortr than B but taller than E, D is tallest, C is just shorter than A, who is shorttest or similar quest...? 7. rectangualr box,, 25*20*2 converted in to cylinder of dia 10... what is height in terms of "pai"

section 2: series... 1. find the numer for ? 2. find the numer for ? 3 1,3,4,8,15,27,? 4. 2,5,9,19,37,? 5. these were reapeat from old quest... section 3. simple reasoning,4-5 quest. 1 quest followed by 3 quest. 1 quest followed by 3/4 quest. other quiz... section 4. 2flowchart .. 2nd was, A & B drawing ball. 2 ball Red & yellow,, if red is the drawen ball 100 points r awaredrd,otherwise 200 points awareded... if A or B score 500,, their points r forfieted,, if A or B score 1000 or more, he is winner... four quest based on this puzzle, given as missing ? in flowchart section 5. RC related to turnover of companies etc...(8 quest) section 6 data sufficiency 1. whether n is odd? a. m+5n is odd b. 2m+9n is odd these quest were easy, but u required speed to attend all1. There is 66x33m rectangular area. Ram is 11/8 times faster than Krishna. Both of them started walking at opposite ends and they met at some point then, Ram said "See you in the other end" Then they continued walking. After some time Ram thought he will have tea so he turned back walked back 15 meters then he changed his mind again and continued walking .How much Krishna has traveled by the time they meet? 2. There are 5 burglars and once went to a bakery to rob it obviously The first guy ate 1/2 of the total bread and 1/2 of the bread The second guy ate 1/2 of the remaining and 1/2 of the bread. The third guy ,fourth guy and fifth guy did the same. After fifth guy there is no bread left out. How many bread are there?

3. The no. of children adults . The no .of adults the no .of boys . The no. of boys no. of girls .The no. of girls no. of family conditions 1. No family is without a child 2. Every girl has at least one brother and sister . 4. All members belonging to D are members of A do........................E.................D do........................C.................both A&D Some members of A does not belong to D do...............D......................E 5 questions are there.

5. Write each statements true or false 1.The sum of the1st three statements and the 2nd false statement gives the true statement. 2.The no. of true statements false statement 3. The sum of 2nd true statement and 1st false statement gives the first true statement. 4. There are at most 3 false statements 5.There is no two consecutive true statements 6.If this contains only 1-5 statements, the answer of this is same as the an answer of the following question

6.Question on Venn diagram. All handsome are also fair skinned Some muscular are fair skinned Some muscular are also handsome All lean are also muscular Some lean are also fair skinned. All rich man is not fair skinned but all rich man are handsome 7. There are twelve consecutive flags at an equal interval of distance. A man passes the 8th flag in 8 seconds. How many more seconds will he take to pass the remaining 4 flags ?

8. A person has to cover the fixed distance through his horses. There are five horses in the cart. They ran at the full potential for the 24 hours continuously at constant speed and then two of the horses ran away to some other direction. So he reached the destination 48 hours behind the schedule. If the five horses would have run 50 miles more, then the person would have been only 24 hours late. Find the distance of the destination. 9. A boat M leaves shore A and at the same time boat B leaves shore B. They move across the river. They met at 500 yards away from A and after that they met 300 yards away from shore B without halting at shores. Find the distance between the shore A & B.

10. A person was going through train from Bombay to Pune. After every five minutes he finds a train coming from opposite direction. Velocity of trains are equal of either direction. If the person reached Pune in one hour then how many trains he saw in the journey ? 11. Food grains are to be sent to city from godown. Owner wants to reach the food grains at 11 O' Clock in the city. If a truck travels

at a speed of 30km/hr then he will reach the city one hour earlier. If the truck travels at a speed of 20km/h then he will reach the city one hour late. Find the distance between the godown to city. Also with which speed the truck should travel in order to reach at exactly 11 'O clock. 12. There are five persons A,B,C,D,E whose birthdays occur at the consecutive days. Birthday of A is some days or day before C & birthday of B is exactly the same days or day after E. D is two days older than E. If birth day of C is on Wednesday then find out the birthdays of other. 13 persons say these statements. A says either Democratic or liberal wins the elections. B says Democratic wins.C says neither democratic nor liberal wins the election. Of these only one is wrong. who wins the election? 14. Six persons A,B,C,D,E &F went to soldier cinema. There are six consecutive seats. A sits in the first seat followed by B, followed by C and so on. If A taken on of the six seats, then B should sit adjacent to A. C should sit adjacent to A or B. D should sit adjacent to A, B or C and so on. How many possibilities are there ? 15. Suppose there are four grades A, B, C, D. (A is the best and D is the worst) 4 persons Jack, Jean, Poul and Lucy wrote the final exam and made the statements like this 1. Jack: If I will get A then Lucy will get D 2. Lucy: If I will get C then Jack will get D. Jack grade is better than Poul grade. 3. Jean: If Jean doesn't get A then Jack will not get A 4. Poul: If Jack get A, then Jean will not get B, Lucy will get C, I won't either A or B If all the above statements are true, then which person will get which grade.

16. Each man dances with 3 women, Each women dances with 3 men. Among each pair of men they have exactly two women in common. Find the no. of men and women.

17. A survey was taken among 100 people to find their preference of watching t.v. programmes. There are 3 channels. Given no of people who watch at least channel 1 " " 2 " " 3 no channels at all

atleast channels 1and 3 " " 1 and 2 " " 2 and 3 find the no of people who watched all three.

18. Albert and Fernandes have two leg swimming race. Both start from opposite and of the pool. On the first leg, the boys pass each other at 18 mt from the deep end of the pool. During the II leg they pass at 10 mt from the shallow end of the pool. Both go at const speed. But one of them is faster. Each boy rests for 4 sec to see at the end of the I leg. What is the length of the pool.

19. A bird keeper has got P pigeon, M mynas and S sparrows. The keeper goes for lunch leaving his assistant to watch the birds. suppose p=10, m=5, s=8 a.) when the bird keeper comes back, the assistant informs that x birds have escaped. The bird keeper exclaims oh no! all my sparrows are gone. How many birds flew away. b.) when the bird keeper come back, the assistant told him that x birds have escaped. The keeper realised that atleast 2 sparrows have escaped. What is minimum no of birds that can escape. 20.Select from the five alternatives A,B,C,D,E At the end of each question , two conditions will be given. The choices are to filled at follows. a. if a definite conclusion can be drawn from condition 1 b. if a definite conclusion can be drawn from condition 2 c. if a definite conclusion can be drawn from condition 1 and 2 d. if a definite conclusion can be drawn from condition 1 or 2 e. no conclusion can be drawn using both conditions 1. person 1 says N<5 person 2 says n>5 person 3 says 3N>20 person 4 says 3n>10 person 5 says N<8 what is the value of N a) 1. no of persons who speak false being less than no of persons who tells the truth. 2. person 2 is telling the truth. b) 1. no of persong telling the truth is greater than no of persons telling lies 2. person 5 is telling the truth.

21. There are N coins on a table. There are two players A&B. You can take 1or 2 coins at a time. The person who takes the last coin is the loser. A always starts first --1. if N=7 a) A can always win by taking two coins in his first chance b) B can win only if A takes two coins in his first chance. c) B can always win by proper play d) none of the above --2. A can win by proper play if N is equal to a) 13 b) 37 c) 22 d) 34 e) 48 ans. e. --3. B can win by proper play if N is equal to a) 25 b)26 c) 32 d) 41 e) none --4. if N<4, can A win by proper play always

22. There are 4 parties A,B,C,D. There are 3 people x,y,z. X-says A or D will win. Y-says A will not win. Z-says B or D will not win. Only one of them is true. Which party won. 23.5. R,S,T,U,V are contesting for a medal. Evaluation is over, sasy English, Maths, Physics, Chemistry and Hindi. Toper will get 5 marks, least will get 1mark. No ties any where. R get 24 and won the overall medal. V-get I st in Chemistry and 3 rd in Hindi, T got consistent scores in 4 subjects. Their final standings where in the alphabetical order. What was the score of S in Chemistry. 24. There are 3 types of castes, say A,B,C. A- always tells truth, Balways false, C- alternating. X says --> Z is of C type, I am of A type Y says --> X is a B type. Z says --> X is of B type. Who is of which type. 25. There are two balls touching each other circumferencially. The radius of the big ball is 4 times the diameter of the small ball.The outer small ball rotates in anticlockwise direction circumferencially over the bigger one at the rate of 16 rev/sec.The bigger wheel also rotates anticlockwise at Nrev/sec. what is 'N' for the horizontal line from the centre of small wheel always is horizontal. 26. There are 2 guards Bal and Pal walking on the side of a wall of a wearhouse(12m X 11m) in opposite directions. They meet at a point and Bal says to Pal "See you again in the other side". After a few moments of walking Bal decides to go back for a smoke but he changes his direction again to his previous one after 10 minutes of walking in the other (opposite) direction remembering that Pal will be waiting for to meet. If Bal and Pal walk 8 and 11 feet respectively,

how much distance they would have traveled before meeting again. 27. xxx)xxxxx(xxx 3xx ------xxx x3x -----xxx 3xx Find the 5 digit No. Hint: 5 is used at least once in the calculation.

28. A fly is there 1 feet below the ceiling right across a wall length is 30m at equal distance from both the ends. There is a spider 1 feet above floor right across the long wall equidistant from both the ends. If the width of the room is 12m and 12m, what distance is to be traveled by the spider to catch the fly? if it takes the shortest path.

27. Ramesh sit around a round table with some other men. He has one rupee more than his right person and this person in turn has 1 rupee more than the person to his right and so on, Ramesh decided to give 1 rupee to his right & he in turn 2 rupees to his right and 3 rupees to his right & so on. This process went on till a person has 'no money' to give to his right. At this time he has 4 times the money to his right person. How many men are there along with Ramesh and what is the money with poorest fellow. 28. Venkat has 1 boy & 2 daughters. The product of these children age is 72. The sum of their ages give the door number of Venkat. Boy is elder of three. Can you tell the ages of all the three.

29. L:says all of my other 4 friends have money M:says that P said that exact one has money N:says that L said that precisely two have money O:says that M said that 3 of others have money. P:Land N said that they have money. All are liers.Who has money & who doesn't have? 30. Post man has a data of name surname door no.pet name of 4 families. But only one is correct for each family. There are a set of statements & questions.

31. couples have a party. Depending on the set of statements, find who insulted whom and who is the host of the party.

32. A person has to go both Northwards & Southwards in search of a job. He decides to go by the first train he encounters. There are trains for every 15 min both southwards and northwards. First train towards south is at 6:00 A.M. and that towards North is at 6:10 .If the person arrives at any random time, what is the probability that he gets into a train towards North.

33. A person has his own coach & whenever he goes to railway station he takes his coach. One day he was supposed to reach the railway station at 5 O'clock. But he finished his work early and reached at 3 O'clock. Then he rung up his residence and asked to send the coach immediately. He came to know that the coach has left just now to the railway station. He thought that he should not waste his time and started moving towards his residence at the speed of 3miles/hr. On the way, he gets the coach and reaches home at 6 o'clock. How far is his residence from railway station.

34. Persons A and B. Person A picks a random no. from 1 to 1000.Then person B picks a random no. from 1 to 1000. What is the probability of B getting no. greater then what A has picked. 35. Three boys and three girls brought up together. Jim, Jane, Tom, Virgina, Dorthy, XXX. They marry among themselves to form three couples. Conditions are : i) Sum of their ages would be the same. ii) Virgina was the oldest. iii) Jim was dorthy's brother. iv) Sum of ages Jane+Jim and Tom+dorthy is same. Give the three couples. 36. X^(1/3) - X^(1/9) =60 Solve for X.

37. X Z Y+X Y Z = Y Z X Find the three digits. 38.. Two boats start from opposite banks of river perpendicular to the shore. One is faster then the other. They meet at 720 yards from one of the ends. After reaching opposite ends they rest for 10mins each. After that they start back. This time on the return journey they meet at 400yards from the other end of the river. Calculate the width of the river.

39. Basketball Tournament organizers decided that two consecutive defeats will knock out the team. There are 51 teams participating. What is the maximum no. of matches that can be played.

40. The Master says to his grandmaster that me and my three cousins have ages in prime nos. only. Summation of our ages is 50. Grandmaster who knows the age of the master instantly tells the ages of the three cousins. Tell the ages of three cousins.( 1 is not considered as prime no.) 41. There are two families Alens and smiths. They have two children each. There names are A,B,C,D whose ages are different and ages are less then or equal to 11. The following conditions are given. i) A's age is three years less then his brother's age . ii) B is eldest among the four. iii) C is half the age of the eldest in Alens family. iv) The difference in sum of the ages of Alens children and smiths children is same as that of five years ago. Find the ages of all the children.

42. a,b,c,d,e are having numerical values. There are some conditions given a) a=c <=== b!=e b) difference between a and c as same as difference between c and b as same as difference between a and d c) c<a and cd Then find a,b,c,d,e 43. There are six cards in which it has two king cards. all cards are turned down and two cards are opened a) what is the possibility to get at least one king. b) what is the possibility to get two kings. 44. There are 5 persons a,b,c,d,e and each is wearing a block or white cap on his head. a person can see the caps of the remaining 4 but can't see his own cap. a person wearing white says true and who wears block says false. i) a says i see 3 whites and 1 block ii) b says i see 4 blocks iii) e says i see 4 whites iiii) c says i see 3 blocks and 1 white. Now find the caps weared by a,b,c,d and e 45. There are two women, kavitha and shamili and two males shyam, aravind who are musicians. out of these four one is a pianist, one flutist, violinist and drummer. i) across aravind beats pianist ii) across shyam is not a flutist iii) kavitha's left is a pianist iiii) shamili's left is not a drummer v) flutist and drummer are married. 46. When Arthur is as old as his father Hailey is now, he shall be 5

times as old as his son Clarke is now. By then, Clarke will be 8 times older than Arthur is now. The combined ages of Hailey and Arthur are 100 years. How old is Clarke ? 47. The seven digits in this subtraction problem are 0,1,2,3,4,5 and 6. Each letter represents the same digit whenever it occurs. D A D C B E B E G -------------------B F E G -------------------What digit is represented by each letter ? 48. The Jones have named their four boys after favorite relatives; their friends, the Smiths, have done the same thing with their three boys. One of the families has twin boys. From the following clues, can you determine the families of all seven children and their ages ? i) Valentine is 4 years older than his twin brothers. ii) Winston, who is 8, and Benedict are not brothers. They are each named after a grandfather. iii) Briscoe is two years younger than his brother Hamilton, But three years older than Dewey. iv) Decatur is 10 years old. v) Benedict is 3 years younger than Valentine; they are not related. vi) The twins are named for uncles. 49. Motorboat A leaves shore P as B leaves Q; they move across the lake at a constant speed. They meet first time 600 yards from P. Each returns from the opposite shore without halting, and they meet 200 yards from. How long is the lake ? 50. Fanta, Pepsi and Citra often eat dinner out. a) Each orders either coffee or tea after dinner. b) If Fanta orders coffee, then Pepsi orders the drink that Citra orders. c) If Pepsi orders coffee, then Fanta orders the drink that Citra does not order. d) If Citra orders tea, then Fanta orders the drink that Pepsi orders. Which person/persons always orders the same drink after dinner ? 51. On the Island of imperfection there is a special road, Logic Lane, on which the houses are usually reserved for the more mathematical inhabitants. Add, Divide and Even live in three different houses on this road (which has houses numbered from 1-50). One of them is a member of the Pukka Tribe, who always tell the truth; another is a member of the Wotta Tribe, who never tell the truth; and the third is a member

of the Shalla Tribe, who make statements which are alternately true and false, or false and true. They make statements as follows : ADD: 1 The number of my house is greater than that of Divide's. 2 My number is divisible by 4. 3 Even's number differs by 13 from that of one of the others. DIVIDE : 1 Add's number is divisible by 12. 2 My number is 37. 3 Even's number is even. EVEN : 1 No one's number is divisible by 10. 2 My number is 30. 3 Add's number is divisible by 3. Find to which tribe each of them belongs, and the number of each of their houses. 52. The names of the inhabitants of Walkie Talkie Land sound strange to the visitors, and they find it difficult to pronounce them, due to their length and a few vowel sounds they contain. The Walkie Talkie guide is discussing the names of four inhabitants - A,B,C and D. Their names each contain upto eight syllables, although none of the four names contain the same number. Two of the names contain no vowel sounds; one contains one vowel sound; and one contains two vowel sounds. From the Guide's statements below, determine the number of syllables and vowel sounds in each of the four Walkie Talkie names : i) The one whose name contains two vowel sounds is not A. ii) C's name does not contain more than one vowel sound or fewer than seven syllables. iii)The name with seven syllables does not contain exactly one vowel sound. iv) B and C do not have names with the same number of vowel sounds. v) Neither the name with five syllables nor the name with seven syllables contains more than one vowel sound. vi) Neither the name with six syllables, nor the B's name, contains two vowel sounds. 53. Multiplication and Division. (1) - - - - - (a) (2) __._ (d)

________________ - - - (b) - - -) - - - - - . 0 ---

_________________

_______________ --------___________________ ------

_______________ - - - - - - - (c) ----======== (1) and (2) represent the same two numbers - in one case multiplied 9 together, in other case divided. Given that there are no 3's. find the missing digits indicated as (a), (b), (c) and (d) above ? 54. Two identical twins have a very unusual characteristic. One tells nothing but lies on Mondays, Wednesdays and Fridays, and tells nothing but the truth all other days. The other tells nothing but lies on Tuesdays, Thursdays and Saturdays, and tells nothing but the truth all other days. On Sundays both children speak the truth. 55. Which of the following statements can be deduced from the information presented ? i) If it is Sunday, the twins will both say so. ii) If it is not Sunday,one twin will give the correct day and the other will lie about everything. iii)On any given day, only one twin will give his correct name. a) i only b) i and ii only c) i and iii only d) ii and iii only e) i,ii and iii 56. According to the information presented, which of the following conversations will be impossible. a)Twin A : "Today you are a lier" Twin B : "You are telling the truth" b)Twin A : "Today you are a lier" Twin B : "Today I am a truthteller" c)Twin A : "Tommorow I shall be a lier" Twin B : "That's correct" d)Twin A : "Tommorow you will be a lier" Twin B : "Today you are a truthteller" e)Twin A : "Yesterday we were both truthtellers" Twin B : "You are lying" 57. Assume that the twins followed a different set of rules, so that on a given day both told only the truth while next day both only lied, alternating days of truth telling and lying. Under these rules,which of the following conversations would be possible ? a) Twin A : "Today you are a lier" Twin B : "That is correct" b) Twin A : "Today you are a lier"

Twin B c) Twin A Twin B d) Twin A Twin B e) Twin A Twin B

: : : : : : :

"That is not so" "Tommorow we will be liers" "Yesterday we were truthtellers" "Tommorow we will be liers" "You are 1 year older than I am" "We always tell the truth" "We some times tell the truth"

58.If the twins are heard saying the following on the same day, which choice presents a correct statement ? Twin A : "It is Sunday Today" Twin B : "Yesterday was Sunday" Twin A : "it is summer season now" a) it is a summer sunday b) it is a summer monday c) it is Monday but not summer d) it is Sunday but not summer e) it is impossible to determine whether it is Sunday or Monday 1.explosion:debris:: flood:water fire:ashes 2.bandage:wound:: cast:fracture glove:hands diaper:baby 3.resolved:doubt:: suggest:idea confirm:suspicion 4.5.6.7.8. antonyms for assuage,sedulous,cogent,scrupulous,jeopardy 9.celebrate:marriage:: face:penalty 10.error correction; due to his carelessness,the form of the the compiler can assimilate because of his carelessness,the form of what ......... due to his carelessness,the form of the what .......... because of his carelessness,the form of that what ........... 13-24 LOGICAL REASONING 13.six palys ABCDEF are to be held in 6 consecutive days from 17 to 22.only one play per day.E is on 22nd.f between B & D.A and C are held after B and D data is different from what the data is different from data is different from that the data is different from

respectively. if B is to be held on 17th what is the minimum date in which D can take palce ANS:19th 14.15. two more questions similar to previous 16.alphabets are coded using special symbols.there is a code which has symbols as follows +++***#&%% ##$++%@@($.the alphabets which are coded AEOBGHKM.here G occurs 5 times while B,K-2 times A,M-2 times.also B is coded as # and O as %.what is the code for K? ANS:* 17.A on seeing the figure in the picture tells that "her mother has only one son whose daughter is my cousin.my cousin's father is my maternal uncle who has only one sister". how is A to the person in the picture ANS:A's mother 18.sushil is son of a teacher.mohan is a teacher ans has son.mohan is the father of sushil definitely true,probably true,cannt say,probably false 19.3415 refers to meet me on sunday.538 not on sunday. 861 why not meet.what is the code for 'on' ANS:cannt be determined some more questions on passage reading,reading comprehension

25-60 MATHS 25.if it is 250kms fro mdelhi to chandigarh ans 120 from delhi to jaipur,what % of distance from delhi to chandigarh si that of delhi to jaipur ANS:48 26.old picture has dimension 33 inches by 27 inches.what length one must cut from each dimension so that ratio of shorter to longer side is 2:3 ANS:6 inches 27. in a bookstore all books are sold for same price.a+b books are selling for 1000Rs. Myron buys 12 books and for each book one has discount of 1Re how much Myron need to pay ANS:[(1000/(a+b))-1]*12

28.the value of B in sequence a=(h/2)(B+b) ANS:2a/h-b 29.in a class of 40,30 speak french and 20 german.what is the lowest possible no of students who speak both. ANS:10 30.a truck leaves its station averaing 40 mph.a car leaves from same place after 24mins.if car averages 50mph, how long will it take to catch the truck ANS:1.6 hours 31.an amount becomes 826Rs. in 3 years at some rate of interest.if the rate is increased by 0.5times amount will be 889Rs. for the same time.what is the initial amount ANS:Rs700 32.in a wood of 10 pounds, only 8 pounda remains after drying.what is the percentage of moisture 33.a cylinder is circumscribed over the hemisphere and a cone is inscriebd inside the cyclinder in such a way that its base is also the cylinders base and apex at center point of another side.what is the ratio of volume of cylinder,hemisphere,cone ANS:3:1:2 34. ABCD is a rectangle with AB as length and BC as breadth.a right angled triangle is cut in such a manner that its height is same as breadth and base 1/4th of length of rectangle if area of triangle is 12sq.m. what is the area of remaining portion ANS:84sq.m. 35. 3 pounds of seedA has 0.05% herbicide while another seedB has 20% herbicide.how much pounds of B should be mixed with A so that mixture has 15% herbicide ANS:6pounds 36.in a meeting there are 66 hansshakes.how many people are there in meeting ANS:12 37.a tank has capacity of 3750 cubic metre.thro a pipe 800 cubic metre is pumped every minute and 300 cubic metre is leaked out thro hole.how

long it will take to fill the tank?? ANS:7.5min 38.a man can buy 50 books and pay money in 3 months hence.if he has to buy 51 books for same price whats the rate of interest? 8,10,12,14 % for the year 39. what is the least number which is to subtracted from 10421 so that the results become a perfect square ANS:15 40. 12 men can do a work in 4 days.how long it will take for 15 men to finish the work ANS:3.2days 41. two pipes can fill tank in 10,12 hours and another pipe can empty the tank in X hours.if three pipes are opened simultaneously it will take 20 hours to fill.wats the valus of X ANS:10hrs 42. 3litre sugar solution contains 40% of sugar.if one more litre of water is added to the solution what is the % of sugar ANS:30% 43. 30% of people contribute 40Rs. 45% Rs.20 and remaining 12Rs.what is the % of 40 Rs. contributors in total contribution? ANS:50% 44. ratio of area of circle is 4:9 what is ration of their circumference ANS: 2:3 45. what is the percentage of numbers between 1 to 50 such that unit digit of their squares is one ANS:20% 46. average rain for march to august is 5cm,for september it is 26cm and for october it is 40cm.what is the overall average fron march to october ANS:12 47. 0.05 times 0f 0.03 is ANS:0.15% 48. a man bought a set for 20% discount and sold it for Rs.800 profit for a price of 16,800.what is the original cost price

ANS:20000 49. if the rate of interest in compounded,then in how many year the amount will be more than doubled ANS:4yrs 50. if Re.1 is doubled every 6 months how long it will take to become 81 ANS:2yrs 51. what is common factor for x^2-4x-5 and x^2-6x-7 ANS:x+1 52. if angles of a traingle are of ratio 3:5:7 then the traiangle is ANS:obtuse angled 53. if A=1,B=2.....Z=26.what is the valus for P+Q+R ANS:51 54. in the series -9,-6,-3,0.... how many terms are required so that their sum is 66 ANS:11 55. to print a page it costs Rs.x for firat 1000 pages and Rs.y for further.what is the cost for printing z pages which is greater than 1000 ANS:1000(x-y)+zy 56. one clock is 2 minutes faster while another is 1 minute slow.both are corrected at noon.what will be difference in the time shown by these clocks at midnight ANS:36min 57. a clock now shows 12PM and it gains 15minutes every day what will the clock show at 4AM tommorrow morning ANS:4:10

Vous aimerez peut-être aussi